Unit 6
Unit 6
Serial Page
UNIT - 6
MATHEMATICS
No. No.
1. Function 1–62
AL Þ Topics required for Advance level preparations useful for competitive exams.
4. Golden Key Points : Important points/formulaes or concepts summarized at the end to have a
quick revision of the topic.
EXERCISE
ETBD (ESSENTIALLY TO BE DISCUSSED IN CLASS)
6. Exercise-1 : Collection of misc. objective problems with single correct to enhance speed &
accuracy during problem solving. [Targeted towards JEE-Mains]
7. Exercise-2 : Collection of misc. objective problems with one or more than one correct
options and comprehension based objective problems to test analytical, reasoning &
comprehension skills as per latest JEE pattern.[Targeted towards JEE-Advance]
8. Exercise-3 : Collection of Numerical answer based and Matrix Match questions to test
analytical, reasoning & problem solving skills as per latest JEE pattern.[Targeted towards
JEE-Advance]
9. Exercise-4A : Questions from 2008-2019 & 2020 (first attempt) AIEEE/JEE–mains both
offline & online. [Fundamental level]
10. Exercise-4B : Questions from 2008-2019 & 2020 (first attempt) AIEEE/JEE–mains both
offline & online. [Advance level]
In earlier classes, we have studied about sets, a collection of elements. We also had a brief idea about functions
and relations. In this chapter, we will systematically discuss “Functions”. Their properties and
application will lead us into further understanding differential and integral calculus and their wider practical
applications.
FUNCTION
This chapter deals with establishing binary relation between elements of one set and elements of another set
according to some particular rule of relationship.
The Cartesian product of two sets A, B is a non-void set of all ordered pair (a, b) where a Î A and b Î B. This
is denoted by A × B
\ A × B = {(a, b) " a Î A and b Î B}
e.g. A = {1, 2} B = {a, b}
A × B = {(1, a), (1, b), (2, a), (2, b)}
Note :
(i) A × B ¹ B × A (Non-commutative)
(ii) n (A × B) = n(A) n(B) and n(P(A × B))= 2n(A)n(B)
(iii) A = f and B = f Û A × B = f
(iv) If A and B are two non-empty sets having n elements in common, then (A × B) and (B × A) have n 2
elements in common.
(v) A × (B È C) = (A × B) È (A × C)
(vi) A × (B Ç C) = (A × B) Ç (A × C)
(vii) A × (B – C) = (A × B) – (A × C)
Illustration 1. If n(A) = 7, n(B) = 8 and n(A Ç B) = 4 then match the following column-1 with column-II
Column-1 Column-II
1
JEE-Mathematics
2.0 RELATION
SL AL
Illustration 4. A = {Jaipur, Patna, Kanpur, Lucknow} and B = {Rajasthan, Uttar Pradesh, Bihar}
aRb Þ a is capital of b, a Î A and b Î B
Solution R = {(Jaipur, Rajasthan), (Patna, Bihar), (Lucknow, Uttar Pradesh)}
1. Roster form : In this form we represents set of all ordered pairs (a, b) suc that (a, b) Î R
where a Î A, b Î B
JPR\COMP.251\Allen(IIT-JEE Wing)\2020–21\Enthusiast\Mathematics\Unit - 6
2. Set builder notation : Here we denote the relation by the rule which co relates the two set.
3. Arrow-diagram (Mapping) : This is the pictorial notation of any relation.
Ex. Let A = {–2, –1, 4}, B = {1, 4, 9}
A relation from A to B i.e. a R b is defined as a is less than b.
This can be represented in the following ways.
1. Roster form :
R = {(–2, 1) (–2, 4), (–2, 9), (–1, 1), (–1, 4), (–1, 9), (4, 9)}
2. Set builder notation :
R = {(a, b) : a Î A and b Î B, a is less than b}
2
Fu ncti on
3. Arrow-diagram
A B
–2 1
–1 4
4 9
If relation R is defined from A to B, then the inverse relation would be defined from B to A, i.e.
R:A®B Þ aRb where a Î A b Î B
R–1 : B ® A Þ bRa where a Î A, b Î B
Domain of R = Range of R–1
and Range of R = Domain of R–1
\ R–1 = {(b, a) : (a, b) Î R}
A relation R is defined on the set of 1st ten natural numbers
e.g. N is a set of first 10 natural nos. \ N = {1, 2, 3 .......10} & a, b Î N
aRb Þ a + 2b = 10
R = {(2, 4), (4, 3) (6, 2), (8, 1)}
R–1={(4, 2), (3, 4), (2, 6), (1, 8)} N N
A relation defined on a set A is said to be an identity relation if each and every element of A is related to itself
and only itself
e.g. A relation defined on the set of natural numbers is
aRb Þ a = b where a & b Î N
R = {(1, 1), (2, 2), (3, 3),......}
R is an identity relation
Types of Relations
(i) Reflexive : A relation R on a set A is said to be reflexive if every element of A is related to itself.
i.e. if (a, b) Î R, then (a, a) Î R. However if there is a single ordered pair of (a, b) Î R such (a, a) Ï R
then R is not reflexive
e.g. A relation defined on (set of natural numbers)
aRb Þ ‘a’ divides ‘b’ a, b Î N
R would contain (a, a) because every natural number divides itself and hence is a reflexive relation.
Note : Every identity relation is a reflexive relation but every reflexie relation need not be an identity.
3
JEE-Mathematics
(3) aRb Þ ‘a’ is brother of ‘b’ is not symmetric relation as a may be sister of b
If R is symmetric
(1) R = R–1
(2) Range of R = Domain of R
(iii) Transitive : A relation on set A is said to be a transitive if aRb and bRc implies aRc
i.e. (a, b) Î R and (b, c) Î R, then (a, c) Î R
and a, b, c need not be distinct
Examples :
(1) A relation R defined on a set of natural numbers N with rule aRb Þ a < b
Let R : {(1, 2), (1, 1)} on set {1, 2}
In this relation a, b, c are not distinct but it is transitive. It is transitive but not symmetric as (2, 1) is
missing. Minimum number of ordered pair that must be added to make it reflexive, symmetric and
transitive is 2 i.e. (2, 1) and (2, 2)
(3) If ‘R’ be a relation defined on the set of integers ‘I’ then, Prove that :
xRy Þ (x–y) is even is an equivalence relation
Sol. (x, x) Î R so reflexive
Let x – y = 2m Þ y – x = –2m, so it is symmetric
x – y = 2m, y – z = 2n Þ x – z = 2(m+n), so It is transitive
(4) R = {(1, 2), (2, 3)} add minimum number of ordered pairs to make it an equivalence relation.
{(1, 1), (2, 2), (3, 3), (2, 1), (3, 2), (1, 3), (3, 1)} = 7
4
Fu ncti on
2. In the set A = {1, 2, 3, 4, 5}, a relation R is defined by R = {(x, y) | x, y Î A and x < y}. Then R is :
(A) Reflexive (B) Symmetric (C) Transitive (D) None of these
3. For real number x and y, we write x R y Û x – y + 2 is an irrational number. then the relation R is :
(A) Reflexive (B) Symmetric (C) Transitive (D) None of these
4. Let L denote the set of all straight lines in a plane. Let a relation R be defined by a R b Û a ^ b, a, b Î L. Then
R is :
(A) Reflexive (B) Symmetric (C) Transitive (D) None of these
5. Let R be a relation over the set N × N and it is defined by (a, b) R (c, d) Þ a + d = b + c. Then R is :
(A) Reflexive only (B) Symmetric only (C) Transitive only (D) An equivalence relation
6. Let L be the set of all straight lines in the Euclidean plane. Two lines l1 and l2 are said to be related by the
relation R if l1 is parallel to l2. Then the relation R is :
(A) Reflexive (B) Symmetric (C) Transitive (D) Equivalence
8.0 FUNCTION
SL AL
A relation R from a set A to a set B is called a function if each element of A has unique image in B.
ƒ
It is denoted by the symbol. ƒ : A ® B or A ¾¾® B which reads ‘ƒ ’ is a function from A to B ‘or’ ƒ maps
A to B,
If an element a Î A is associated with an element b Î B, then b is called ‘the ƒ image of a’ or ‘image of a under
ƒ ‘or’ the value of the function ƒ at a’. Also a is called the pre-image of b or argument of b under the function
ƒ. We write it as b = ƒ (a) or ƒ : a ® b or ƒ : (a, b)
Thus a function ‘ƒ ’ from a set A to a set B is a subset of A × B in which each 'a' belonging to A appears in one
JPR\COMP.251\Allen(IIT-JEE Wing)\2020–21\Enthusiast\Mathematics\Unit - 6
5
JEE-Mathematics
ìx2 x³0
(iv) ƒ(x) = í (non-uniformly defined)
î- x - 1 x<0
y1 y1
x1 x1
y2
Let ƒ : A ® B, then the set A is known as the domain of ƒ & the set B is known as co-domain of ƒ . The
set of ƒ images of all the elements of A is known as the range of ƒ .
Thus Domain of ƒ = {a ½ a Î A, (a, ƒ (a)) Î ƒ }
Range of ƒ = {ƒ (a) ½ a Î A, ƒ (a) Î B}
NOTE :
(i) It should be noted that range is a subset of co-domain.
(ii) If only the rule of function is given then the domain of the function is the set of those real numbers, where
function is defined. For a continuous function, the interval from minimum to maximum value of a function
gives the range
If f & g are real valued functions of x with domain set A, B respectively, f + g, f – g, (f. g) & (f/g) as
follows.
(a) (f ± g)(x) = f(x) ± g(x) domain in each case is A Ç B
(b) (f. g)(x) = f(x).g(x) domain is A Ç B
æ fö f(x)
çè g ÷ø (x) = g(x)
JPR\COMP.251\Allen(IIT-JEE Wing)\2020–21\Enthusiast\Mathematics\Unit - 6
6
Fu ncti on
NOTE
(i) A polynomial of degree one with no constant term is called an odd linear function. i.e. f(x) = ax, a ¹ 0
(ii) There are two polynomial functions, satisfying the relation; f(x). f(1/x) = f(x) + f(1/x).
They are (1) f(x) = xn + 1 and (2) f(x) = 1 – xn, where n is a positive integer.
(iii) Domain of a polynomial function is R
(iv) Range of odd degree polynomial is R whereas range of an even degree polynomial is never R.
y
1 y = 1/x
(ii) y = Domain : R – {0} or R 0
x
o x
Range : R – {0}
1 y = 1/x2
(iii) y = 2 Domain : R0
x
o x
Range : R + or (0, ¥)
y y = x3
(iv) y = x3 1 Domain : R
o x
1
Range : R
g(x)
A rational function is a function of the form y = f(x) = ,
h(x)
where g(x) & h(x) are polynomials& h(x) ¹ 0, Domain – R–{x | h(x)=0}
Any rational function is automatically an algebraic function. y
7
JEE-Mathematics
(e) Constant function
y
f : A ® B is said to be constant function if every element of A has y=c
the same f image in B. Thus f : A ® B ; f(x) = c, " x Î A, c Î
x
B is constant function. Note that the range of a constant function is o
a singleton set.
Domain – R Range – {c}
f(x) = sin x
o p
Domain – R X
f(x) = cos x o p /2 X
Domain : R
(–p ,–1) (p ,–1)
Y
(iii) Tangent function
f(x) = tan x
-p p p/2
–3p/2 O p/2 3 X
(2n + 1)p
Domain – R - ìí x|x = ü
,n Î I ý
î 2 þ
Range – R , period p Y
y=1
(iv) Cosecant function (–3 p/2,1) (p/2,1)
f(x) = cosec x o X
(–p/2,–1)
y=1
f(x) = sec x (–2p ,1) (0,1)
o X
p (-p ,-1) (p , -1)
Domain – R – {x|x = (2n + 1) , n Î I} y=-1
2
8
Fu ncti on
f(x) = cot x
O
Domain – R – {x|x = np, n ÎI} x
p ,0) ( p ,0)
(– 3p ,0) (– ( 3 p ,0)
2 2 2 2
Range – R, period p
x=–2p x=–p x=p x=2p
(g) Exponential and Logarithmic Function
A function f(x) = a x (a > 0), a ¹ 1, x ÎR is called an exponential function. The inverse of
the exponential function is called the logarithmic function, i.e. g(x)= log ax.
Note that f(x) & g(x) are inverse of each other & their graphs are as shown. (If functions are
mirror image of each other about the line y = x)
Domain of a x is R Range R +
Domain of log a x is R + Range R
y y
+¥ +¥
(0,1)
f(x)=a , a Î(0,1)
x
f(x)=a, a>1
x
(0,1)
45°
(1,0) x (1,0) x
x x g(x)=loga x
y= y=
g(x)=logx
a
y y = log2x
y = log3x
1 y = log5x
y = log10x
0 1 x
y y
y=e
x
y=e–x
x x
JPR\COMP.251\Allen(IIT-JEE Wing)\2020–21\Enthusiast\Mathematics\Unit - 6
O O
1
Note-2 – f(x) = log x a = Domain –R + – {1} Range – R – {0}
log a x
(a > 0) (a ¹ 1)
9
JEE-Mathematics
(h) Absolute value function y
The absolute value (or modulus) of a real number y = –x y=x
x (written |x|) is a non negative real number that
satisfies the conditions.
ì x if x ³ 0 x
|x| = í O
î- x if x < 0
Domain : R
The properties of absolute value function are
Range : [0, ¥)
(i) The inequality |x| £ a means that - a £ x £ a ; if a > 0
(ii) The inequality |x| ³ a means that x ³ a or x £ – a if a > 0
(iii) |x ± y| £ |x| + |y| (Triangle Inequality) Equality holds when x .y ³ 0
(iv) |x ± y| ³ ||x| – |y|| (Triangle Inequality) Equality holds when x .y ³ 0
(v) |xy| = |x|.|y|
x |x|
(vi) = , (y ¹ 0)
y |y|
1
Note – f(x)= , Domain : R – {0}, Range : R +
|x|
Illustration 7. If set A has 4 element, set B has 3 elements then how many relation R : A ® B are possible?
Solution Number of subsets of A × B = 24×3
\ Number of relations = 212
æ p2 ö æ3-xö
(a) f ( x ) = 5 sin ç - x2 ÷ (b) f ( x ) = log10 ç ÷
ç 16 ÷ è x ø
JPR\COMP.251\Allen(IIT-JEE Wing)\2020–21\Enthusiast\Mathematics\Unit - 6
è ø
1
(c) f ( x ) = (d) f(x) = sin x + 16 - x2
sin x + sin x
æ æ 1 ö ö
(e) y = log(x–4) (x2 – 11x + 24) (f) f(x) = log2 ç - log1 / 2 çè 1 + 4 ÷ø - 1÷
è x ø
10
Fu ncti on
p2
Solution (a) - x2 ³ 0
16
é p pù
\ x Î ê- , ú
ë 4 4û
æ3-xö æ3-xö
(b) log10 ç ÷ is defined for log10 ç ÷³0
è x ø è x ø
3-x æ 3ù
³1 Þ x Î ç 0, ú ... (1)
x è 2û
æ3-xö 3-x
Also, log10 ç ÷ is defined for >0
è x ø x
Þ x Î (0, 3) ... (2)
æ 3ù
(1) Ç (2) Þ x Î ç 0, ú
è 2û
æ æ 1 ö ö
(f) We have f(x) = log2 ç - log1/ 2 ç 1 + ÷ø - 1÷
è è 4
x ø
æ 1 ö
f(x) is defined if - log1/ 2 ç 1 + 4 ÷ - 1 > 0
è xø
æ 1 ö æ 1 ö -1
or if log1 / 2 ç 1 + 4 ÷ < -1 or if ç 1 + 4 ÷ > (1 / 2)
è xø è xø
1 1
or if 1+ 4
> 2 or if > 1 or if x1/4 < 1 or if 0 < x < 1
x 4
x
\ D(f) = (0, 1)
(i)
æ
f(x) = log2 sin x - cos x + 3 2
çè
ö
÷ø
(ii) f(x) = log 2
(2 - log 2 (16 sin2 x + 1) )
2
æ ö
Solution (i) Let y = log2 sin x - cos x + 3 2
çè ÷ø
2
æ pö
Þ 2y = sin çè x - ÷ø + 3 Þ –1 £ 2y – 3 £ 1
4
Þ 2 £ 2y £ 4 Þ y Þ [1, 2]
11
JEE-Mathematics
1 £ 16 sin2 x + 1 £ 17
\ 0 £ log2 (16 sin2x +1) £ log2 17
\ 2 – log2 17 £ 2 – log2 (16 sin2 x + 1) £ 2
Now consider 0 < 2 – log2 (16 sin2x + 1) £ 2
Illustration 12. Determine the values of x satisfying the equality |x4 – x2 – 6| = |x4 – 4| – |x2 + 2|.
Solution The equality |a – b| = |a|–|b| holds true if and only if a and b have the same sign and
|a|³|b|.
In our case the equality will hold true for the value of x at which x4 – 4 ³ x2 + 2.
Hence x2 – 2 ³ 1; |x| ³ 3.
12
Fu ncti on
æ pö x2
6. (a) f(x) = 2 sin ç 2x + ÷ (b) f(x) =
è 4 ø x4 + 1
x
(c) y = (d) y = ln(3x2 – 4x + 5)
1 + x2
1 æ 1 ö
(a) f(x) = + x+ 2 (b) f(x) = log x log 2 ç
log 10 (1 - x) è x - 1 / 2 ÷ø
|x - 4|
(a) f(x) = (b) f(x) = 5 + 3sin x + 4 cos x
x-4
(c) f(x) = 4 x + 2x + 1 (d) f(x) = log 5
( 2 (sin x - cos x) + 3)
(e) f(x) = x 3 - 12x x Î [–3, 1]
y
(i) Signum function y=1
Signum function y = sgn (x) is defined as follows
ìx ì1 for x > 0
ï ,x ¹ 0 ï o x
y=íx = í0 for x = 0 y = Sgn x
ïî0, x = 0 ï
î-1 for x < 0
Domain – R y = -1
Range – {–1, 0, 1}
x [x] 1
[–2,–1) –2
x
–3 –2 –1 0 1 2 3
[–1,0) –1
–1
[0,1) 0
[1,2) 1 –2
Domain – R –3
JPR\COMP.251\Allen(IIT-JEE Wing)\2020–21\Enthusiast\Mathematics\Unit - 6
Range – I
Properties of greatest integer function
(i) [x] £ x < [x] + 1 and x – 1 < [x] £ x, 0 £ x – [x] < 1
(ii) [x + m] = [x] + m if m is an integer.
ì0, x Î I
(iii)[x] + [–x] = í
î -1, x Î
/I
1 1
Note – f(x) = Domain – R – [0, 1) Range – {x|x = , n ÎI 0 }
[x] n
13
JEE-Mathematics
(k) Fractional part function y
It is defined as – g(x) = {x} = x – [x] e.g.
the fractional part of the number 2.1 is graph of y = {x}
2.1–2 = 0.1 and the fractional part of
–3.7 is 0.3 The period of this function is
1 and graph of this function is as shown.
1
x {x}
0 x
[–2,–1) x+2 -2 1 2 3
-1
[–1,0) x+1
[0,1) x
[1,2) x–1
Illustration 13. If y = 2[x] + 3 & y = 3[x – 2] + 5 then find [x + y] where [ . ] denotes greatest integer function.
Solution y = 3[x – 2] + 5 = 3[x] – 1
so 3 [x] – 1 = 2 [x] + 3
[x] = 4 Þ 4 £ x < 5
then y = 11
so x + y will lie in the interval [15, 16)
so [x + y] = 15
é1 ù é1 1 ù é 1 2946 ù
Illustration 14. Find the value of ê ú + ê + ú + ...... ê + where [ . ] denotes greatest integer
ë 2 û ë 2 1000 û ë 2 1000 úû
function ?
JPR\COMP.251\Allen(IIT-JEE Wing)\2020–21\Enthusiast\Mathematics\Unit - 6
14
Fu ncti on
1
Illustration 15. Find the domain f(x) = where [.] denotes greatest integer function.
[|x|-5] - 11
x - [x] {x}
Solution y= =
1 + x - [x] 1 + {x}
1 1 1 1-y y
\ = +1 Þ = Þ {x} =
y {x} {x} y 1- y
y
0 £ {x} < 1 Þ 0 £ <1
1- y
Range = [0, 1/2)
Illustration 17. Solve the equation |2x – 1| = 3[x] + 2{x} where [.] denotes greatest integer and { .} denotes
fractional part function.
Solution We are given that, |2x – 1| = 3[x] + 2{x}
1
Let, 2x – 1 £ 0 i.e. x £ . The given equation yields.
2
1 – 2x = 3[x] + 2{x}
1 - 5[x]
Þ 1 – 2[x] – 2{x} = 3[x] + 2{x} Þ 1 – 5[x] = 4{x} Þ {x} =
4
1 - 5[x] 3 1
Þ 0£ < 1 Þ 0 £ 1 – 5[x] < 4 Þ – < [x] £
4 5 5
3 1
Now, [x] = 0 as zero is the only integer lying between – and
5 5
1 1 1
Þ {x} = Þx= which is less than ,
4 4 2
1
Hence is one solution.
4
JPR\COMP.251\Allen(IIT-JEE Wing)\2020–21\Enthusiast\Mathematics\Unit - 6
1
Now, let 2x – 1 > 0 i.e x >
2
Þ 2x – 1 = 3[x] + 2{x}
Þ 2[x] + 2{x} – 1 = 3[x] + 2{x}
1
Þ [x] = –1 Þ –1 £ x < 0 which is not a solution as x >
2
1
Þ x= is the only solution.
4
15
JEE-Mathematics
Illustration 18. The functions f(x) = log (x – 1) – log (x – 2) and g(x) = log æç x - 1 ö÷ are identical when x lies in the
è x - 2ø
interval
(A) [1, 2] (B) [2, ¥) (C) (2, ¥) (D) (–¥, ¥)
Solution. Since f(x) = log (x – 1) –log (x – 2).
Domain of f(x) is x > 2 or x Î (2, ¥) ...(i)
æ x - 1ö x -1
g(x) = log ç is defined if >0
è x - 2 ÷ø x-2
Þ x Î (–¥, 1) È (2, ¥) ...(ii)
From (i) and (ii), x Î (2, ¥). Ans. (C)
Column-I Column-II
(A) [x ] > 3
2
(p) x Î [2, 4)
(B) [x] – 5[x] + 6 = 0
2
(q) x Î (–¥, –2] È [2, ¥)
(C) x = {x} (r) x Î {0}
(D) {x} = [x] (s) x Î (–¥, –5)
(E) [x] < –5.2 (t) x Î {–2}
(F) 1 + x = sgn(x) (u) x Î [0, 1)
2. Are the following functions identical ?
x x2
(a) f(x) = & f(x) = (b) f(x) = x & f(x) = x2 (c) f(x)=log10x2 & f(x)= 2log10|x|
x2 x
3. Area of region enclosed by solution set of [x] . [y] = 2 is
1000
{x + r}
4. If [ x ] & { x } represents integral & fractional part of then the function defined by f (x) = [x] + å
r =1 1000
is
JPR\COMP.251\Allen(IIT-JEE Wing)\2020–21\Enthusiast\Mathematics\Unit - 6
equal to :
(A) 2[x] + {x} (B) 4x (C) x (D) 4[x] + 1000 {x}
16
Fu ncti on
13.0 ODD & EVEN FUNCTIONS
AL
If a function is such that whenever 'x' is in it's domain '–x' is also in it's domain & it satisfies
f(–x) = f(x) it is an even function
f(–x) = –f(x) it is an odd function
Note :
(i) A function may neither be odd nor even.
(ii) Inverse of an even function is not defined, as it is many – one function.
(iii) Every even function is symmetric about the y-axis & every odd function is symmetric about the origin.
(iv) Every function which has '–x' in it's domain whenever 'x' is in it's domain, can be expressed as the
sum of an even & an odd function .
f(x) + f(-x) f(x) - f(-x)
+
e.g. f(x) = 2 2
EVEN ODD
(v) The only function which is defined on the entire number line & even and odd at the same time is f(x)=0
ƒ(x) g(x) ƒ (x) + g(x) ƒ(x) – g(x) ƒ (x) . g(x) ƒ (x)/g(x) (goƒ) (x) (ƒog)(x)
odd odd odd odd even even odd odd
even even even even even even even even
odd even neither odd nor even neither odd nor even odd odd even even
even odd neither odd nor even neither odd nor even odd odd even even
Illustration 19. Which of the following functions is (are) even, odd or neither :
(i) f(x) = x2sinx
(ii) f(x) = 1 + x + x2 - 1 - x + x2
Solution (i) f(–x) = (–x)2 sin(–x) = –x2 sinx = –f(x).Hence f(x) is odd.
(ii) f(–x) = 1 + ( - x) + ( - x)2 - 1 - ( - x) + ( - x)2
Illustration 20. Which of the following functions is (are) even, odd or neither :
æ1 - xö e x + e- x
(i) f(x) = log çè ÷ (ii) f(x) = sinx – cosx (iii) f(x) =
1 + xø 2
æ 1 - ( - x) ö æ 1 + xö
Solution (i) f(–x) = log ç ÷ = log çè ÷ = –f(x). Hence f(x) is odd
è 1 + ( - x) ø 1 - xø
e - x + e - ( - x) e - x + e x
JPR\COMP.251\Allen(IIT-JEE Wing)\2020–21\Enthusiast\Mathematics\Unit - 6
x x
(i) f(x) = + +1
x
e -1 2
17
JEE-Mathematics
x x
Solution (i) f(x) = + +1
ex - 1 2
Clearly domain of f(x) is R ~ {0}. We have,
-x x -e x .x x (e x - 1 + 1)x x
f(–x) = -x
- +1 = x
- +1 = - +1
e -1 2 1-e 2 (e x - 1) 2
x x x x
= x+ - +1 = x + + 1 = f(x)
e -1 2 x
e -1 2
Hence f(x) is an even function.
(ii) f(x + y) = f(x) + f(y) for all x, y Î R
Replacing x, y by zero, we get f(0) = 2f(0)
Þ f(0) = 0
Replacing y by –x, we get f(x) + f(–x) = f(0) = 0
Þ f(x) = –f(–x)
Hence f(x) is an odd function.
(1 + 2x )
2
2 1/3 2 1/3
(a) f(x) = (b) f(x) = [(x + 1) ] + [(x – 1) ]
2x
1
6. The function f(x) = [x] + , x Ï I is (where [.] dentoes greatest integer function) :
2
JPR\COMP.251\Allen(IIT-JEE Wing)\2020–21\Enthusiast\Mathematics\Unit - 6
7. Let function f(x) satisfying the equation f(x + y) + f(x – y) = 2f(x)f(y) (where f(0) ¹ 0). Then prove that it is even
function.
ax - 1
8. If the graph of the function f(x) = is symmetric about y-axis. Then find n. (n Î I)
x n (a x + 1)
18
Fu ncti on
or
Note – If a continuous function has local maximum or local minimum, then f(x) is many-one because
atleast one line parallel to x-axis will intersect the graph of function atleast twice.
(c) Onto function (Surjective mapping)
If the function f : A ® B is such that each element in B (co-domain) is the ‘f’ image
of atleast one element in A, then we say that f is a function of A 'onto' B. Thus f : A ®
B is surjective if " b Î B, $ some a Î A such that f(a) = b
or
JPR\COMP.251\Allen(IIT-JEE Wing)\2020–21\Enthusiast\Mathematics\Unit - 6
or
19
JEE-Mathematics
Note – (i) If ‘f ’ is both injective & surjective, then it is called a Bijective mapping. The bijective
functions are also named as invertible, non singular or biuniform functions.
(ii) If a set A contains n distinct elements then the number of different functions defined
from A ® A is n n & out of it n! are one one and rest are many one.
(iii) f–R ® R is a polynomial
(a) Of even degree, then it will neither be injective nor surjective.
(b) Of odd degree, then it will always be surjective, no general comment can be given
on its injectivity.
Illustration 22. Let A=B={x : –1 £ x £ 1} be a mapping f : A ® B. For each of the following functions from A
to B, find whether it is injective, surjective or bijective.
(a) f(x) = |x| (b) f(x) = x|x| (c) f(x) = x3
px
(d) f(x) = [x] (e) f(x) = sin
2
Solution (a) f(x) = |x|
Y
Graphically ;
Which shows many one, as the straight line
is parallel to x-axis and cuts at two points.
Here range for f(x) Î [0, 1]
Which is clearly subset of co-domain i.e., –1 O +1 X
[0, 1] Í [–1,1]
Thus, into.
Hence, function is many-one-into
JPR\COMP.251\Allen(IIT-JEE Wing)\2020–21\Enthusiast\Mathematics\Unit - 6
20
Fu ncti on
f(x) Î [–1, 1]
Thus, range = co-domain
Hence, onto.
Therefore, f(x) is one-one onto or (Bijective).
(c) f(x) = x3, Y
1
Graphically;
Graph shows f(x) is one-one onto
–1 O 1 X
(i.e. Bijective)
[as explained in above example] –1
x 2 + 3x + a
Illustration 24. Let f(x) = , where f : R ® R. Find the value of parameter 'a' so that the given function
x2 + x + 1
is one-one.
x 2 + 3x + a
Solution f(x) =
x2 + x + 1
f'(x) = =
(x2 + x + 1)2 (x2 + x + 1)2
21
JEE-Mathematics
CLASSIFICATION OF FUNCTIONS
x-2
1. Let A = R – {3}, B = R – {1} and let f : A ® B defined by f(x) = . Check whether the function f(x) is
x-3
bijective or not.
2. The function f : R ® R, where R is the set of all real numbers defined by f(x) = 2x +3 is -
(A) f is both one-one and onto (B) f is one-one but not onto
(C) f is onto but not one-one (D) f is neither one-one nor onto
x2 + x + 1
4. Classify the following function f(x) defined in R ® R as injective, surjective, both or none f(x) =
x2 - x + 1
5. Classify the following function f(x) defined in R ® R as injective, surjective, both or none
3 2
f(x) = x + 6x + 11x + 6
2
6. The function f : [2, ¥) ® Y, defined by f(x) = x – 4x+5 is both one-one and onto if Y equals
7. Let A be a set of n distinct elements. Then find total number of onto function defined from A to A.
(i) Drawing the graph of y = f(x) + b, b Î R, from the known graph of y = f(x)
y=f(x)+b,b > 0
y=f(x)
A1
x0 y=f(x)+b,b < 0
O A
A2
It is obvious that domain of f(x) and f(x) + b are the same. Let us take any point x 0 in the domain of
f(x). y x = x0 = f(x 0 ) .
22
Fu ncti on
(ii) Drawing the graph of y = –f(x) from the known graph of y = f(x)
To draw y = –f(x), take the image of the curve y = f(x) in the x-axis as plane mirror.
y y=f(x) y
Þ
x x
0
0
y=–f(x)
(iii) Drawing the graph of y = f(–x) from the known graph of y = f(x)
To draw y = f(–x), take the image of the curve y = f(x) in the y-axis as plane mirror.
y y = f(x) y
Þ y = f(–x)
x x
0 0
(iv) Drawing the graph of y = |f(x)| from the known graph of y = f(x)
|f(x)| = f(x) if f(x) ³ 0 and |f(x)| = –f(x) if f(x) < 0. It means that the graph of f(x) and |f(x)| would
coincide if f(x) ³ 0 and for the portions where f(x) < 0 graph of |f(x)| would be image of y = f(x) in
x-axis.
y y
y=|f(x)|
y=f(x)
x Þ x
O O
(v) Drawing the graph of y = f(|x|) from the known graph of y = f(x)
ìf(x), x ³ 0
It is clear that, f(|x|) = í . Thus f(|x|) would be a even function, graph of f(|x|) and f(x)
îf(- x), x < 0
would be identical in the first and the fourth quadrants (ax x ³ 0) and as such the graph of f(|x|) would
be symmetric about the y-axis (as (|x|) is even).
y y
y=f(x) y=f(|x|)
Þ
O x O x
(vi) Drawing the graph of |y| = f(x) from the known graph of y = f(x)
JPR\COMP.251\Allen(IIT-JEE Wing)\2020–21\Enthusiast\Mathematics\Unit - 6
Clearly |y| ³ 0. If f(x) < 0, graph of |y| = f(x) would not exist. And if f(x) ³ 0, |y| = f(x) would give
y = ±f(x). Hence graph of |y| = f(x) would exist only in the regions where f(x) is non-negative and will
be reflected about the x-axis only in those regions.
y y
|y|=f(x)
Þ
O x O x
y=f(x)
23
JEE-Mathematics
(vii) Drawing the graph of y = f(x + a), a Î R from the known graph of y = f(x)
y=f(x)
y=f(x+a),a>0
y=f(x–a)
a<0
x 0–|a| x0 x0+|a|
(i) If a > 0, shift the graph of f(x) through 'a' units towards left of f(x).
(ii) If a < 0, shift the graph of f(x) through 'a' units towards right of f(x).
(viii) Drawing the graph of y = af(x) from the known graph of y = f(x)
y=af(x), a >1
y=f(x)
y=af(x),
0<a<1
It is clear that the corresponding points (points with same x co-ordinates) would have their ordinates in the ratio
of 1 : a.
(ix) Drawing the graph of y = f(ax) from the known graph of y = f(x).
y y=f(x)
y=f(ax), a >1
y=f(ax),0<a<1
O
x
x0
Let us take any point x0 Î domain of f(x). Let ax = x0 or x = .
a
1
Clearly if 0 < a < 1, then x > x0 and ƒ (x) will stretch by units along the y-axis and if a > 1, x < x 0, then
a
f(x) will compress by 'a' units along the y-axis.
JPR\COMP.251\Allen(IIT-JEE Wing)\2020–21\Enthusiast\Mathematics\Unit - 6
0,1
ì1 - x ; x < -1
ï
f(x) = í2 ; -1 £ x £ 1 (–1,0) (1,0)
x
ï1 + x ; x > 1 (0,0)
î
24
Fu ncti on
1 y=|x–1|
x–
y= (0,1) y=–|x–1|
(1,0) (1,0)
Solution Þ Þ x
O O O 1
1
(–1,0) (0,–1)
2 2 y=|2–|x–1|
y=2–|x–1|
Þ
O 1 x (–1,0) O 1 (3,0)
4
Illustration 27. Draw the graph of y = 2 -
|x - 1|
y
y
1 1
y= y=
x x–1
x=1
x O
Solution O Þ x
(0,–1)
y y
1 1 x=1
y= y=–
|x–1| |x–1|
O x
x
Þ O Þ
(0,–1)
y
y y=2– 4
x=1 y=2 |x–1|
–4
y=
|x–1|
O x
Þ Þ (3,0)
x
(–1,0)
(0,–4)
(0,–2)
y=ex
1 y=e|x|
JPR\COMP.251\Allen(IIT-JEE Wing)\2020–21\Enthusiast\Mathematics\Unit - 6
Solution O Þ
y y |x|
y=|e –2|
y=e|x|–2 (0,1)
O x
Þ x
(0,–1) O
25
JEE-Mathematics
Illustration 29. Draw the graph of f(x) = cosx cos(x + 2) – cos2(x + 1).
Solution f(x) = cosx cos(x + 2) – cos2(x + 1)
0
1 1
= ëécos(2x + 2) + cos 2 ûù – ëécos(2x + 2) + 1ûù
2 2 ½ cos2 – ½
1 1
= cos 2 - < 0 .
2 2
-x 1
(a) Plot y = e -
2
Draw the graph of following functions where [.] denotes greatest integer function and { .} denotes fractional
part function.
(b) y = {sin x }
(c) y = [x] + { x}
2
7. Draw the graph of the function f(x)= x - 4|x | + 3 and also find the set of values of ‘a’ for which the equation
JPR\COMP.251\Allen(IIT-JEE Wing)\2020–21\Enthusiast\Mathematics\Unit - 6
26
Fu ncti on
16.0 COMPOSITE OF UNIFORMLY & NON-UNIFORMLY DEFINED FUNCTION
SL AL
Let f – A ® B & g : B ® C be two functions. Then the function gof : A ® C defined by (gof ) (x) = g(f(x)) " x Î
A is called the composite of the two functions f & g.
x f(x)
Diagramatically f g g (f(x))
Thus the image of every x Î A under the function gof is the g-image of f-image of x.
Note that gof is defined only if " x Î A, f(x) is an element of the domain of ‘g’ so that we can take its
g-image. Hence in gof(x) the range of ‘f’ must be a subset of the domain of ‘g’.
Illustration 30. If f be the greatest integer function and g be the modulus function, then
æ 5ö æ 5ö
(gof) ç - ÷ – (fog) ç - ÷ =
è 3ø è 3ø
æ -5 ö æ -5 ö ì æ -5 ö ü ì æ -5 ö ü
Solution Given (gof) ç ÷ – (fog) ç ÷ = g íf çè ÷ø ý - f íg çè ÷ø ý
è 3ø è 3ø î 3 þ î 3 þ
æ 5ö
= g(–2) – f çè ÷ø = 2 – 1 = 1 Ans.(A)
3
Illustration 31. Find the domain and range of h(x) = g(f(x)), where
ì [x], -2 £ x £ -1
f(x) = í and
î x + 1, -1 < x £ 2
ì [x], -p £ x £ 0
g(x) = í , [.] denotes the greatest integer function.
îsin x, 0 £ x £ p
ì [f(x)], -p £ f(x) < 0
Solution h(x) = g(f(x)) = í
îsin(f(x)), 0 £ f(x) £ p f(x)=p
+3
From graph of f(x), we get
JPR\COMP.251\Allen(IIT-JEE Wing)\2020–21\Enthusiast\Mathematics\Unit - 6
ì [[x]], -2 £ x £ -1 –2 –1 +1
h(x) = í
î sin( x + 1), -1 < x £ 2 –1 2
–2
Þ Domain of h(x) is [–2, 2]
f(x)=–p
and Range of h(x) is {–2, 1} È [sin3, 1]
27
JEE-Mathematics
ì x + 1, x £1 ì x 2 , -1 £ x < 2
Illustration 32. Let f(x) = í and g(x) = í , find (fog)
î2x + 1, 1 < x £ 2 î x + 2, 2 £ x £ 3
y
ì g(x) + 1, g(x) £ 1
Solution f(g(x)) = í 5
î2g(x) + 1, 1 < g(x) £ 2 2
4 x+
Here, g(x) becomes the variable that means
we should draw the graph. 3 x2
It is clear that g(x) £ 1 ; " x Î [–1, 1] 2
and 1 < g(x) £ 2 ; " x Î (1, 2] 1
x
–2 –1 0 1 2 3
ïì x + 1, -1 £ x £ 1
2
Þ f(g(x)) = í
ïî2x2 + 1, 1 < x £ 2
A function is said to be homogeneous with respect to any set of variables when each of its terms is of
the same degree with respect to those variables.
For examples 5x2 + 3y2 – xy is homogenous in x & y. Symbolically if, f(tx, ty) = tn f(x, y) then f(x, y)
is homogeneous function of degree n.
Illustration 34. If x, y Î [0, 10], then the number of solution (x, y) of the inequality 3sec
2
x -1
9y 2 - 6y + 2 £ 1
are
Solution If x, y Î [0, 10], then the.......
We have, 3 sec
2
x -1
9y 2 - 6y + 2 £ 1
2
2 æ 1ö 1
Þ 3sec x
çy - ÷ + £ 1
è 3ø 9
JPR\COMP.251\Allen(IIT-JEE Wing)\2020–21\Enthusiast\Mathematics\Unit - 6
2
æ 1ö 1 1
but 3sec 2 x
³ 3 and çy - ÷ + ³
è 3ø 9 3
1
So, using boundness we must have sec2 x = 1 and y - =0
3
1
Þ x = 0, p 2p, 3p and y =
3
Þ There are 4 solution.
28
Fu ncti on
æ æ p öö æ æ p öö
(d) f ç g ç ÷ ÷ (e) g(f(1)) (f) g ç g ç ÷ ÷
è è 4 øø è è 2 øø
ì x + 1; 0 £ x < 2
2. (a) If f(x) = í , then find fof(x).
î|x|; 2 £ x < 3
ì1 + x, 0£ x£2
(b) Let f(x) = í , then find (fof)(x).
î3 - x, 2<x£3
x2
3. Find the boundness of the function f(x) =
x4 + 1
6. Let f(x) be a function whose domain is [–5, 7]. Let g(x) = |2x + 5|. Then find the domain of (fog) (x) :
A function defined by an equation not solved for the dependent variable is called an implicit function.
e.g. the equations x3 + y3 = 1 & xy = yx, defines y as an implicit function. If y has been expressed in
terms of x alone then it is called an Explicit function.
x2 + e x + 5
(A) y = -1
(B) y = x2
1 - cos x
x 2 log x
JPR\COMP.251\Allen(IIT-JEE Wing)\2020–21\Enthusiast\Mathematics\Unit - 6
Let f : A ® B be a one-one & onto function, then their exists a unique function g : B ® A such that
f(x) = y Û g(y) = x, " x Î A & y Î B. Then g is said to be inverse of f.
Thus g = f–1 : B ® A = {(f(x), x))|(x, f (x)) Î f}.
29
JEE-Mathematics
l Properties of inverse function
(a) The inverse of a bijection is unique.
(b) If f : A ® B is a bijection & g : B ® A is the inverse of f, then fog = I B and gof = IA, where IA & IB are
identity functions on the sets A & B respectively. If fof = I, then f is inverse of itself.
(c) The inverse of a bijection is also a bijection.
(d) If f & g are two bijections f : A ® B, g : B ® C then the inverse of gof exists and (gof)–1 = f–1 o g–1.
(e) Since f(a) = b if and only if f–1(b) = a, the point (a, b) is on the graph of ‘f’ if and only if the
point (b, a) is on the graph of f–1. But we get the point (b, a) from (a, b) by reflecting about the line
y = x.
y y y
(b, a)
y = f(x) y=x
–1
f
(a, b)
0 x
0 x 0 x (–1,0)
(0,–1)
y=x y=x f –1
y = f (x)
The graph of f -1 is obtained by reflecting the graph of f about the line y =x.
é p pù
= sinx is invertible if f – ê - , ú ® [-1, 1]
ë 2 2û
é p pù
Þ the inverse mapping would be f –1 – [-1, 1] ® ê - , ú .
ë 2 2û
y
(1,p/2)
p/2 y=x
y=sinx
1 (p/2,1)
JPR\COMP.251\Allen(IIT-JEE Wing)\2020–21\Enthusiast\Mathematics\Unit - 6
–p/2 –1 x
O 1 p/2
(0,–1)
(–p/2,1)
(-1-p/2)
30
Fu ncti on
Illustration 36. Let f : R ® R be defined by f(x) = (ex–e–x)/2. Is f(x) invertible ? If so, find its inverse.
Solution Let us check for invertibility of f(x) :
(a) One-One
Let x1, x2 Î R and x1< x2
Þ e x1 < e x2 (Because base e > 1) ... (i)
Also x1 < x2 Þ –x2 < –x1
Þ e- x2 < e - x1 (Because base e > 1) ... (ii)
(i) + (ii) Þ e x1 + e - x2 < e x2 + e- x1
1 x1
Þ ( e - e- x1 ) < 1 ( e x2 - e - x2 ) Þ f(x1) < f(x2) i.e. f is one-one.
2 2
(b) Onto
As x tends to larger and larger values so does f(x) and
when x ® ¥, f(x) ® ¥.
Similarly as x ® -¥, f(x) ® -¥ i.e. - ¥ < f(x) < ¥ so long as x Î (-¥, ¥)
Hence the range of f is same as the set R. Therefore f(x) is onto.
Since f(x) is both one-one and onto, f(x) is invertible.
-1 -1
ef ( x)
- e- f ( x)
-1 -1
= x Þ e2f (x)
- 2xe f (x)
-1 = 0
2
-1 2x ± 4x2 + 4 -1
Þ ef ( x)
= Þ ef ( x ) = x ± 1 + x2
2
-1
Since e f (x)
> 0, hence negative sign is ruled out and
-1
Hence e f (x)
= x + 1 + x2
ì x; < 1
ï 2
Illustration 37. Find the inverse of the function f(x) = í x ;1 £ x £ 4
ï
î 8 x; x > 4
ì x; < 1
ï 2
Solution Given f(x) = í x ;1 £ x £ 4
ï
î 8 x; x > 4
JPR\COMP.251\Allen(IIT-JEE Wing)\2020–21\Enthusiast\Mathematics\Unit - 6
ì
ï y, y <1
ïï
\ x = í y, 1£ y £4
ï 2
ïy , y2
>4
ïî 64 64
31
JEE-Mathematics
ì
ï y, y <1
ïï
-1
Þ f (y) = í y, 1 £ y £ 16
ï 2
ïy , y > 16
ïî 64
ì x; < 1
ï
ïï
Hence f (x) = í
–1 x;1 £ x £ 16
ï
ï x2
; x > 16
ïî 64
a x - a -x
4. f(x) =
a x + a -x
y = [1 - ( x - 3 ) ]1 / 7
4
5.
é 2p p ù é p 5p ù
(A) A = ê- , ú , B = [2, 6] (B) A = ê , ú , B = [–2, 2]
ë 3 3û ë6 6 û
é p pù é p pù
(C) A = ê - , ú , B = [2, 6] (D) A = ê - , ú , B = [2, 6]
ë 2 2û ë 3 3û
ex - e- x
(c) y =
e x + e- x
8. Write explicitly, functions of y defined by the following equations and also find the domains of definition of
the given implicit functions :
x y
(a) 10 + 10 = 10 (b) x + |y| = 2y
32
Fu ncti on
21.0 PERIODIC FUNCTION
AL
A function f(x) is called periodic if there exists a least positive number T(T >0) called the period of the function
such that f(x + T) = f(x), for all values of x within the domain of f(x).
e.g. The function sinx & cosx both are periodic over 2p & tan x is periodic over p.
Note – For periodic function
(i) f(T) = f(0) = f(–T), where 'T' is the period.
(ii) Inverse of a periodic function does not exist.
(iii) Every constant function is periodic, but its period is not defined.
(iv) If f(x) has a period T & g(x) also has a period T then it does not mean that f(x) + g(x)
must have a period T. e.g. f(x) = |sin x| + |cos x|.
1
(v) If f(x) has period p, then and f(x) also has a period p.
f(x)
(vi) If f(x) has period T then f(ax + b) has a period T/|a| (a ¹ 0).
22.0 GENERAL
AL
Illustration 38. Find the periods (if periodic) of the following functions, where [.] denotes the greatest integer
function
(i) f(x) = eln(sinx) + tan3x – cosec(3x – 5)
(ii) f(x) = x – [x – b], b Î R
sin x + cos x
(iii) f(x) =
sin x + cos x
p
(iv) f(x) = tan [x]
2
(v) f(x) = cos(sinx) + cos(cosx)
2p
cosec (3x – 5) =
3
\ Period = 2p
(ii) f(x) = x – [x – b] = b + {x – b}
\ Period = 1
33
JEE-Mathematics
sin x + cos x
(iii) f(x) =
sin x + cos x
p
Since period of |sinx + cosx| = p and period of |sinx| + |cosx| is .
2
Hence f(x) is periodic with p as its period
p
(iv) f(x) = tan [x]
2
p p p p
tan [x + T]= tan [x] Þ [x + T] = np + [x]
2 2 2 2
\ T=2
\ Period = 2
1
Period of |cos2px| =
2
......................................
1
Period of |cosnpx| =
n
So period of f(x) will be L.C.M. of all period = 1
JPR\COMP.251\Allen(IIT-JEE Wing)\2020–21\Enthusiast\Mathematics\Unit - 6
Illustration 39. Find the periods (if periodic) of the following functions, where [.] denotes the greatest integer function
px px
(ii) f(x) = sin + cos
2 3
px px
(iii) f(x) = sin + cos
3 2 3
34
Fu ncti on
px 2p
Period of cos = =2 3
3 p/ 3
Q L.C.M. of two different kinds of irrational number does not exist.
\ not periodic.
px 2p
(iii) Period of sin = =2 3
3 p/ 3
px 2p
Period of cos = =4 3
2 3 p/2 3
Q L.C.M. of two similar irrational number exist.
2. If f : R ® R is a function satisfying the property f(2x + 3) + f(2x + 7) = 2 " x Î R, then find the period of f(x)
35
JEE-Mathematics
æ1ö 2
Illustration 40. If 2f(x) – 3f ç ÷ = x , x is not equal to zero, then f(2) is equal to-
èxø
æ1ö
Solution 2f ( x ) - 3f ç ÷ = x 2 ... (1)
èxø
1
Put x =
x
æ1ö 1
2.f ç ÷ - 3f ( x ) = 2 ... (2)
x
è ø x
(1) × 2 and (2) × 3
æ1ö
4f ( x ) - 6f ç ÷ = 2x 2
èxø
æ1ö 3
6f ç ÷ - 9f ( x ) = 2
èxø x
3
-5f ( x ) = 2x 2 +
x2
æ 2x 4 + 3 ö æ 2 ´ 16 + 3 ö æ 35 ö 7
f (x) = - ç f ( 2) = - ç
2 ÷; ç 5 ( 4 ) ÷÷ = - ç 20 ÷ = - 4
è 5x ø è ø è ø
2002 ö
Illustration 41. Let f be a real valued function such that f (x) + 2f æç ÷ = 3x for all x > 0. Find f (2).
è x ø
Solution
æ 2002 ö = 3x
f (x) + 2f ç ....(1)
÷
è x ø
2002
Replace x by
x
æ 2002 ö 6006
fç ÷ + 2f(x) = ....(2)
è x ø x
2 × Eq.(2) – Eq.(1)
12012
3f(x) = - 3x
x
put x = 2
3f(2) = 6006 – 6 = 6000
f(2) = 2000
æ 1ö 1 æ 1ö
Illustration 42. If f ç x + ÷ = x 3 + 3 - 4 ç x 2 + 2 ÷ + 13 , then find the value of f(4).
è xø x è x ø
æ 1ö 1 æ 1 ö
Solution f ç x + ÷ = x3 + 3 - 4 ç x2 + 2 ÷ + 13
JPR\COMP.251\Allen(IIT-JEE Wing)\2020–21\Enthusiast\Mathematics\Unit - 6
è xø x è x ø
æ 1ö
3
æ 1ö éæ 1ö
2
ù
= ç x + ÷ - 3 ç x + ÷ - 4 êç x + ÷ - 2ú + 13
è xø è xø êëè xø úû
3 2
æ 1ö æ 1ö æ 1ö
= ç x + ÷ - 4 ç x + ÷ - 3 ç x + ÷ + 21
è xø è xø è xø
\ ( )
f 2 + 3 = 43 – 4 × 42 – 3 × 4 + 21 = 9
36
Fu ncti on
FUNCTIONAL EQUATIONS
10
1. If f(x + y) = f(x).f(y) " x, y ÎN and f(1) = 2 find å f(n)
n= 1
æ 1ö 1
2. If y=f(x) satisfies the condition f ç x + ÷ = x 2 + 2 (x ¹ 0) find f(x)
è xø x
1 1
4. If f ( x ) f æç ö÷ = f ( x ) + f æç ö÷ " x Î R – {0}, where f(x) be a polynomial function and f(5) = 126, find f(3)=
èxø èxø
l To check whether relation is a function, vertical line test can be applied on its graph.
l f and g one are two functions defined for same domain, range of f is R and g is a bounded in domain. Then range
of f(x) + g(x) is R.
l If domain consist of discrete number of elements range can be found by direct substituting the values of x.
l f & g are two one-one function defined for all real values then both fog & gof are one-one.
37
JEE-Mathematics
l
Case-I when x = AP £ OA, i.e., x £ O
2 F l
P
1 x
ar(DAEF) = x.2x = x 2 (Q PE = PF = AP = x)
2 A B
E
l
Case-II when x = AP > OA, i.e., x > but x £ 2l
2 D
F
C
2–lx
ar(ABEFDA) = ar(ABCD) – ar(DCFE)
P
= l -
1
2
2
( ) (
2l - x .2 2l - x ) [Q CP = 2l - x ]
O E
( )
= l2 - 2l2 + x 2 - 2 2lx = 2 2lx - x 2 - l 2 A B
\ the required function s(x) is as follows :
ì 2 l
ï x , 0£x£
ï 2
s(x) = í ;
ï2 2 lx – x 2 – l2 , l
< x £ 2l
ïî 2
ì 1 1
ï 2 at x =
area of s(x) = í 2 Ans.
ï
î8( 2 – 1) at x = 2
Illustration 2. If the function f(x) satisfies the functional rule, f(x + y) = f(x) + f(y) " x,y Î R & f(1) = 5, then
m
find å f(n)
n =1
and also prove that f(x) is odd function.
Solution Here, f(x + y) = f(x) + f(y); put x = t – 1, y = 1
f(t) = f(t – 1) + f(1) ....(1)
\ f(t) = f(t – 1) + 5
Þ f(t) = {f(t – 2) + 5} + 5
Þ f(t) = f(t – 2) + 2(5)
Þ f(t) = f(t – 3) + 3(5)
.............................
.............................
Þ f(t) = f{t – (t –1)} + (t – 1)5
JPR\COMP.251\Allen(IIT-JEE Wing)\2020–21\Enthusiast\Mathematics\Unit - 6
38
Fu ncti on
Now putting x=0, y=0 in the given function, we have
f(0 + 0) = f(0) + f(0)
\ f(0) = 0
Also putting (–x) for (y) in the given function.
f(x – x) = f(x) + f(– x)
Þ f(0) = f(x) + f(–x)
Þ 0 = f(x) + f(–x)
Þ f(–x) = –f(x) ....(ii)
m
5m(m + 1)
Thus, å f(n) =
n =1 2
and f(x) is odd function.
x x
Illustration 3. Range of f(x) = 4 + 2 + 1 is
x x
Solution f(x) = 4 + 2 + 1
x
Let 2 = t > 0, " x Î R
2
\ f(x) = g(t) = t + t + 1 t>0
2
æ 1ö 3
g(t) = ç t + ÷ +
è 2ø 4
2 2
æ 1ö 1 æ 1ö 1 æ 1ö 3
çt + ÷ > Þ çt + ÷ > Þ çt + ÷ + >1
è 2ø 2 è 2ø 4 è 2ø 4
Range is (1, ¥)
x–1
Illustration 4. Let f : N ® N, where f(x) = x + ( – 1) , then the inverse of f is
x–1
Solution f(x) = x + (–1)
é3 1 ù é3 2 ù é3 3 ù é 3 24 ù
Solution êë 4 + 100 úû + êë 4 + 100 úû + êë 4 + 100 úû + ....... êë 4 + 100 úû
é 3 25 ù é 3 26 ù é 3 27 ù é 3 99 ù
+ê + ú +ê + ú +ê + ú + ....... ê +
ë 4 100 û ë 4 100 û ë 4 100 û ë 4 100 úû
= 0 + 75 = 75
39
JEE-Mathematics
1
Illustration 7. Fundamental period of the function f(x) = + cos (cos6x) is
-
sin 4x cos 4x
p
Solution As |sin 4x| – |cos 4x| has period
4
æ pö æp ö æp ö
we get g ç x + 8 ÷ = sin ç 2 + 4x ÷ - cos ç 2 + 4x ÷
è ø è ø è ø
p
\ Fundamental period of g(x) is .
8
Now h(x) = cos (cos 6x)
æ pö
then h ç x + ÷ = cos(cos(p + 6x))
è 6ø
p
= cos (–cos 6x) = cos (cos 6x) \ Period is
6
p p p
Taking L.C.M. of , we get
6 8 2
Illustration 9. Let f (x) and g (x) be bijective functions where f: {7, 8, 9, 10} ® {1, 2, 3, 4} and
g:{3, 4, 5, 6} ® {13, 14, 15, 16 } respectively then find the number of elements in domain and
range of gof(x) ?
Solution domain of gof is {x Î {7, 8, 9, 10} : f(x) Î {3,4}}
\ there are 2 elements in the domain of gof
Since gof is one-one, therefore, there are 2 elements in the range of gof.
0 1 2 3 4
Þ go f (x) = 1+ x 0 £x<1
= 3–x 1£x<2 (2, –11)
= x–1 2£x<3 (1, 2)
= 5–x 3£x£4
fog(x) = –1 – x, –1 £ x < 0
= –1 + x, 0£x<1
0 1 2 3 4
40
Fu ncti on
2x (sin x + tan x)
Illustration 11. Prove that f(x) = is an odd function, where [ ] denotes greatest integer function.
é x + 2p ù
2ê - 3
ë p úû
éx + yù
Solution Let g(x) = 2 ê –3
ë p úû
éxù
=2 ê ú +1
ë pû
é xù
Þ g(x) = 2 ê – ú + 1
ë pû
æ éxùö
= 2 ç –1 – ê ú ÷ + 1
è ëpû ø
æ éx ù ö
= – ç 2 ê ú + 1÷
è ëpû ø
Þ f is an odd function
Illustration 12. Let ‘f’ be a real valued function defined for all real numbers x such that for some positive constant
1
‘a’ the equation f(x + a)= + f(x) - (f(x))2 holds for all x. Prove that the function f is periodic.
2
2 2
æ 1ö 1 æ 1ö
Solution ç f(x + a) – ÷ = – ç f(x) – ÷
è 2ø 4 è 2ø
2 2
æ 1ö æ 1ö
Þ ç f(x + 2a) – ÷ = ç f(x) – ÷
è 2ø è 2ø
Þ f is periodic fucntion
JPR\COMP.251\Allen(IIT-JEE Wing)\2020–21\Enthusiast\Mathematics\Unit - 6
41
JEE-Mathematics
ANSWER KEY
BEGINNER'S BOX-1
1. (A) 2. (C) 3. (A) 4. (B) 5. (D) 6.(D) 7.(D) 8. (C)
BEGINNER'S BOX-2
1. (a) Not a function (b) function (c) not a function (d) not a function
2 on sweeping a vertical line parallely if it cuts the graphs of relation on atmost one point, relation is a
function.
3. (a) x Î (0, ¥) (b) x Î (– ¥, 5/2] (c) x Î (– ¥, 0) È (4, ¥) (d) x Î (– ¥, 0)
4. (4, ¥)
é1 1ù
5. (a) [–1, 1] (b) ê , ú (c) (0, ¥) (d) [–1, 1]
ë4 2û
é 1ù é 1 1ù é 11 ö
6. (a) [–2, 2] (b) ê0, ú (c) ê - , ú (d) êln , ¥ ÷
ë 2û ë 2 2û ë 3 ø
æ 1 ö æ 3ö
7. (a) [–2, 0) È (0, 1) (b) ç , 1÷ È ç 1, ÷
è 2 ø è 2ø
8. (a) {–1, 1} (b) [0, 10] (c) (1, ¥) (d) [0, 2] (e) [–11, 16]
BEGINNER'S BOX-3
1. (A) ® (q), (B) ® (p), (C) ® (u), (D) ® (r), (E) ® (s), (F) ® (t)
2. (a) No (b) No (c) Yes
3. 4 4. (C) 5. domain = R, Range = R
é3- 5 3+ 5 ù
6. Domain = R, Range = ê , ú
ë 2 2 û
BEGINNER'S BOX-4
1. (a) even (b) odd (c) odd (d) neither even nor odd
2. (a) odd function (b) odd function (c) even function
3. (a) odd function (b) even function
4. (a) neither even not odd (b) even function
5. (a) even function (b) even function
6. (B) 8. Odd Integer
JPR\COMP.251\Allen(IIT-JEE Wing)\2020–21\Enthusiast\Mathematics\Unit - 6
BEGINNER'S BOX-5
1. yes
2. (A)
3. (a) 65 (b) 6C5 × 5! (c) 65 – 6! (d) 6 (e) 0 (f) 65
4. function is neither injective nor surjective
5. function is surjective
6. [1, ¥]
7. (n!)
42
Fu ncti on
BEGINNER'S BOX-6
1. 3
æ pö
y = sin ç x + ÷
è 4ø
2
y = |x – 1|+ 2
y=sin2x
1
(c)
0 p
p 3p 2p
2 2
–1
3
y=ln(–x)
y=|ln|x|| 2
2
3. (a) (b) (c) 1 2 (d)
–1 1
5. |x| + |y| = 1 –1 1
–1
1/2
6. (a) x (b)
(c)
JPR\COMP.251\Allen(IIT-JEE Wing)\2020–21\Enthusiast\Mathematics\Unit - 6
7. a Î (1, 3) È {0}
8. 1 9. 3 10. 4
43
JEE-Mathematics
BEGINNER'S BOX-7
1. (a) 0 (b) 0 (c) 0 (d) 0 (e) 0 (f) 0
ì x + 2, 0 £ x < 1 ì2 + x, 0£ x £1
ï ï
2. (a) í x + 1, 1 £ x < 2 (b) (fof)(x) = í2 - x, 1< x£2
ï x, 2£x<3 ï4 - x,
î î 2<x£3
é 1ù
3. ê0, 2 ú
ë û
6. [–6, 1]
BEGINNER'S BOX-8
1. (A)
1 1
2. (a) y = (b) y =
x x2
ìï- - x, -1 £ x £ 0
3. f -1 (x) = í
ïî x, 0£ x £1
1 æ1+ x ö
4. log a ç ÷
2 è1- x ø
5. 3 + (1 – x7)1/4
6. (A)
ex – e– x log x2 1 æ1+ x ö
7. (a) (b) (c) ln ç ÷
2 log x2 – 1 2 è1– x ø
8. (a) (–¥, 1) (b) R
BEGINNER'S BOX-9
p
1. (a) 2p (b) 2p (c) 1 (d) p (e)
2
2. 8
p p
3. (a) f ( +x) = f(x) Þ Period = (b) p
2 2
p 10p
4. (a) (b) LCM of , 7p, 70p
2 3
5. (ABD)
JPR\COMP.251\Allen(IIT-JEE Wing)\2020–21\Enthusiast\Mathematics\Unit - 6
6. (a) 2 (b) p
BEGINNER'S BOX-10
1. 2046
2. x2 – 2, x Î (–¥,–2] È [2, ¥)
3. –x2 +1
4. 28
44
Fu ncti on
1
3. The range of the function f(x) = , is :
4 + 3cos x
é 1 ù é 1 ö æ 1 ù
(A) ê ú (B) ê ÷ (C) ç ú (D) None of these
ë 7, 1 û ë 7, 1ø è 7, 1 û
1
4. The domain of the function f(x) = + x + 2 , is :
log10 (1 - x)
(A) [– 2, 0) È (0, 1) (B) (–2, 0) È (0, 1] (C) (– 2, 0) È (0, 2] (D) (– 2, 0) È [0, 1]
x –x
6. The range of the function f (x) = e - e , is :
(A) [0, ¥) (B) (– ¥, 0) (C) (– ¥, ¥) (D) None of these
7. Let [x] denote the greatest integer function of x . Then in the interval [0, 3] the number of solutions of the
equation, x2 - 3x + [x] = 0 is :
(A) 6 (B) 4 (C) 2 (D) 0
log2 (x + 3)
8. The domain of definition of f (x) = is :
x2 + 3 x + 2
R R (-3, ¥)
(A) (B) (- 2, ¥) (C) (D)
{ -1, -2} { -1, -2, -3} { -1, -2}
9. If ¦(x) = {x} + {x + 1} + {x + 2}........{x + 99}, then the value of [¦ ( 2 ) ] is, (where {.} denotes fractional
part function & [.] denotes the greatest integer function) :
(A) 5050 (B) 4950 (C) 41 (D) 14
JPR\COMP.251\Allen(IIT-JEE Wing)\2020–21\Enthusiast\Mathematics\Unit -6
3 6
11. Function f(x) = loge(x + 1 + x ) is :
(A) even (B) odd (C) neither even nor odd (D) None of these
45
JEE-Mathematics
3
12. If f : R ® R, f(x) = x + x, then f is:
(A) one-one onto (B) one-one into (C) many-one onto (D) many-one into
+
13. If S be the set of all triangles and f : S ® R , f(D) = Area of D, then f is :
(A) one-one onto (B) one-one into (C) many-one onto (D) many-one into
p p
14. The function defined as f : é - , ù ® [–1, 1], f (x) = sin x is :
ê 2 2ú
ë û
(A) one-one onto (B) many-one onto (C) one-one into (D) many-one into
x 2 - 3x + 4
15. Let f: R ® R be a function defined by f(x) = then f is :
x2 + 3x + 4
(A) one – one but not onto (B) onto but not one – one
(C) onto as well as one – one (D) neither onto nor one – one
16. The values of the parameter a, for which the function f(x) = 1 + ax, a ¹ 0 is the inverse of itself, is :
(A) 1 (B) 2 (C) –1 (D) 0
2 2
17. If f : R ® R, f(x) = (x + 1) and g : R ® R, g(x) = x + 1 then (fog)(–3) is equal to :
(A) 121 (B) 144 (C) 112 (D) 11
18. If f(x) = [x] and g(x) = cos(px), then the range of gof is :
(A) {0} (B) {–1, 1} (C) {–1, 0, 1} (D) [–1, 1]
4 4
20. The fundamental period of sin x + cos x is :
(A) p/2 (B) p (C) 2p (D) None of these
æxö æxö
21. The fundamental period of the function f(x) = sin ç ÷ + cos ç ÷ is :
è3ø è2ø
(A) 2p (B) 3p (C) 6p (D) 12p
22. Let f (x) be a function whose domain is [–5, 7]. Let g(x) = | 2x + 5 |, then the domain of fog(x) is :
(A) [–5, 1] (B) [–4, 0] (C) [–6, 1] (D) None of these
23. Let f(x) = sin [a] x (where [ ] denotes the greatest integer function). If f is periodic with fundamental period
p, then a belongs to :
(A) [2, 3) (B) {4, 5} (C) [4, 5] (D) [4, 5)
JPR\COMP.251\Allen(IIT-JEE Wing)\2020–21\Enthusiast\Mathematics\Unit -6
24. If f : R ® R is a function satisfying the property f(x+1) + f(x+3) = K (constant) " x Î R then the period of
f (x) is :
(A) 4 (B) K (C) 1 (D) p
5 1/7
25. The inverse of the function y = [1 – (x – 3) ] is :
7 1/5 7 1/5 7 1/5
(A) 3 + (1 – x ) (B) 3 – (1 – x ) (C) 3 – (1 + x ) (D) None of these
46
Fu ncti on
x(x – 1) –1
26. If the function f : [1, ¥) ® [1, ¥) is defined by f(x) = 2 , then f (x) is -
x(x -1)
æ 1ö 1
(A) ç ÷
è 2ø
(B)
2
(
1 + 1 + 4 log 2 x )
(C)
1
2
(
1 - 1 + 4 log2 x ) (D) Not defined
4x æ 1 ö æ 2 ö æ 1996 ö
27. Let f(x) = . Then value of fç + fç + × ×× + f ç is :
4x + 2 è 1997 ÷ø è 1997 ÷ø è 1997 ÷ø
(A) 898 (B) 998 (C) 991 (D) None of these
28. Let n(A) = 4 & n(B) = 6 then the number of one-one functions from A to B is :
(A) 120 (B) 360 (C) 24 (D) none of these
29. Set A has 3 elements and set B has 4 elements. The number of injective mappings that can be defined from
A to B is :
(A) 144 (B) 12 (C) 24 (D) 64
30. The graph of the function y = f(x) is as shown in the figure. Then which of the following could represent the
graph of the function y = f(x) ?
(–2, 1) (2, 1)
(–1, 0) (1, 0)
(0, –1)
y
y
1 1
(A) (B)
–2 0 1 2 x –2 0 1 2
y
y
1
1
(C) (D)
x
–2 –1 0 1 2 –2 0 1 2 x
JPR\COMP.251\Allen(IIT-JEE Wing)\2020–21\Enthusiast\Mathematics\Unit -6
47
JEE-Mathematics
2. Range of the function f(x) = cos (K sinx) is [–1, 1], then k can take values :
(A) 1 (B) 2 (C) 5 (D) 4
æ1ö æ æ 1öö px p
3. Given the function f(x) such that 2f(x) + xf ç ÷ - 2f ç 2 sin p ç x + ÷ ÷ = 4 cos 2 + x cos , then which
èxø è è 4øø 2 x
one of the following is correct ?
æ 1ö
(A) f ( 2) + f ç ÷ = 1
è 2ø
æ 1ö
(B) f(1) = –1, but the values of f(2), f ç ÷ cannot be determined
è 2ø
æ 1ö
(C) f(2) + f(1) = f ç ÷
è 2ø
(D) f(2) + f(1) = 0
æ 1 + x2 ö æ 1 + x3 ö (1 + 2x )2
2 2
(A) log ç ÷ (B) sin x + cos x (C) log ç ÷ (D)
è 1 - x2 ø è 1 - x3 ø 2x
e|x| - e - x
6. Let f : R ® R be a function defined by f(x) = then :
ex + e- x
(A) injective (B) not injective (C) surjective (D) not surjective
8. f(x) and g(x) are two functions defined for all real values of x. f(x) is an even function and g(x) is periodic
function, then :
(A) f [g(x)] is a periodic function (B) g [f(x)] is a periodic function
(C) f [g(x)] is an even function (D) g [f(x)] is an even function
JPR\COMP.251\Allen(IIT-JEE Wing)\2020–21\Enthusiast\Mathematics\Unit -6
48
Fu ncti on
Consider functions ƒ(x) = 4x 2 + 81x + 400, x Î R + , g(x) = x 2 + 20x + 100, x Î R + and h(x) = ƒ(x) – 2g(x).
æ 1ö æ 1ö æ 2ö æ 1ö
(A) çè 0, ÷ (B) çè 0, ÷ (C) çè 0, ÷ (D) çè 0, ÷ø
2ø 6ø 3ø 4
Let f (x) = | x + 1 | – | x – 4 | , x Î R.
13. If range of f (x) is [a, b]
(A) (– 5, 5] (B) [– 5, 5] (C) [– 5, 5) (D) None of these
14. If the equation f (x) = b has atleast one real solution then the number of integral values in the range of b, is
(A) 7 (B) 9 (C) 11 (D) None of these
15. If the equation f(|x|) = b has exactly two real solutions then the number of integral values in the range of b,
is
(A) 7 (B) 9 (C) 11 (D) None of these
JPR\COMP.251\Allen(IIT-JEE Wing)\2020–21\Enthusiast\Mathematics\Unit -6
49
JEE-Mathematics
2. If f (x) is a function that is odd and even simultaneously, then f (3) – f (2) is equal to :
3. Let f be a function such that f(3) = 1 and f(3x) = x + f(3x – 3) for all x. Then find the value of f(300).
9x æ æ 1 ö æ 2 ö æ 3 ö æ 2007 ö ö
4. Let f(x) = then find the value of the sum 2 ç f çè ÷ + fç ÷ + fç ÷ + .......f ç
x
9 +3 è 2008 ø è 2008 ø è 2008 ø è 2008 ÷ø ÷ø
é1 ö é3 ö
A function f : ê , ¥ ÷ ® ê , ¥ ÷ defined as, f(x) = x –x + 1. Then solve the equation f(x) = f (x).
2 –1
5.
ë2 ø ë4 ø
6. Suppose p(x) is a polynomial with integer coefficients. The remainder when p(x) is divided by x – 1 is 1 and the
remainder when p(x) is divided by x – 4 is 10. If r(x) is the remainder when p(x) is divided by (x –1)(x – 4), find
the value of r(2006).
7. Let f(x) = (x + 1)(x + 2)(x + 3)(x + 4) + 5 where x Î [–6, 6]. If the range of the function is [a, b] where
a, b Î N then find the value of (a + b).
é3ù é4ù
8. The set of real values of ‘x’ satisfying the equality ê ú + ê ú = 5 (where [ ] denotes the greatest integer func-
ëxû ëxû
æ bù b
tion) belongs to the interval ç a, where a, b, c Î N and is in its lowest form. Find the value of
è c úû c
a + b + c + abc.
Column – I Column – II
JPR\COMP.251\Allen(IIT-JEE Wing)\2020–21\Enthusiast\Mathematics\Unit -6
æ 1+ x ö
(A) y = ln ç is (p) odd
è 1 - x ÷ø
é 2 ù
(B) y = sin x ln ê x + 1 - x ú (q) even
ë û
(C) y = sgn (sin 5p x) is (r) constant function
50
Fu ncti on
2
(A) If f(x) = x – 4x + 3, then graph of f(|x|) is (p) o x
1
1
1
(B) If g(x) = , then it's graph is (q)
ln x –2 –1 O 1 2 3 x
2
(C) If f(x) = x – 4x + 3, then graph of |f(x)| is (r) –3 –1 1 3 x
1
(D) If k(x) = , then its graph is (s)
{ x} 1 3 x
Column - I Column - II
2x + 1
¦(x) =
3x + 4
sinx –sinx
¦(x) = e +e
51
JEE-Mathematics
3
1. For real x, let f(x) = x + 5x + 1, then :- [AIEEE - 2009]
(1) f is one-one and onto R
(2) f is neither one-one nor onto R
(3) f is one-one but not onto R
(4) f is onto R but not one-one
1
2. The domain of the function f(x) = is :- [AIEEE - 2011]
|x|- x
(1) (–¥, 0) (2) (–¥, ¥) – {0} (3) (–¥, ¥) (4) (0, ¥)
2
3. Let f be a function defined by f(x) = (x – 1) + 1, f : [1, ¥) ® [1, ¥). [AIEEE - 2011]
f -1 (x) = 1 + x - 1 , x > 1.
(1) Statement–1 is true, Statement–2 is false.
(2) Statement–1 is false, Statement–2 is true.
(3) Statement–1 is true, Statement–2 is true ; Statement–2 is a correct explanation for Statement–1.
(4) Statement–1 is true, Statement–2 is true ; Statement–2 is not a correct explanation for statement–1.
5. Statement-1 : Let A and B be two sets having p and q elements respectively, where q > p. Then the total
p
number of functions from set A to set B is q .
q
Statement-2 : The total number of selection of p different objects out of q objects is Cp.
[AIEEE ONLINE - 2012]
(1) Statement-1 is true, Statement-2 is true and Statement-2 is the correct explanation of Statement-1.
(2) Statement-1 is true, Statement-2 is true and Statement-2 is not the correct explanation of statement-1.
(3) Statement-1 is true, Statement-2 is false
(4) Statement-1 is false, Statement-2 is true
2 2
7. Let R = {(x, y) : x, y Î N and x – 4xy + 3y = 0}, where N is the set of all natural numbers. Then the relation
R is : [JEE-MAIN ONLINE - 2013]
(1) reflexive and transitive. (2) symmetric and transitive
(3) reflexive but neither symmetric nor transitive (4) reflexive and symmetric.
52
Fu ncti on
8. Let f be an odd function defined on the set of real numbers such that for x ³ 0, f(x) = 3sinx + 4cosx. Then
11p
f(x) at x = - is equal to [JEE-MAIN ONLINE-2014]
6
3 3 3 3
(1) -2 3 (2) - -2 3 (3) - +2 3 (4) +2 3
2 2 2 2
9. A relation on the set A = { x : | x | < 3, x Î Z}, where Z is the set of integers is defined by R = {(x, y)
: y = | x |, x ¹ –1}. Then the number of elements in the power set of R is :
[JEE-MAIN ONLINE-2014]
(1) 32 (2) 64 (3) 16 (4) 8
10. The function f(x) = |sin 4x| + |cos 2x|, is a periodic function with period : [JEE-MAIN ONLINE-2014]
p p
(1) 2p (2) (3) p (4)
2 4
æ 1ö
11. If f(x) + 2f ç ÷ = 3x, x ¹ 0, and S = {x Î R : f(x) = f(–x)} ; then S : [JEE MAINS - 2016]
è xø
(1) is an empty set (2) contains exactly one element
(3) contains exactly two elemwnts (4) contains more than two elements
1 1 x
12. The function f : R ® éê - , ùú defined as f(x) = is : [JEE MAINS - 2017]
ë 2 2û 1 + x2
(1) surjective but not injective (2) neither injective nor surjective
(3) invertible (4) injective but not surjective
éxù
13. The function f : N ® N defined by f(x) = x - 5 ê ú , (where N is the set of natural numbers and [x] denotes
ë5û
the greatest integer less than or equal to x), is : [JEE-MAIN ONLINE-2017]
(1) onto but not one-one (2) one-one and onto
(3) neither one-one nor onto (4) one-one but not onto
2x
14. Let A={x Î R : x is not a positive integer} Define a function f : A®R as f(x) = then f is :
x -1
[JEE-MAIN ONLINE-2019]
(1) injective but not surjective (2) not injective
(3) surjective but not injective (4) neither injective nor surjective
x
15. Let f : R ® R be defined by f(x) = , x Î R. Then the range of f is : [JEE-MAIN ONLINE-2019]
1 + x2
é 1 1ù é 1 1ù
(1) (–1, 1) – {0} (2) ê - , ú (3) R - ê - , ú (4) R – [–1, 1]
ë 2 2û ë 2 2û
1
16. Let a function f : (0, ¥) ® [0, ¥) be defined by f(x) = 1 - . Then f is : [JEE-MAIN ONLINE-2019]
JPR\COMP.251\Allen(IIT-JEE Wing)\2020–21\Enthusiast\Mathematics\Unit -6
x
(1) Injective only (2) Not injective but it is surjective
(3) Both injective as well as surjective (4) Neither injective nor surjective
ì ( x + 2)( x 2 - 5x + 6) ü
17. Let Z be the set of integers. If A = íx Î Z: 2 = 1ý and B = { x Î Z: -3 < 2x - 1 < 9} , then the
î þ
number of subsets of the set A × B, is : [JEE-MAIN ONLINE-2019]
18 10 15 12
(1) 2 (2) 2 (3) 2 (4) 2
53
JEE-Mathematics
æ1 - xö æ 2x ö
18. If ƒ(x) = log e ç ,|x |< 1 , then ƒ ç is equal to : [JEE-MAIN ONLINE-2019]
÷
è1 + xø è 1 + x 2 ÷ø
2 2
(1) 2ƒ(x) (2) 2ƒ(x ) (3) (ƒ(x)) (4) –2ƒ(x)
x2
19. If the function ƒ : R – {1, –1} ® A defined by ƒ(x) = , is surjective, then A is equal to
1 - x2
[JEE-MAIN ONLINE-2019]
(1) R – [–1, 0) (2) R – (–1, 0) (3) R – {–1} (4) [0, ¥)
1
20. The domain of the definition of the function f(x) = + log10 (x 3 - x) is : [JEE-MAIN ONLINE-2019]
2
4-x
(1) (1, 2) È (2, ¥) (2) (–1, 0) È (1, 2) È (3, ¥)
(3) (–1, 0) È (1, 2) È (2, ¥) (4) (–2, –1) È (–1, 0) È (2, ¥)
x 2
21. Let f(x) = e – x and g(x) x –x, "x Î R. Then the set of all x Î R, where the function h(x) = (fog) (x) is
increasing, is : [JEE-MAIN ONLINE-2019]
-1 1 1 -1
(1) éê -1, ùú È éê , ¥ ö÷ (2) éê0, ùú È [1, ¥ ) (3) éê ,0ùú È [1, ¥ ) (4) [ 0,¥ )
ë 2 û ë2 ø ë 2û ë 2 û
2 2 æ 5ö
22. If g(x) = x + x – 1 and (gof)(x) = 4x – 10x + 5, then f çè ÷ø is equal to[JEE-MAIN ONLINE-2020]
4
3 1 3 1
(1) (2) - (3) - (4)
2 2 2 2
1+x 1–x x –x
23. Let ƒ : R ® R be such that for all x Î R (2 +2 ), ƒ(x) and (3 + 3 ) are in A.P., then the minimum
value of ƒ(x) is [JEE-MAIN ONLINE-2020]
(1) 0 (2) 3 (3) 2 (4) 4
8 2x - 8 -2x
24. The inverse function of ƒ ( x ) = , x Î ( -1,1) , is [JEE-MAIN ONLINE-2020]
8 2x + 8 -2x
1 æ1 - xö 1 æ1 - xö 1 æ1+ xö 1 æ1+ xö
(1)
4
( log 8 e ) log e ç
è1+ xø÷ (2)
4
log e ç
è 1 + x ÷ø
(3)
4
( log 8 e ) log e ç ÷
è1 - xø
(4)
4
log e ç
è 1 - x ÷ø
JPR\COMP.251\Allen(IIT-JEE Wing)\2020–21\Enthusiast\Mathematics\Unit -6
54
Fu ncti on
y+3 y-3
(3) g(y) = (4) g(y) =
4 4
2
2. Let f(x) = (x + 1) – 1, f : [–1, ¥) ® [–1, ¥). [AIEEE - 2009]
–1
Statement–1 – The set {x : f(x) = f (x)} = {0, –1}.
Statement–2 – f is a bijection.
(1) Statement–1 is true, Statement–2 is false.
(2) Statement–1 is false, Statement–2 is true.
(3) Statement–1 is true, Statement–2 is true ; Statement–2 is a correct explanation for Statement–1.
(4) Statement–1 is true, Statement–2 is true ; Statement–2 is not a correct explanation for statement–1.
x
3. The range of the function f(x) = , x Î R, is :- [AIEEE ONLINE - 2012]
1+ |x |
(1) [–1, 1] (2) R (3) R – {0} (4) (–1, 1)
4. If P(S) denotes the set of all subsets of a given set S, then the number of one–to–one functions from the set
S = {1, 2, 3} to the set P(S) is :- [AIEEE ONLINE - 2012]
(1) 24 (2) 8 (3) 336 (4) 320
+ +
5. If A = {xÎz ; x < 10 and x is a multiple of 3 or 4}, where z is the set of positive integers, then the total
number of symmetric relations on A is :- [AIEEE ONLINE - 2012]
5 20 10 15
(1) 2 (2) 2 (3) 2 (4) 2
x2 - x d –1
6. Let f(x) = , x ¹ 0, –2. Then [f (x)] (wherever it is defined) is equal to :
2
x + 2x dx
[JEE-MAIN ONLINE - 2013]
3 -1 1 -3
(1) (2) (3) (4)
(1 - x)2 (1 - x)2 (1 - x)2 (1 - x)2
7. Let R = {(3, 3), (5, 5), (9, 9), (12, 12), (5, 12), (3, 9), (3, 12), (3, 5)} be a relation on the set
A = {3, 5, 9, 12}. Then, R is :- [JEE-MAIN ONLINE - 2013]
(1) reflexive, transitive but not symmetric
(2) symmetric, transitive but not reflexive.
(3) an equivalence relation
JPR\COMP.251\Allen(IIT-JEE Wing)\2020–21\Enthusiast\Mathematics\Unit -6
1
8. If g is the inverse of a function f and f '(x) = , then g ' (x) is equal to [JEE MAINS - 2014]
1 + x5
1 5 4
(1) (2) 1 + {g(x)}5 (3) 1 + x (4) 5x
1 + {g(x)}5
55
JEE-Mathematics
1 k k
9. Let fk (x) = (sin x + cos x) where x Î R and k ³ 1. Then f4(x) – f6(x) equals: [JEE MAINS - 2014]
k
1 1 1 1
(1) (2) (3) (4)
4 12 6 3
2 2
10. Let P be the relation defined on the set of all real numbers such that P = {(a, b) : sec a – tan b = 1}.
Then P is : [JEE-MAIN ONLINE - 2014]
(1) reflexive and transitive but not symmetric.
(2) reflexive and symmetric but not transitive
(3) symmetric and transitive but not reflexive
(4) an equivalence relation
x x
3 4
11. If f(x) = æç ö÷ + æç ö÷ - 1 , x Î R, then the equation f(x) = 0 has : [JEE-MAIN ONLINE-2014]
è 5ø è 5ø
(1) One solution (2) no solution
(3) more than two solutions (4) two solutions
2 1
12. If f(x) = x – x + 5, x > , and g(x) is its inverse function, then g'(7) equals :
2
[JEE-MAIN ONLINE-2014]
1 1 1 1
(1) (2) - (3) - (4)
3 3 13 13
56
é 1 3n ù
13. Let f(n) = ê +
ë 3 100 úû
n , where [n] denotes the greatest integer less than or equal to n. Then å f(n) is equal
n= 1
to : [JEE-MAIN ONLINE-2014]
(1) 56 (2) 1399 (3) 689 (4) 1287
15. In a certain town, 25% of the families own a phone and 15% own a car; 65% families own neither a phone
nor a car and 2,000 families own both a car and a phone. Consider the following three statements
(a) 5% families own both a car and a phone.
(b) 35% families own either a car or a phone.
(c) 40,000 families live in the town.
Then, [JEE-MAIN ONLINE - 2015]
(1) Only (a) and (b) are correct (2) Only (a) and (c) are correct
(3) Only (b) and (c) are correct (4) All (a), (b) and (c) are correct
JPR\COMP.251\Allen(IIT-JEE Wing)\2020–21\Enthusiast\Mathematics\Unit -6
16. Let A = {x1, x2,..., x7} and B {y1, y2, y3} be two sets containing seven and three distinct elements respectively.
Then the total number of functions f : A ® B that are onto, if there exist exactly three elements x in A such that
f(x) = y2, is equal to : [JEE-MAIN ONLINE - 2015]
7 7 7 7
(1) 14. C2 (2) 16. C3 (3) 14. C3 (4) 12. C2
56
Fu ncti on
1
17. For x Î R, x ¹ 0, x ¹ 1, let f0 (x) = and fn+1(x) = f0(fn(x)), n = 0, 1, 2, ..... Then the value of
1-x
æ 2ö æ 3ö
f100 (3) + f1 ç ÷ + f2 ç ÷ is equal to : [JEE-MAIN ONLINE - 2016]
è 3ø è 2ø
8 5 1 4
(1) (2) (3) (4)
3 3 3 3
18. Let a, b, c Î R. If f(x) = ax2 +bx +c is such that a +b + c = 3 and f(x+y) = f(x) + f(y) + xy, " x, y Î R,
10
10 10
19. Let f(x) = 2 ·x + 1 and g(x) = 3 · x – 1. If (fog) (x) = x, then x is equal to :
[JEE-MAIN ONLINE - 2017]
20. Let f be a polynomial function such that f(3x) = f'(x) · f''(x), for all x Î R. Then :
[JEE-MAIN ONLINE - 2017]
(1) f''(2) – f'(2) = 0 (2) f(2) + f'(2) = 28
(3) f(2) – f'(2) + f''(2) = 10 (4) f''(2) – f(2) = 4
{
B = (a, b) Î R ´ R : 4(a - 6)2 + 9(b - 5)2 £ 36 . Then : } [JEE-MAIN-2018]
(1) B Ì A (2) A Ì B
(3) A Ç B = f (an empty set) (4) Neither A Ì B nor B Ì A
1 1
22. For x Î R – {0, 1}, let f1(x) = , f2(x) = 1 – x and f3(x) = be three given functions. If a function,
x 1- x
J(x) satisfies (f2oJ°f1)(x) = f3(x) then J(x) is equal to : [JEE-MAIN ONLINE-2019]
1
(1) f3(x) (2) f1(x) (3) f2(x) (4) f (x)
x 3
23. In a class of 140 students numbered 1 to 140, all even numbered students opted mathematics course, those
whose number is divisible by 3 opted Physics course and theose whose number is divisible by 5 opted Chemistry
course. Then the number of students who did not opt for any of the three courses is :
[JEE-MAIN ONLINE-2019]
(1) 102 (2) 42 (3) 1 (4) 38
JPR\COMP.251\Allen(IIT-JEE Wing)\2020–21\Enthusiast\Mathematics\Unit -6
24. Let N be the set of natural numbers and two functions f and g be defined as f,g : N ® N
æn+1
if n is odd
ç
such that : f ( n ) = ç 2 and g(n) = n–(–1)n. The fog is : [JEE-MAIN ONLINE-2019]
ç n
if n is even
è 2
(1) Both one-one and onto (2) One-one but not onto
(3) Neither one-one nor onto (4) onto but not one-one
57
JEE-Mathematics
x
25. Let ƒ(x) = a (a > 0) be written as ƒ(x) = ƒ1(x) + ƒ2(x), where ƒ1(x) is an even function of ƒ2(x) is an odd
function. Then ƒ1(x + y) + ƒ1(x – y) equals [JEE-MAIN ONLINE-2019]
(1) 2ƒ1(x)ƒ1(y) (2) 2ƒ1(x)ƒ2(y) (3) 2ƒ1(x + y)ƒ2(x – y) (4) 2ƒ1(x + y)ƒ1(x – y)
10
26. Let å ƒ(a + k) = 16 ( 210 - 1) , where the function ƒ satisfies ƒ(x + y) = ƒ(x)ƒ(y) for all natural numbers x,
k =1
y and ƒ(1) = 2. then the natural number 'a' is [JEE-MAIN ONLINE-2019]
(1) 4 (2) 3 (3) 16 (4) 2
2
27. Le f(x) = x , x Î R. For any A Í R , define g(A) = {x Î R, f(x) Î A}. If S = [0, 4], then which one of the
following statements is not true ? [JEE-MAIN ONLINE-2019]
(1) f(g(S)) ¹ f(S) (2) f(g (S)) = S (3) g(f(S)) = g(S) (4) g(f(S)) ¹ S
æ 3ö 1 - x2 æ pö
28. For x Î ç 0, ÷ , let f ( x ) = x , g(x) = tan x and h(x) = . If f(x) = ((hof)og)(x), then f = çè ÷ø is equal
è 2ø 1+ x 2 3
to : [JEE-MAIN ONLINE-2019]
p 7p 11p 5p
(1) tan (2) tan (3) tan (4) tan
12 12 12 12
29. For x Î R, let [x] denote the greatest integer £ x, then the sum of the series
é 1ù é 1 1 ù é 1 2 ù é 1 99 ù
êë - 3 úû + êë - 3 - 100 úû + êë - 3 - 100 úû + ..... + êë - 3 - 100 úû is [JEE-MAIN ONLINE-2019]
x[x]
30. Let ƒ : (1,3) ® R be a function defined by ƒ(x) = , where [x] denotes the greatest integer £ x. Then the
1 + x2
range of ƒ is [JEE-MAIN ONLINE-2020]
3 4 2 3 3 4 2 4 2 1 3 4
(1) æç , ö÷ (2) æç , ùú È çæ , ÷ö (3) æç , ùú (4) æç , ÷ö È çæ , ùú
è 5 5ø è 5 5û è 4 5ø è 5 5û è 5 2ø è 5 5 û
JPR\COMP.251\Allen(IIT-JEE Wing)\2020–21\Enthusiast\Mathematics\Unit -6
58
Fu ncti on
2
1. Let f(x) = x and g(x) = sinx for all xÎR. Then the set of all x satisfying
(ƒ o g o g o ƒ) (x) = (g o g o ƒ) (x), where (ƒ o g) (x) = ƒ(g(x)), is- [JEE 2011, 3, (–1)]
{
(A) ± np, n Î 0,1,2,.... } (B) ± np , n Î{1, 2,.....}
p
(C) +2np,n Î {...., –2, –1,0,1,2,.....} (D) 2np,n Î {...., –2, –1,0,1,2,.....}
2
3 2
2. The function ƒ : [0, 3] ® [1, 29], defined by ƒ(x) = 2x – 15x + 36x + 1, is : [JEE 2012, 3, –1]
(A) one-one and onto (B) onto but not one-one
(C) one-one but not onto (D) neither one-one nor onto
2 æ pö æp pö æ1ö
3. Let ƒ : (–1,1) ® IR be such that ƒ(cos4q ) = for q Î ç 0, ÷ È ç , ÷ . Then the value(s) of ƒ ç ÷ is
2
2 – sec q è 4ø è4 2ø è3ø
(are)- [JEE 2012, 4]
3 3 2 2
(A) 1 – (B) 1+ (C) 1 – (D) 1+
2 2 3 3
4.
æ p pö
Let f : ç - , ÷ ® ¡ be given by f(x) = (log(sec x + tan x)) 3. Then [JEE 2014, 4]
è 2 2ø
(A) f(x) is an odd function (B) f(x) is a one-one function
(C) f(x) is an onto function (D) f(x) is an even function
5. Let X be a set with exactly 5 elements and Y be a set with exactly 7 elements. If a is the number of one-one
1
functions from X to Y and b is the number of onto functions from Y to X, then the value of (b - a) is____
5!
[JEE 2018]
6. Answer the following by appropriately matching the lists based on the information given in the
paragraph [JEE 2019]
Let ƒ(x) = sin(p cosx) and g(x) = cos(2p sinx) be two functions defined for x > 0. Define the following sets
whose elements are written in the increasing order :
X = {x : ƒ(x) = 0}, Y = {x : ƒ'(x) = 0}, Z = {x : g(x) = 0}, W = {x : g'(x) = 0}.
List-I contains the sets X,Y,Z and W. List -II contains some information regarding these sets.
List-I List-II
ì p 3p ü
(I) X (P) Ê í , ,4 p,7pý
î2 2 þ
(II) Y (Q) an arithmetic progression
(III) Z (R) NOT an arithmetic progression
ì p 7 p 13p ü
(IV) W (S) Êí , , ý
î6 6 6 þ
JPR\COMP.251\Allen(IIT-JEE Wing)\2020–21\Enthusiast\Mathematics\Unit -6
ì p 2p ü
(T) Ê í , , pý
î3 3 þ
ì p 3p ü
(U) Êí , ý
î6 4 þ
59
JEE-Mathematics
7. Answer the following by appropriately matching the lists based on the information given in the
paragraph [JEE 2019]
Let ƒ(x) = sin(p cosx) and g(x) = cos(2p sinx) be two functions defined for x > 0. Define the following sets
whose elements are written in the increasing order :
X = {x : ƒ(x) = 0}, Y = {x : ƒ'(x) = 0}.
Z = {x : g(x) = 0}, W = {x : g'(x) = 0}.
List-I contains the sets X,Y,Z and W. List -II contains some information regarding these sets.
List-I List-II
ì p 3p ü
(I) X (P) Ê í , ,4 p,7pý
î2 2 þ
(II) Y (Q) an arithmetic progression
(III) Z (R) NOT an arithmetic progression
ì p 7 p 13p ü
(IV) W (S) Êí , , ý
î6 6 6 þ
ì p 2p ü
(T) Ê í , , pý
î3 3 þ
ì p 3p ü
(U) Êí , ý
î6 4 þ
JPR\COMP.251\Allen(IIT-JEE Wing)\2020–21\Enthusiast\Mathematics\Unit -6
60
Fu ncti on
ANSWER KEY
EXERCISE-1
Que. 1 2 3 4 5 6 7 8 9 10
Ans. C C A A C C C D C A
Que. 11 12 13 14 15 16 17 18 19 20
Ans. B A C A D C A B A A
Que. 21 22 23 24 25 26 27 28 29 30
Ans. D C D A A B B B C C
EXERCISE-2
Que. 1 2 3 4 5 6 7 8 9 10
Ans. BD CD ACD ABD AC BD ABC AD AD AB
Que. 11 12 13 14 15
Ans. D C B C A
EXERCISE-3
Que. 1 2 3 4 5 6 7 8
Ans. 2 0 5050 2007 1 6016 5049 20
Que. 9 10 11
Ans. (A) – (p), (B) – (q), (C)– (p), (D)–(s) (A) – (r), (B) – (p), (C)– (s), (D)–(q) (A) – (r,q), (B) – (p,s), (C)– (r,s), (D)–(r,s)
EXERCISE-4(A)
Que. 1 2 3 4 5 6 7 8 9 10
Ans. 1 1 3 1 2 1 3 1 3 2
Que. 11 12 13 14 15 16 17 18 19 20
Ans. 3 1 3 1 2 2 3 1 1 3
Que. 21 22 23 24
Ans. 2 2 2 3
EXERCISE-4(B)
Que. 1 2 3 4 5 6 7 8 9 10
Ans. 4 4 4 3 4 1 1 2 2 4
Que. 11 12 13 14 15 16 17 18 19 20
Ans. 1 1 2 2 4 3 2 3 4 1
Que. 21 22 23 24 25 26 27 28 29 30
Ans. 2 1 4 4 1 2 3 3 2 4
JPR\COMP.251\Allen(IIT-JEE Wing)\2020–21\Enthusiast\Mathematics\Unit -6
EXERCISE-5
Que. 1 2 3 4 5 6 7
Ans. A B AB ABC 119 C B
61
62
JEE-Mathematics
IMPORTANT NOTES
JPR\COMP.251\Allen(IIT-JEE Wing)\2020–21\Enthusiast\Mathematics\Unit -6
INVERSE TRIGONOMETRIC
CHAPTER
2 FUNCTIONS
1.0 INTRODUCTION
EXERCISE-1
EXERCISE-2
EXERCISE-3
EXERCISE-4(A)
EXERCISE-4(B)
EXERCISE-5
Inverse Trig onometric Fu nctions
The inverse trigonometric functions, denoted by sin–1x or (arc sinx), cos–1x etc., denote the angles whose sine,
cosine etc, is equal to x. The angles are usually the numerically smallest angles, except in the case of cot–1x, and
if positive & negative angles have same numerical value, the positive angle has been chosen.
It is worthwhile noting that the functions sinx, cosx etc are in general not invertible. Their inverse is defined by
choosing an appropriate domain & co-domain so that they become invertible. For this reason the chosen value
is usually the simplest and easy to remember.
æ p pö
(3) tan–1x xÎR ç- , ÷
è 2 2ø
ìpü é pö æp ù
(4) sec–1x |x| ³ 1 ëé0, pûù - í 2 ý or ê0, ÷ È ç , pú
î þ ë 2ø è2 û
é p pù
(5) cosec–1x |x| ³ 1 ê - 2 , 2 ú - {0}
ë û
(6) cot–1x xÎR (0, p)
é -p p ù
(a) f : ê , ú ® [-1, 1] f -1 : [-1, 1] ® [–p/2, p/2]
ë 2 2û
y y
y=arcsinx y=arc sinx
p/2 y=x p/2
1 y=sinx
–p/2 –1 –1
x x
0 1 p/2 0 1
JPR\COMP.251\Allen(IIT-JEE Wing)\2020–21\Enthusiast\Mathematics\Unit-6
y=sinx –1
63
JEE-Mathematics
y
y=arc cosx y=x y
p
p/2 p
1
p/2 p p/2
0 x
–1
–1
y=cosx –1 1 x
O
y=x
p/2
y=arc tanx
p/2
y=arc tanx
x x
–p/2 0 p/2 p 0
y=arc tanx y=arc tanx
p/2 –p/2
y=x
y=tanx –p
p/2
y=arc cotx p/2
0 x y=arc cotx
0 x
–p/2
y=x –p
y=cotx
y
é p ö æ pù
(f) f : ê - ,0÷ È ç 0, ú ® ( -¥, –1] È [1, ¥)
ë 2 ø è 2û p/2
f(x) = cosec x
é p ö æ pù x
f -1 : ( -¥, –1] È [1, ¥ ) ® ê - ,0÷ È ç 0, ú –1 0
ë 2 ø è 2û
1 æ 1ö
Illustration 1. The value of tan–1(1) + cos–1 æç - ö÷ + sin–1 çè - 2 ÷ø is equal to
è 2ø
p 5p 3p 13p
(A) (B) (C) (D)
4 12 4 12
æ 1ö æ 1 ö p 2p - p = p + p = 3 p
Solution tan–1 ( 1) + cos–1 ç - ÷ + sin–1 ç - ÷ = + Ans.(C)
è 2ø è 2ø 4 3 6 4 2 4
2n 2n
Illustration 2. If å cos -1
x i = 0 then find the value of åx i
i =1 i =1
2n
Þ åx i = 2n Ans.
i =1
65
JEE-Mathematics
4.0 PROPERTIES OF INVERSE CIRCULAR FUNCTIONS
SL AL
y
P-1 (i) y = sin (sin–1x) = x 1
x
x Î [–1,1], y Î [–1,1] y=
45°
–1 O +1 x
–1
y
(ii) y = cos (cos–1 x) = x
1
x
x Î [–1,1], y Î [–1,1] y=
45°
–1 O +1 x
–1
(iii) y = tan(tan–1 x) = x x
y=
45° x
x Î R, y Î R O
y
(iv) y = cot(cot–1 x) = x,
x
y=
x ÎR; y ÎR x
O
y
x
=
y
1
(v) y = cosec (cosec–1 x) = x,
–1 O 1 x
|x| > 1, |y| > 1
–1
x
=
y
y
JPR\COMP.251\Allen(IIT-JEE Wing)\2020–21\Enthusiast\Mathematics\Unit-6
x
(vi) y = sec(sec–1 x) = x
=
y
–1 O 1 x
–1
x
66
Inverse Trig onometric Fu nctions
æp æ -1 ö ö æ p æ pöö 2p 3
(iii) sin ç - sin -1 ç ÷ ÷ = sin ç - ç - ÷ ÷ = sin = Ans.
è2 è 2 øø è 2 è 6ø ø 3 2
æ x - 3ö
3. Find domain of following functions f(x) = sin -1 ç - log10 (4 - x)
è 2 ÷ø
-1 æ 2 ö
4. Find domain of following functions f ( x ) = cos ç
è 2 + sin x ÷ø
6. Find the domain and range of the following functions : f(x) = cot –1 (2x – x2)
8. Draw graph of following functions : g(x) = [cos –1x], where [ ] denotes greatest integer function.
67
JEE-Mathematics
é p pù
P-2 (i) y = sin–1 (sin x), x Î R, y Î ê - , ú periodic with period 2p and it is an odd function.
ë 2 2û
y
ì p
ï -p - x , -p £ x £ - p
2
y
x
2
2p
=
ï
=
x
2p
x–
–(
y=
p–
ï p p p 3p
p+
=
=
x
–
sin -1 (sin x) = í x , - <x<
y
2
x)
y
45° 2
ï 2 2 –2p
–
3p –p O p p 2p x
2 2
ï p
ïp - x , £x£p
î 2
p
–
2
(ii) y = cos–1 (cos x), x Î R, y Î [0,p], periodic with period 2p and it is an even function.
y
p
x
y
2p
ì - x , -p £ x £ 0 p =
=
–
cos -1 (cos x) = í +
2 y
2p
–x
x
îx , 0 < x £ p
–x
=
y
x
– 2p –p p O p p 2p
– 2
2
(iii) y = tan–1 (tan x)
ì p ü æ p pö
x Î R – í(2n - 1) ,n Î I ý ; y Î ç - , ÷ , periodic with period p and it is an odd function.
î 2 þ è 2 2ø
y
ì 3p p
ïx + p , -
2
<x<-
2
ï
p
p
x
x+
x–
y=
ï p p
tan -1 (tan x) = í x
y=
- <x<
=
, 2p
y
ï 2 2 –2p 3p –p –p O x
ï p 3p 2 2
ïx - p , <x<
î 2 2
p
–
2
(iv) y = cot–1(cot x), x Î R – {n p, n Î I}, y Î (0, p), periodic with period p and neither even nor odd function.
y
p
p
2p
ì x + p , -p < x < 0
+
x–
x
+
ï
y=
y=
=
x
-1
cot (cot x) = í x , 0<x<p
=
y
y
ï x - p , p < x < 2p
î
–2p –p O p 2p
é p ö æ pù
(v) y = cosec–1 (cosec x), x Î R – {n p, n Î I} y Î ê - ,0÷ È ç 0, ú , is periodic with period 2p and it is an
ë 2 ø è 2û
odd function.
JPR\COMP.251\Allen(IIT-JEE Wing)\2020–21\Enthusiast\Mathematics\Unit-6
p
2p
y
y=
2
=
x–
y=
p 3p
–(
p–
–
p+
2 45° 2
y
x)
3p –p O p p 2p x
–
2 2
p
–
2
68
Inverse Trig onometric Fu nctions
y
x –
=
2
y=
–x
2p
ì p ü é pö æ p ù
x Î R – í(2n - 1) n Î I ý , y Î ê0, ÷ È ç , p ú
–
x
î 2 þ ë 2ø è 2 û –2p –p O p 2p x
3p p p 3p
– –
2 2 2 2
æ 3ö æ æ 7p ö ö
Illustration 4. The value of sin -1 ç - ÷ + cos -1 ç cos ç ÷ ÷ is -
è 2 ø è è 9 øø
5p p 3p
(A) (B) (C) (D) None of these
6 2 2
æ 3ö æ 3ö p
Solution sin -1 ç - ÷ = - sin -1 ç ÷ =-
è 2 ø è 2 ø 3
æ æ 7p ö ö æ 5p ö -1 æ 5 p ö 5p
and cos -1 ç cos ç ÷ ÷ = cos -1 cos ç 2p - ÷ = cos çè ÷ø =
è è 6 øø è 6ø 6 6
æ 3ö æ 7p ö p 5p p
Hence sin -1 ç - ÷ + cos -1 ç cos ÷ = - + = Ans.(B)
è 2 ø è 6 ø 3 6 2
æ æ pöö -1 æ 7p ö
(i) sin -1 ç sin ç ÷ ÷ (ii) cos çè cos ÷ø
è è 4øø 6
æ æ pöö p
Solution (i) sin -1 ç sin ç ÷ ÷ =
è è 4øø 4
æ 7p ö 7p 7p
(ii) cos–1 ç cos ÷ ¹ , because does not lie between 0 and p.
è 6 ø 6 6
æ 7p ö æ æ 5p ö ö ì 7p 5p ü
Now, cos–1 çè cos ÷ø = cos–1 çè cos çè 2p - ÷÷ íQ = 2p - ý
6 6 øø î 6 6 þ
æ 5p ö 5p
= cos -1 ç cos ÷ = Ans.
è 6ø 6
Illustration 6. Evaluate the following :
(i) sin –1(sin10) (ii) tan–1(tan (– 6)) (iii) cot–1(cot 4)
JPR\COMP.251\Allen(IIT-JEE Wing)\2020–21\Enthusiast\Mathematics\Unit-6
p p
Solution (i) We know that sin–1(sinq) = q, if - £q£
2 2
p p
Here, q = 10 radians which does not lie between - and
2 2
p p
But, 3p – q i.e., 3p – 10 lie between – and
2 2
Also, sin(3p – 10) = sin 10
\ sin–1(sin 10) = sin–1 (sin (3p – 10)) = (3p – 10)
69
JEE-Mathematics
(ii) We know that,
p p
tan–1(tanq) = q, if - <q< .
2 2
p p
Here, q = –6, radians which does not lie between - and .
2 2
p p
We find that 2p – 6 lies between - and such that;
2 2
tan (2p – 6) = –tan 6 = tan(–6)
\ tan–1(tan(–6)) = tan–1 (tan(2p – 6)) = (2p – 6)
(iii) cot–1(cot4) = cot–1(cot(p + (4 – p))) = cot–1(cot(4 – p)) = (4 – p) Ans.
2 2
ì æ 2 öü ì æ 3 öü
= ísec ç tan -1 ÷ ý + ícos ec ç cot -1 ÷ ý
î è 1 øþ î è 1 øþ
{ ( )} + {cosec ( cosec )} = ( 5 ) + ( )
2 2 2 2
= sec sec -1 5 –1
10 10 = 15
Illustration 8. Find the number of solutions of (x, y) which satisfy |y| = cos x and y = sin –1(sin x), where |x| £ 3p.
Solution Graphs of y = sin–1(sinx) and |y| = cosx meet exactly six times in [–3p, 3p].
5p p 3p
–3p 2 2 O 2 3p
x
–2p 3p p 2p 5p
2 2 2
p
P-3 (i) sin–1 x + cos–1 x = –1 < x < 1
2
p
(ii) tan–1 x + cot–1 x = xÎR
2
p
(iii) cosec–1 x + sec–1 x = |x| > 1
2
JPR\COMP.251\Allen(IIT-JEE Wing)\2020–21\Enthusiast\Mathematics\Unit-6
70
Inverse Trig onometric Fu nctions
1
P-5 (i) cosec–1 x = sin–1 ; x <–1, x >1
x
1
(ii) sec–1 x = cos–1 ; x <–1, x >1
x
ì -1 1
ïï tan x ; x>0
-1
(iii) cot x = í
ï p + tan -1 1 ; x < 0
ïî x
-1 -1 1 ì p / 2 , if x > 0
Illustration 9. Prove that tan x + tan =í
x î -p / 2 , if x < 0
æ 1 ö ì cot x
-1
, for x > 0
Solution. We have, tan -1 ç ÷ = í
è x ø î -p + cot x , for x < 0
-1
PROPERTIES OF ITF
Evaluate the following
1. (a) sin -1 sin 4 (b) cos -1 cos10
-1 æ 5p ö æ æ 5p ö ö
2. (a) cos cos çè ÷ø (b) sin -1 ç sin ç ÷÷
4 è è 6 øø
-1 æ 3p ö
3. (a) cot -1 cot ( -10) (b) tan ç tan
è 4 ÷ø
æ 1 æ 9p 9p ö ö
4. cos -1 ç ç cos - sin ÷ ÷
è 2è 10 10 ø ø
æ æ 1ö ö
5. cos ç 2 tan-1 ç ÷ ÷ equals -
è è 7ø ø
(A) sin(4 cot -1 3) (B) sin(3 cot -1 4) (C) cos(3 cot -1 4) (D) cos(4 cot -1 4)
æ -p 3p ö æ 3p ù
(A) (0, p) (B) çè , ÷ (C) çè 0, (D) none
2 2ø 4 úû
1
7. Find the value of sin(tan –1a + tan –1 ); a ¹ 0
a
p
8. Solve the equation : sin –1 (x 2 – 2x + 1) + cos –1(x 2 – x) =
2
71
JEE-Mathematics
x+y
ì tan–1 where x > 0, y > 0 & xy < 1
ï 1 - xy
ïï x+y
P-6 (i) (a) tan–1 x + tan–1 y = í p +tan–1 where x > 0, y > 0 & xy > 1
1 - xy
ï
ï p
ïî 2 , where x > 0, y > 0 & xy = 1
x-y
(b) tan–1 x – tan–1 y = tan–1 where x > 0, y > 0
1 + xy
é x + y + z - xyz ù
(c) tan–1x + tan–1y + tan–1 z = tan–1 ê ú if x > 0, y > 0, z > 0 & xy + yz + zx < 1
ë 1 - xy - yz - zx û
ìïsin -1 [x 1 - y 2 + y 1 - x 2 ] where x > 0, y > 0 & (x 2 + y 2 ) £ 1
(ii) (a) sin –1 x + sin –1 y = í
ïî p - sin -1 [x 1 - y 2 + y 1 - x 2 ] where x > 0, y > 0 & x 2 + y 2 > 1
ì 1 1 ü
+ ì -1 æ x + y ö ü
ïï ïï íQ tan x + tan y = tan ç
= tan -1 í 7 13 ý
-1 -1
÷ ; if xy < 1ý
1 1 î è 1 - xy ø ïþ
ï1 - ´ ï
îï 7 13 þï
æ 20 ö æ 2ö
= tan -1 ç ÷ = tan -1 ç ÷ = R.H.S.
è 90 ø è 9ø
æ -1 1 1ö æ 1 1ö
(ii) çè tan + tan-1 ÷ + ç tan-1 + tan-1 ÷
5 7ø è 3 8ø
æ 1 1 ö æ 1 1 ö
+ +
JPR\COMP.251\Allen(IIT-JEE Wing)\2020–21\Enthusiast\Mathematics\Unit-6
ç 5 7 ÷
-1 -1 ç 3 8 ÷ æ 6ö æ 11 ö
= tan ç + tan = tan -1 ç ÷ + tan-1 ç ÷
1 1÷ ç 1 1÷ è 17 ø è 23 ø
ç1- ´ ÷ ç1- ´ ÷
è 5 7ø è 3 8ø
æ 6 11 ö
ç 17 + 23 ÷
-1 -1 æ 325 ö -1 p
= tan ç ÷ = tan ç ÷ = tan (1) = Ans.
çç 1 - 6 11 ÷÷ è 325 ø 4
´
è 17 23 ø
72
Inverse Trig onometric Fu nctions
12 4 63
Illustration 11. Prove that sin -1 + cot -1 + tan -1 =p
13 3 16
Solution We have,
12 4 63
sin -1 + cos -1 + tan -1
13 5 16
12 3 63 é - 1 12 -1 12 -1 4 -1 3 ù
êQ sin 13 = tan 5 and cos 5 = tan 4 ú
-1
= tan + tan -1 + tan -1
5 4 16 ë û
ì 12 3 ü
ï + ï 63 é ù
p + tan í 5 4 ý + tan-1
-1
-1 æ x + y ö
êQ tan x + tan y = p + tan ç
-1 -1
= 12 3 16 ÷ ,if xy > 1ú
ï1 - ´ ï ë è 1 - xy ø û
î 5 4þ
-1 æ 63 ö -1 æ 63 ö
= p + tan ç -16 ÷ + tan ç 16 ÷
è ø è ø
63 63 éëQ tan -1 ( - x) = - tan -1 x ùû
= p - tan -1 + tan -1
16 16
= p
-1 12 3 56
Illustration 12. Prove that : cos + sin -1 = sin -1
13 5 65
12 3 -1 5 3 é -1 12 -1 5 ù
Solution We have, cos -1 + sin -1 = sin + sin -1 êQ cos 13 = sin 13 ú
13 5 13 5 ë û
ì 2 2ü
-1 ï 5 æ3ö 3 æ 5 ö ï
= sin í ´ 1 - ç ÷ + ´ 1 - ç ÷ ý
ïî13 è5ø 5 è 13 ø ï
þ
-1 ì 5 4 3 12 ü - 1 56
= sin í ´ + ´ ý = sin
î13 5 5 13 þ 65
ì 1
x = cosec(tan–1(cos(cot–1(sec(sin–1a))))) í Let sinq = a Þ secq =
î 1 - a2
= cosec(tan–1(cos(cot–1(secq))))
æ æ æ æ 1 öööö ì -1 æ 1 ö 1
Þ x = cosec ç tan -1 ç cos ç cot -1 ç ÷ í Let cot ç = f Þ cot f =
2 ÷÷÷ è 2 ÷
ø
è è è è ø
1 - a øøø î 1 - a 1 - a2
1
JPR\COMP.251\Allen(IIT-JEE Wing)\2020–21\Enthusiast\Mathematics\Unit-6
æ æ 1 öö ì æ 1 ö 1
Þ x = cosec ç tan -1 ç í Let, tan -1 ç = y Þ tan y =
è è 2 - a 2 ÷ø ÷ø î è 2 - a 2 ÷ø 2 - a2
Þ x= 3 - a2 ...... (i)
73
JEE-Mathematics
ì 1
and y = sec(cot–1(sin(tan–1(cosec(cos–1 a))))) í Let cos–1a = a Þ cosa = aÞ coseca =
î 1 - a2
= sec(cot–1(sin(tan–1(coseca))))
æ æ æ æ 1 öööö ì 1 1
Þ y = sec ç cot -1 ç sin ç tan -1 ç tan -1 = b Þ tan b =
÷÷ í Let,
è 1 - a2 ÷ø ÷ø ø ø
2
è è è î 1-a 1 - a2
1
= sec(cot–1(sin(b))) Þ sin b =
2 - a2
ìï 1 1
æ 1 öö
y = sec cot -1 æ
-1
Þ íLet cot = g Þ cot g = Þ sec g = 3 - a 2
ç çè ÷÷ ïî 2 - a2 2-a 2
è 2 - a2 ø ø
= sec g
Þ y = 3 - a2 ...... (ii)
PROPERTIES OF ITF
-1 4 3 27
2. Prove that cos + tan -1 = tan -1
5 5 11
2p
3. If sin–1 x + sin–1y = , then cos–1x + cos–1y is equal to-
3
2p p p
(A) (B) (C) (D) p
3 3 6
-1 -1 1 1 p
4. Prove that tan 1 + tan + tan -1 =
2 3 2
p
5. Solve for x, tan–1 2x + tan–1 3x =
4
1 p
6. Solution of the equation cot -1 x + sin -1 = is:
5 4
1
(A) x = 3 (B) x = (C) x = 0 (D) None of these
JPR\COMP.251\Allen(IIT-JEE Wing)\2020–21\Enthusiast\Mathematics\Unit-6
x - 1ö æ x + 1ö p
8. Solve for x : tan-1 æç + tan -1 ç =
è x - 2 ÷ø è x + 2 ÷ø 4
74
Inverse Trig onometric Fu nctions
–p/2
y
p
D p/2 I
-1
æ 1 - x2 ö é 2 tan -1 x if x ³ 0
(b) y = f(x) = cos ç ÷ =ê
è 1 + x2 ø êë -2 tan -1 x if x < 0
–1 0 1 X
y
p/2
I I
é 2 tan -1 x if |x|< 1 x
-1 2x
ê -1
–1 1
(c) y = f(x) = tan = ê p + 2 tan x if x < -1
1 - x2 ê I
-1 I
êë - (p - 2 tan x) if x > 1
–p/2
y
(d) y = f(x) = sin (3x – 4x )
–1 3 p/2
é -1 1
ê -(p + 3sin x) if -1 £ x < - 2 D I D
ê
1 1 – 3/2
= ê 3sin-1 x
–1/2 +1/2
if - £ x £ x
ê –1 3/2 1
2 2 I
ê 1
ê p - 3sin-1 x if < x£1
êë 2
–p/2
y
JPR\COMP.251\Allen(IIT-JEE Wing)\2020–21\Enthusiast\Mathematics\Unit-6
p
(e) y = f(x) = cos -1 (4x 3 - 3x)
D I D
é -1 1
ê3cos x - 2p if -1 £ x < - 2 p/2
ê
1 1
= ê2p - 3cos-1 x if - £ x £
ê 2 2
ê 1 I
ê 3cos-1 x if < x£1 –1/2 +1/2 x
êë 2 –1 – 3 /2 3/2 1
75
JEE-Mathematics
p/2
ì ( -1 ) 1
ï - p + 2sin x -1 £ x < -
2 1
ï – x
2
-1 ( 2) ï -1 1 1
-
(f) sin 2x 1 x = í2sin x - £x£ 1
ï 2 2
2
ï -1 1
ï p - 2sin x < x£1
î 2 -p/2
y
p
x
–1
2c
s
co
os
–2
–1
-1 (
ìï2cos -1 x 0 £ x £ 1
)
x
2p
2
(g) cos 2x - 1 = í -1
ïî2p - 2cos x -1 £ x < 0 –1 0 1 x
ì 1 pü ïì1 -1 5 ïü
Illustration 14. Evalulate : (i) tan í2 tan -1 - ý (ii) tan í cos ý
î 5 4þ ïî 2 3 ïþ
ì æ 1 ö ü
ì 1 pü ï ç 2´ 5 ÷ ï é -1 æ 2x ö ù
Solution (i) tan í2 tan -1 - ý = tan ïí tan -1 ç ï
÷ - tan 1ý êQ2 tan x = tan ç
-1 -1
2 ÷
, if|x|< 1ú
î 5 4þ ï çç 1 - 1 ÷÷ ï ë è 1 - x ø û
îï è 25 ø þï
ì æ 5 öü
ïï -1 ç 12 - 1 ÷ ïï ì -1 æ -7 ö ü -7
ì -1 5 -1 ü tan í tan ç ÷ ý = tan í tan ç ÷ý =
= tan í tan - tan 1 ý =
î 12 þ ï çç 1 + 5 ÷÷ ï î è 17 ø þ 17
îï è 12 ø þï
5 5
(ii) Let cos-1 = q. Then, cosq = , 0 < q < p/2
3 3
æ1 5ö
Now, tan çç cos -1 ÷
è2 3 ÷ø
5
1-
q 1 - cos q 3
= tan = =
JPR\COMP.251\Allen(IIT-JEE Wing)\2020–21\Enthusiast\Mathematics\Unit-6
2 1 + cos q 5
1+
3
3- 5 (3 - 5 )2 (3 - 5)2 3 - 5
= = = =
3+ 5 (3 + 5)(3 - 5) 9-5 2
76
Inverse Trig onometric Fu nctions
ì ü
ï 2´ 1 ï
-1 ï 2 ï + tan -1 1 é -1 æ 2 x ö ù
êQ2 tan x = tan ç
-1
= tan í 2ý
7 2 ÷ ,if - 1 < x < 1ú
ï1 - æ 1 ö ï ë è1 - x ø û
ï ç2÷ ï
î è ø þ
ì 4 1 ü
ï +
tan
4-1
+ tan -1 1
= tan -1 ï 3 7 ïï = tan-1 31
= í ý
3 7 ï1 - 4 ´ 1 ï 17
ïî 3 7 ïþ
1 æ1- x ö
Illustration 16. Prove that tan -1 x = cos -1 ç ÷ , x Î ëé0,1ûù
2 è1 + x ø
ì1 -
( x) ü
2
1 æ1 - x ö 1 -1 ï ï 1
Solution We have, cos -1 ç ÷ = cos í 2 ý
= ´ 2 tan-1 x = tan -1 x.
2 è1 + x ø 2 ï1 +
î ( x) ï 2
þ
é pù
Alter : Putting x = tan q, we have Þ q Î ê0, ú
ë 4û
1 æ1 - x ö 1 æ 1 - tan2 q ö 1 æ é pù ö
RHS = cos -1 ç ÷ = cos -1 ç -1
÷ = cos (cos 2q) = q Q ç 2q Î ê0, ú ÷
2 è1 + x ø 2
2
è 1 + tan q ø 2 è ë 2û ø
= tan -1 x = LHS
1 1 1 p
Illustration 17. Prove that : (i) 4 tan -1 - tan -1 + tan -1 =
5 70 99 4
1 5 2 1 p
(ii) 2 tan -1 + sec -1 + 2 tan -1 =
5 7 8 4
ì 1ü 1 1
Solution (i) 4 tan -1 1 - tan -1 1 + tan -1 1 = 2 í2 tan -1 ý - tan -1 + tan -1
5 70 99 î 5 þ 70 99
é Q2 tan -1 x ù
ì 2 ´1 / 5 ü 1 1 ê ú
= 2 í tan -1 2 ý
- tan -1 + tan -1 ê = tan -1 2x ,if|x|< 1ú
î 1 - (1 / 5) þ 70 99 êë úû
1 - x2
ì 1 1 ü
JPR\COMP.251\Allen(IIT-JEE Wing)\2020–21\Enthusiast\Mathematics\Unit-6
ì 2 ´ 5 /12 ü ïï 70 - 99 ïï
5 ì -1 1 1 ü = tan í -1 -1
- tan . í
= 2 tan -1 - í tan - tan -1 ý 2ý ý
12 î 70 99 þ î1 - (5 /12) þ ï1 + 1 ´ 1 ï
ïî 70 99 ïþ
ì 120 1 ü
120 29 -
120 1 ïï ïï p
= tan-1 í 119 239 ý = tan -1 1 =
-1
= tan - tan -1 = tan-1 - tan-1
119 6931 119 239 ï 120 1 ï 4
1+ ´
îï 119 239 þï
77
JEE-Mathematics
1 5 2 1 ì -1 1 1ü 5 2
(ii) 2 tan -1 + sec -1 + 2 tan -1 = 2 ítan + tan-1 ý + sec -1
5 7 8 î 5 8þ 7
ì 1 1 ü
+ 2
ïï 5 8 ïï
-1 -1
æ5 2 ö
= 2 tan í ý + tan çç ÷÷ - 1 éQsec -1 x = tan -1 x 2 - 1 ù
1 1
ï1 - ´ ï è 7 ø ëê úû
ïî 5 8 ïþ
13 1 1 1
= 2 tan -1 + tan -1 = 2 tan -1 + tan -1
39 7 3 7
ì 2 ´ 1/ 3 ü 1 é 2x ù
= tan -1 í + tan -1 êQ2 tan x = tan 1 - x2 , if|x|< 1ú
-1 -1
2 ý
î1 - (1 / 3) þ 7 ë û
ì 3 1 ü
ïï + ïï
3 1 p
= tan -1 + tan-1 = tan -1 í 4 7 ý = tan-1 1 =
4 7 ï1 - 3 ´ 1 ï 4
îï 4 7 þï
11p
Illustration 18. The equation 2cos–1x + sin–1x = has
6
(A) no solution (B) only one solution (C) two solutions (D) three solutions
11p
Solution Given equation is 2 cos–1 x + sin–1x =
6
11p
Þ cos–1x + ( cos–1 x + sin–1x ) =
6
p 11p
Þ cos–1 x + = Þ cos–1 x = 4p / 3
2 6
which is not possible as cos–1 x Î [ 0 , p ] Ans.(A)
5p 2
Illustration 19. If (tan–1 x)2 + (cot–1 x )2 = , then x is equal to-
8
(A) –1 (B) 0 (C) 1 (D) None of these
5p 2
Solution The given equation can be written as (tan–1 x + cot–1 x )2 – 2 tan–1 x cot–1 x =
8
p
Since tan–1 x + cot–1 x = we have
2
JPR\COMP.251\Allen(IIT-JEE Wing)\2020–21\Enthusiast\Mathematics\Unit-6
2
æ pö -1 æ p -1 ö 5p 2
çè ÷ø - 2 tan x çè - tan x ÷ø =
2 2 8
2
( ) æ pö 3p
2
Þ 2 tan -1 x - 2 ç ÷ tan -1 x - =0
è 2ø 8
p
Þ tan -1 x = - Þ x = –1 Ans. (A)
4
78
Inverse Trig onometric Fu nctions
x -1 x +1 p
Illustration 20. Solve the equation : tan-1 + tan -1 =
x-2 x+2 4
x -1 x +1 p
Solution tan-1 + tan -1 =
x-2 x+2 4
taking tangent on both sides
æ æ x - 1ö æ x + 1öö
tan ç tan -1 ç + tan -1 ç =1
Þ è è x - 2 ÷ø è x + 2 ÷ø ÷ø
æ æ x - 1ö ö æ æ x + 1ö ö
tan ç tan -1 ç + tan ç tan -1 ç
è è x - 2 ø÷ ÷ø è è x + 2 ø÷ ÷ø
Þ =1
æ æ x - 1öö æ æ x + 1ö ö
1 - tan ç tan-1 ç tan ç tan -1 ç
è è x - 2 ø÷ ÷ø è è x + 2 ø÷ ÷ø
x -1 x +1
+ (x - 1)(x + 2) + (x - 2)(x + 1)
x -2 x + 2 =1 =1
Þ Þ
x -1 x +1 x2 - 4 - (x 2 - 1)
1- .
x-2 x+2
1
Þ 2x2 – 4 = – 3 Þ x=±
2
1
Now verify x=
2
æ 1 ö æ 1 ö
-1 +1
ç 2 ÷ ç 2 ÷ æ 2 -1 ö æ 2 +1 ö
= tan -1 ç -1
÷ + tan ç ÷ = tan -1 ç ÷ + tan -1 ç
ç 1
- 2 ÷÷ ç 1
+ 2 ÷÷ è 2 2 - 1ø è 2 2 + 1÷ø
çè ø çè ø
2 2
æ (2 2 + 1)( 2 - 1) + ( 2 2 - 1)( 2 + 1) ö æ 6ö p
= tan-1 ç ÷ = tan -1 ç ÷ = tan-1 (1) =
è (2 2 - 1)( 2 2 + 1) - ( 2 - 1)( 2 + 1) ø è 6ø 4
1
x=–
2
æ 1 ö æ 1 ö
- -1 - +1 æ 2 +1 ö æ 2 -1 ö
ç 2 ÷ ç 2 ÷ -1 -1
= tan -1 ç ÷ = tan ç ÷ø + tan çè
-1
÷ + tan ç
è ÷ {same as above}
ç- 1
- 2 ÷÷ ç- 1
+ 2 ÷÷
2 2 +1 2 2 - 1ø
çè ø çè ø
2 2
p
= tan -1 (1) =
4
1
\ x =± are solutions Ans.
2
Þ – 2x . 2(x + 1) = 2x(2x2 + 2x + 1)
Þ 2x(2x2 + 2x + 1 + 2x + 2) = 0
Þ 2x(2x2 + 4x + 3) = 0
Þ x = 0 or 2x2 + 4x + 3 = 0 {No solution}
Verify x = 0
p p
2tan–1(1) = cos–1(1) Þ =
2 2
\ x = 0 is only the solution Ans.
79
JEE-Mathematics
1.
-1 -1
Find x for which p - 2 sin x = sin 2x 1 - x
2
( )
2.
1 æ 1 ö
If x = 2cos–1 æç ö÷ + sin–1 ç
è 2ø è 2 ÷ø
+ tan–1 ( 3 ) and y = cos æçè 12 sin -1 æ xöö
çè sin 2 ÷ø ÷ø then which of the following
-1 æ 2x ö -1
3. If x < –1 then sin ç ÷ + 2 tan x
è 1+ x2 ø
æ1ö æ1ö æ 4ö
4. 2 tan -1 ç ÷ + tan -1 ç ÷ = tan -1 ç ÷
è5ø è8ø è 7ø
æ3ö æ 17 ö p
5. 2 sin -1 ç ÷ - tan -1 ç ÷=
5
è ø è 31 ø 4
6. é æ1ö ù
sin êsin -1 ç ÷ + cos -1 x ú = 1
ë 5
è ø û
x p
7. cos -1 x + sin -1 =
2 6
Illustration 22. Find the complete solution set of sin–1(sin5) > x2 – 4x.
Solution sin–1(sin5) > x2 – 4x
JPR\COMP.251\Allen(IIT-JEE Wing)\2020–21\Enthusiast\Mathematics\Unit-6
Þ 2 - 9 - 2p < x < 2 + 9 - 2 p
Þ x Î (2 - 9 - 2p, 2 + 9 - 2p ) Ans.
80
Inverse Trig onometric Fu nctions
Illustration 23. Find the complete solution set of [cot–1x]2 – 6[cot–1x] + 9 £ 0, where [.] denotes the greatest
integer function.
Solution [cot–1x]2 – 6[cot–1x] + 9 £ 0
Þ ([cot–1x] – 3)2 £ 0 Þ [cot–1x] = 3
Þ 3 £ cot–1x < 4 Þ x Î (–¥, cot3]
n p
Illustration 24. If cot –1 > , n Î N , then the maximum value of n is -
p 6
(A) 1 (B) 5 (C) 9 (D) None of these
n p
Solution cot –1 >
p 6
æ æ n öö æ pö n
Þ cot ç cot -1 ç ÷ ÷ < cot ç ÷ Þ < 3
è è p ø ø è 6 ø p
æc x-yö -1 æ c 2 - c1 ö -1 æ c 3 - c 2 ö -1 æ c n - c n -1 ö -1 æ 1 ö -1 æ x ö
tan-1 ç 1 ÷ + tan ç ÷ + tan ç ÷ + ... + tan ç ÷ + tan ç ÷ = tan ç ÷
è c1 y + x ø è 1 + c 2 c1 ø è 1 + c 3 c2 ø è 1 + c n c n -1 ø è cn ø èyø
Solution L.H.S.
æc x-yö -1 æ c2 - c1 ö -1 æ c 3 - c 2 ö -1 æ c n - c n -1 ö -1 æ 1 ö
tan-1 ç 1 ÷ + tan ç ÷ + tan ç ÷ + ... + tan ç ÷ + tan ç ÷
è c1 y + x ø è 1 + c2 c1 ø è 1 + c3c2 ø è 1 + c n c n -1 ø è cn ø
æ x 1 ö æ 1 1 ö æ 1 1 ö æ 1 1 ö
ç y-c ÷ ç c -c ÷ ç c -c ÷ ç c -c ÷ æ1ö
= tan ç -1 1 ÷ + tan ç 1
-1 2 ÷ + tan ç 2
-1 3 ÷ + ... + tan ç
-1 n -1 n ÷ + tan -1 ç ÷
ç1 + x . 1 ÷ ç1 + 1 . 1 ÷ ç1+ 1 . 1 ÷ ç1 + 1 . 1 ÷ è cn ø
ç y c1 ÷ ç c1 c 2 ÷ ç c2 c 3 ÷ ç c n -1 c n ÷
è ø è ø è ø è ø
æ -1 x 1ö æ 1 1ö æ 1 1ö
= ç tan - tan -1 ÷ + ç tan-1 - tan -1 ÷ + ç tan -1 tan -1 ÷ +......
è y c1 ø è c1 c2 ø è c2 c3 ø
JPR\COMP.251\Allen(IIT-JEE Wing)\2020–21\Enthusiast\Mathematics\Unit-6
æ 1 1ö æ1ö
+ ç tan -1 - tan-1 ÷ + tan -1 ç ÷
è c n-1 cn ø è cn ø
æxö
= tan -1 ç ÷ = R.H.S.
èyø
81
JEE-Mathematics
-1 n p
4. If cot > n Î N. Then find maximum value of n.
p 6
¥
æ 2 ö
6. Evaluate : å tan-1 çè 1 + (2r + 1)(2r - 1) ÷ø
r =1
æ 2 3ö æ 2 3ö æ 2 3ö
7. The sum of the infinite terms of the series cot -1 ç1 + ÷ + cot -1 ç 2 + ÷ + cot -1 ç 3 + ÷ + ..... is equal to :
è 4 ø è 4 ø è 4ø
(A) tan–1 (1) (B) tan–1 (2) (C) tan–1 (3) (D) tan–1 (4)
¥ æ ö
4n
8. å tan -1 çè n 4 - 2n 2 + 2 ÷ø equals
n =1
82
Inverse Trig onometric Fu nctions
æ 1ö æ 3ö
Illustration 2. If A = 2 tan–1 ( 2 2 - 1) and B = 3 sin -1 ç ÷ + sin-1 ç ÷ , then show A > B.
è 3ø è 5ø
Solution We have, A = 2tan–1 ( 2 2 - 1) = 2tan–1(1.828)
2p
Þ A > 2tan–1 ( 3 ) Þ A> ..... (i)
3
æ 1ö æ 1ö æ 1ö p
also we have, sin -1 ç ÷ < sin -1 ç ÷ Þ sin -1 ç ÷ <
è 3ø è 2ø è 3ø 6
Þ æ 1ö p
3 sin -1 ç ÷ <
è 3ø 2
æ 1ö æ 1 æ 1ö ö
3
æ 23 ö
also, 3 sin-1 ç ÷ = sin -1 ç 3. - 4 ç ÷ ÷ = sin -1 ç ÷ = sin–1(0.852)
è 3ø è 3 è 3ø ø è 27 ø
æ 1ö æ 3ö æ 1ö p
Þ 3 sin -1 ç ÷ < sin -1 ç ÷ Þ 3 sin-1 ç ÷ <
è 3ø è 2 ø è 3ø 3
æ 3ö æ 3ö æ 3ö p
also, sin -1 ç ÷ = sin -1 ( 0.6 ) < sin -1 ç ÷ Þ sin -1 ç ÷ <
è 5ø è 2 ø è 5ø 3
-1 æ 1 ö -1 æ 3 ö 2p
Hence, B = 3 sin ç ÷ + sin ç ÷ < ........ (ii)
è 3ø è 5ø 3
From (i) and (ii), we have A > B.
Illustration 3. Solve for x : If [sin–1cos–1sin–1tan–1x] = 1, where [.] denotes the greatest integer function.
JPR\COMP.251\Allen(IIT-JEE Wing)\2020–21\Enthusiast\Mathematics\Unit-6
p
Þ 1 £ sin–1 . cos–1 . sin–1 . tan–1x £ Þ sin1 £ cos–1 . sin–1 . tan–1x £ 1
2
Þ cos sin1 ³ sin–1 . tan–1x ³ cos1 Þ sin cos sin1 ³ tan–1x ³ sin cos1
Þ tan sin cos sin1 ³ x ³ tan sin cos1
Hence, x Î [tan sin cos 1, tan sin cos sin1] Ans.
83
JEE-Mathematics
1 æ 3sin 2q ö
Illustration 4. If q = tan–1(2 tan2q) - sin -1 ç then find the sum of all possible values of tanq.
2 è 5 + 4cos2q ÷ø
1 æ 3sin 2q ö
Solution q = tan–1(2 tan2q) - sin -1 ç Þ q = tan–1(2 tan2q)
2 è 5 + 4 cos 2q ÷ø
1 æ 6 tan q ö
- sin -1 ç
2 è 9 + tan2 q ÷ø
é æ1 ö ù
ê 2 ç tan q÷ ú
1 è3 ø
Þ q = tan–1(2 tan2q) - sin -1 ê 2
ú
2 ê æ1 ö ú
ê1 + çè 3 tan q÷ø ú
ë û
2 æ1 ö
Þ q = tan–1(2 tan2q) - tan-1 ç tan q÷
2 è3 ø
-1 æ 1 ö
Þ q = tan–1(2 tan2q) - tan çè tan q÷ø .... (i)
3
taking tangent on both sides
6 tan2 q - tan q
Þ tan q =
3 + 2 tan3 q
Illustration 5. Transform sin–1x in other inverse trigonometric functions, where x Î (–1, 1) – {0}
Solution Case -I : 0 < x < 1
æ pö
Let sin–1x = q, q Î ç 0, ÷
è 2ø
1
Now, cos q = 1 - sin2 q Þ q = cos -1 1 - x2
x
q
æ 1 ö
Þ -1 -1 2 -1 2
sin x = cos 1 - x = sec ç ÷ 1– x
ç 2 ÷
è 1- x ø
x
tan q =
1 - x2
x x
Þ q = tan -1 Þ sin -1 x = tan -1
1 - x2 1 - x2
x æ 1 - x2 ö
JPR\COMP.251\Allen(IIT-JEE Wing)\2020–21\Enthusiast\Mathematics\Unit-6
1 æ x ö æ 1 - x2 ö 1
= sec
-1
= tan -1 ç -1
÷÷ = cot ç ÷ = cosec -1 æç ö÷ , 0 < x < 1
2 ç 2 ç x ÷ èxø
1- x è 1-x ø è ø
84
Inverse Trig onometric Fu nctions
Case-II : -1 < x < 0
æ p ö
Let sin -1 x = q q Î ç - , 0 ÷ , Then x = sinq
è 2 ø
Þ cos q = 1 - x2 Þ cos ( -q ) = 1 - x 2
æ 1 ö
Þ q = - cos -1 1 - x2 Þ sin-1 x = - cos-1 1 - x2 = - sec -1 ç ÷
ç 2 ÷
è 1- x ø
x x x
Again, tan q = Þ q = tan -1 Þ sin -1 x = tan -1
2
1- x 1 - x2 1 - x2
x æ 1 - x2 ö é -1 æ 1 ö ù
÷ êQ tan x = -p + cot ç ÷ , x < 0 ú
-1
Þ sin -1 x = tan -1 = -p + cot -1 ç
ç x ÷ë x
è ø û
1 - x2 è ø
1 æ x ö æ 1 - x2 ö 1
= - sec
-1
= tan-1 ç -1
÷÷ = -p + cot ç ÷ = cosec -1 æç ö÷ , - 1 < x < 0
ç ç x ÷ èxø
1 - x2 è 1- x
2
ø è ø
æ 5p ö æ æ 3 öö
Illustration 6. Find the value of cos–1 ç - sin ÷ + sin–1 ç cos çç sin -1 ÷÷
è 6 ø ç
è è 2 ÷ø ÷ø ?
æ 1ö 1 2p p 5p
Solution Given expression = cos–1 ç - ÷ + sin–1 = + =
è 2ø 2 3 6 6
æ 5 -1 ö
Illustration 7. Find the value of cos–1 (cos(2 tan–1 çç ÷ )) ?
÷
è 10 + 2 5 ø
5 -1 p
5 -1 sin
Solution =
4
= 10 = tan p
10 + 2 5 10 + 2 5 p 10
cos
4 10
æ p ö æ pö p
\ Given expression = cos–1 ç cos 2 . ÷ = cos–1 ç cos . ÷ =
è 10 ø è 5ø 5
x æ 4x ö
Illustration 8. Solution of 2 tan–1 + sin–1 ç ÷ = p is
2 è 4 + x2 ø
æ ö ì -1 x
ç 2x ÷ ï-p - 2 tan 2 : x < -2
ç 2 ÷ ï
ç ï -1 x
x ÷ = í 2 tan
2
Solution : -2 £ x £ 2
JPR\COMP.251\Allen(IIT-JEE Wing)\2020–21\Enthusiast\Mathematics\Unit-6
sin–1
çç 1 + ç ö÷ ÷÷
æ
ï 2
è è2ø ø ï -1 x
ï p - 2 tan 2 : x>2
î
x < – 2 will not satisfy
x x
If – 2 £ x £ 2, then 4 tan–1 =p i.e. =1 i.e. x=2
2 2
If x > 2, then p = p
\ x Î [2, ¥)
85
JEE-Mathematics
Illustration 9. Number of solutions of the equation sin –1 |x| = |cos–1 x|, is
Solution sin–1 |x| = |cos–1 x| = cos–1 x
\ 1 solution
y
p
p
2
O x
p
–
2
Illustration 10. The number of real solution of the equation 1 + cos 2x = 2 sin (sinx), –p £ x £ p is
–1
y= |cos x|
y=sin–1(sinx)
–p
–p/2 p/2 p
2
-1 æ 3 x ö b
Illustration 11. The range of the function f(x) = cot çç 2 ÷
÷ is (a, b], then the value of is
è 1+ x ø a
3 x2
Solution 0£ < 3
1+ x2
p æ 3x2 ö p
Þ < cot -1 çç ÷£
2÷
6 è 1+ x ø 2
-1 p
So Sn = tan (n + 1) -
4
p
lim Sn =
n®¥ 4
86
Inverse Trig onometric Fu nctions
Illustration 13. No. of solution of the equation 2(sin-1 x)2 - (sin-1 x) - 6 = 0 is ...
Solution sin–1x = y
2y2 – y – 6 = 0
y = – 1.5, 2
sin–1x = – 1.5, 2
Hence, sin–1 x = –1.5 (rejecting 2)
Only one solution
p
Illustration 14. The range of the function y = is
sin-1 x
-p p
Solution £ sin-1 x £
2 2
1 æ -2 ù é 2 ö
Þ -1
Î ç -¥, ú È ê , ¥ ÷
sin x è p û ëp ø
p
Þ Î ( -¥, -2 ] È [ 2, ¥ )
sin-1 x
Illustration 15. The number of integral roots of the equaiton. sin -1 sin x = cos -1 ( cos x ) in x Î [ 0, 4 p ] are
–1
cos (cos x)
sin -1 sin x
p p
Solution 2
p x
x 2p 3p 4p
p p 3p 2p
2 2
-1 1 2 -1 n - n -1 p
Illustration 16. Prove that : sin + sin -1 + ......+ sin -1 + .........¥ =
2 6 n(n + 1) 2
æ n - n -1 ö
Solution Tn = sin -1 ç ÷
ç n ( n + 1) ÷
JPR\COMP.251\Allen(IIT-JEE Wing)\2020–21\Enthusiast\Mathematics\Unit-6
è ø
æ 1 n n -1 1 ö
Tn = sin -1 ç . - . ÷
ç n n +1 n n + 1 ÷ø
è
æ ö
ç 1 1 1 1 ÷
= sin -1 ç 1- - . 1- ÷
( ) ( n)
2 2
ç n n +1 n+1 ÷
è ø
87
JEE-Mathematics
æ 1 ö -1 æ 1 ö
= sin -1 ç ÷ - sin ç ÷
è nø è n +1 ø
M
¥
1
å
n =1
t n = sin-1 (1) - sin -1
12
æ 1 ö -1 æ 1 ö
sin -1 ç ÷ - sin ç ÷
è 2ø è 3ø
M M
æ1ö æ 1 ö
sin -1 ç ÷ - sin -1 ç ÷
n
è ø è n +1 ø
M M
p
= -0 Hence proved.
2
Illustration 17. Find the integral values of K for which the system of equations
é 2 Kp2
ê arc cos x + (arc sin y) =
ê 4
possesses solutions & find those solutions.
ê 2 p 4
kp2
Solution cos–1 x + (sin–1 y)2 = ... (1)
4
p2
and (sin–1 y)2 . (cos–1 x) = ... (2)
16
Let cos–1 x = t Î [0, p]
æ kp2 ö p2
ç -t + ÷t =
è 4 ø 16
kp 2 p2
t2 - . t+ =0 ...(A)
4 16
D³0
k2 – 4 ³ 0
k ³ 2 or k £ –2.
Also 0 £ cos–1 x £ p
p2
0 £ ( sin-1 y ) £
2
4
kp 2 p2
Þ 0£ £ p+
4 4
4
Þ 0£k£ +1
p
JPR\COMP.251\Allen(IIT-JEE Wing)\2020–21\Enthusiast\Mathematics\Unit-6
88
Inverse Trig onometric Fu nctions
ANSWER KEY
BEGINNER'S BOX-1
15 1 4 3 4
1. (a) , (b) , (c) 2. (a) 1, (b) , (c)
8 10 5 2 5
3. x Î [1, 4) 4. x Î [2np, 2np + p], n Î I
p
5. Domain = R 6. Domain x Î R, Range = éê , p÷ö
ë4 ø
y
p/2 p y
3
2 0 x
p/2
7. –3 –2 –1 0 x 8. 1
9.
-p/2
–1 cos3 cos2 0 cos1 1 x
-p/2
BEGINNER'S BOX-2
æ 3p ö p
1. (a) p – 4, (b) 4p – 10 2. (a) ç ÷ , (b)
è 4ø 6
-p 17 p
3. (a) (4p – 10), (b) 4.
4 20
ì1, if a >0
5. (A) 6. (A) 7. í 8. (1)
î-1, if a<0
BEGINNER'S BOX-3
1
1. (B) 3. (B) 5. 6. (A)
6
1
8. ±
2
BEGINNER'S BOX-4
é 1 ù 1
1. ê , 1ú 2. (A) 3. –p 6.
ë 2 û 5
7. 1
JPR\COMP.251\Allen(IIT-JEE Wing)\2020–21\Enthusiast\Mathematics\Unit-6
BEGINNER'S BOX-5
p 3p
5. (B) 6. 7. (B) 8.
4 4
89
JEE-Mathematics
p
1. Domain of f (x) = sin -1 (2x) + is :
6
æ 1 1ù é 1 3ö é 1 1ù é 1 1ù
(A) ç - , ú (B) ê - , ÷ (C) ê - , ú (D) ê - , ú
è 2 2û ë 4 4ø ë 4 4û ë 4 2û
é æ4ö æ 2 öù
2. The value of tan êcos -1 ç ÷ + tan -1 ç ÷ ú is :
ë è5ø è 3 øû
6 7 16
(A) (B) (C) (D) None of these
17 16 7
-1 æ x2 x3 ö -1 æ 2 x4 x6 ö p
4. If sin çç x - 2 + 4 - ..... ÷÷ + cos çç x - 2 + 4 - .... ÷÷ = 2 for 0 < |x| < 2 , then x equals:
è ø è ø
1 1
(A) (B) 1 (C) - (D) None of these
2 2
é æ 50p ö æ 31p ö ù
5. The value of sec ê sin-1 ç - sin -1
÷ + cos cos ç - ÷ is equal to :
ë è 9 ø è 9 ø úû
10p p
(A) sec (B) sec (C) 1 (D) –1
9 9
æ æ 8p ö æ 8p ö ö
6. cos ç cos -1 cos ç -1
÷ + tan tan ç ÷ ÷ has the value equal to :
è è 7 ø è 7 øø
p
(A) 1 (B) –1 (C) cos (D) 0
7
9. tan(cos–1 x) is equal to
x 1 + x2 1 - x2
(A) (B) (C) (D) 1 - 2x
1 + x2 x x
90
Inverse Trig onometric Fu nctions
é pö
10. If range of the function f(x) = tan–1(3x2 + bx + 3), x Î R is ê 0, ÷ , then square of sum of all possible values of
ë 2ø
b will be :
(A) 0 (B) 18 (C) 72 (D) None of these
æ 1ö 1 æ1 1 ö
11. If x = tan-1 1 - cos -1 ç - ÷ + sin -1 ; y = cos ç cos -1 æç ö÷ ÷ then :
è 2ø 2 è 2 8
è øø
æ4ö æ4ö
(A) x = py (B) y = px (C) tan x = - ç ÷ y (D) tan x = ç ÷ y
è3ø è3ø
13. The number k is such that tan {arc tan(2) + arc tan(20k)} = k. The sum of all possible values of k is :
19 21 1
(A) - (B) - (C) 0 (D)
40 40 5
1
14. Range of the function f (x) = tan–1 [x] + [-x] + 2 -|x| + is :
x2
where [*] is the greatest integer function.
é1 ö ì1 ü ì1 ü é1 ù
(A) ê , ¥ ÷ (B) í ý È [ 2, ¥ ) (C) í , 2ý (D) ê , 2ú
ë4 ø î4 þ î4 þ ë4 û
æ 1ö
15. If tan ( cos -1 x ) = sin ç cot -1 ÷ then x is equal to :
è 2ø
1 2 3 5
(A) (B) (C) (D)
5 5 5 3
é 2 2ù æ 2 2ö
(C) ( -1,1) - ê - , ú (D) [ -1,1] - çç - , ÷
ë 2 2 û è 2 2 ÷ø
1 2
17. If sin -1 + sin-1 = sin -1 x , then the value of x is :
3 3
( 5 - 4 2) ( 5 + 4 2) p
JPR\COMP.251\Allen(IIT-JEE Wing)\2020–21\Enthusiast\Mathematics\Unit-6
¥
æ 1 ö
18. The value of å tan
r=2
-1
ç 2 ÷ , is :
è r - 5r + 7 ø
p p 3p 5p
(A) (B) (C) (D)
4 2 4 4
91
JEE-Mathematics
é 1 1 ù é 1 1ù é 3 3ù
(A) x Î [0, 1] (B) ê - , ú (C) ê - , ú (D) ê - 2 , 2 ú
ë 2 2û ë 2 2û ë û
-1 -1 y
20. If cos x - cos = a , then 4x2 – 4xy cos a + y2 is equal to :
2
(A) –4sin2a (B) 4sin2a (C) 4 (D) 2 sin 2a
x p
22. If tan -1 < , x Î N , then the maximum value of x is :
p 3
(A) 2 (B) 5 (C) 7 (D) None of these
23. Solution set of [sin–1 x] > [cos–1 x], where [ . ] denotes greatest integer function :
é 1 ù
(A) [sin 1, 1] (B) ê , 1ú (C) (cos 1, sin 1) (D) None of these
ë 2 û
24. Find the value of function f(x) = sin–1 cos(sin–1 x) + cos–1sin(cos–1 x) (where |x| £ 1)
p p p
(A) - (B) (C) (D) 0
2 4 2
é d d d ù
tan ê tan-1 + tan -1 + .... + tan-1 ú is equal to :
ë 1 + a1 a 2 1 + a 2a 3 1 + a n -1a n û
(n - 1)d (n - 1)d nd a n - a1
(A) (B) (C) (D)
a1 + a n 1 + a1a n 1 + a1a n a n + a1
JPR\COMP.251\Allen(IIT-JEE Wing)\2020–21\Enthusiast\Mathematics\Unit-6
92
Inverse Trig onometric Fu nctions
æ1ö æ 2ö
1. tan-1 ç ÷ + tan -1 ç ÷ equals to :
è4ø è 9ø
1 æ3ö 1 -1 æ 3 ö 1 æ3ö æ1ö
(A) cos -1 ç ÷ (B) sin ç ÷ (C) tan-1 ç ÷ (D) tan -1 ç ÷
2 è5ø 2 è5ø 2 è5ø è2ø
3x 4x
2. sin -1 + sin -1 = sin-1 x , then roots of the equation are :
5 5
(A) 0 (B) 1 (C) –1 (D) –2
æ1 æ 3 öö æ1 æ 3 öö
3. The value of sin ç cot -1 ç - ÷ ÷ + cos ç cot -1 ç - ÷ ÷ is/are equal to :
è2 è 4 øø è2 è 4 øø
3 2
(A) 1 (B)
10
æ1 æ 3ö ö æ 1 4ö
(C) 2 sin ç cot -1 ç - ÷ + cot -1 (1) ÷ (D) 2 sin ç p - tan-1 (1) - tan -1 ÷
è2 è 4ø ø è 2 3ø
4. For the equation 2x = tan(2tan–1a) + 2tan(tan–1a+tan–1a3), which of the following is/are invalid ?
(A) a2x + 2a = x (B) a2 + 2ax +1= 0 (C) a ¹ 0 (D) a ¹ -1, 1
-1
é ìp 1 æ a öü ìp 1 æ a ö üù
5. The value of ê tan í + sin-1 ç ÷ ý + tan í - sin -1 ç ÷ ý ú , where ( 0 < a < b), is :
ë î4 2 è b øþ î4 2 è b ø þû
b 1 a2 b2 - a 2 b2 - a 2
(A) (B) 1- 2 (C) (D)
2a 2 b 2b 2a
1 - x2 1
(A) y = tan (cos–1x) ; y = (B) y = tan (cot–1 x) ; y =
x x
x
(C) y = sin (arc tan x) ; y = (D) y = cos (arc tan x) ; y = sin (arc cot x)
1 + x2
æ 17 ö p
7. Consider the equation sin -1 ç x 2 - 6x + ÷ + cos -1 k = , then :
è 2 ø 2
JPR\COMP.251\Allen(IIT-JEE Wing)\2020–21\Enthusiast\Mathematics\Unit-6
(A) the largest value of 'k' for which equation has 2 distinct solution is 1
æ 1 ö
(B) the equation must have real root if k Î ç - , 1 ÷
è 2 ø
æ 1ö
(C) the equation must have real root " k Î ç -1, ÷
è 2ø
1
(D) the equation has a unique solution if k = -
2
93
JEE-Mathematics
3p 2
( ) ( ) ( )
2 2 2
8. If cosec -1x + cosec -1 y + cosec -1 z = , then the value of (x – y + z) can be :
4
(A) –1 (B) 1 (C) 3 (D) –3
p p -1 æ 4 7 -1 ö
If f : R ® éê - , ö÷ is a function defined by f ( x ) = tan çè x - x - + tan a÷ø and f is surjective then
2
10.
ë 4 2ø 4
-1 æ 1 - a ö
2
1
(A) cos ç ÷ =2 (B) a + = 2cos ec 2
è 1 + a2 ø a
-1 æ 2a ö -1 æ 2a ö
(C) sin çè 2 ÷ =p-2 (D) tan çè 2 ÷ = 2-p
a + 1ø a - 1ø
ì p p
-1
ïï q - 2np If 2np -
2
£ q £ 2np +
2
sin (sin q) = í
ï(2n + 1)p - q If (2n + 1)p - p p
£ q £ (2n + 1)p +
ïî 2 2
p p p
(ii) When q ¹ pp + , p Î I , we have tan–1 (tan q) = q – np if np – < q < np +
2 2 2
(iii) When q ¹ pp,p Î I , we have cot–1 (cot q) = q – np if np < q < np + p
Read the above passage and answer the following
11. sin–1(sin 100) + cos–1(cos 100) + tan–1(tan 100) + cot–1(cot 100) equals to:
(A) 100 – 31p (B) 100 – 32p (C) 200 – 63p (D) 200 – 32p
JPR\COMP.251\Allen(IIT-JEE Wing)\2020–21\Enthusiast\Mathematics\Unit-6
æ 15p ö
12. If q Î ç ,8 p÷ , then sin–1 (sin q) + cos–1 (cos q) equals to :
è 2 ø
(A) 0 (B) 2q – 16p (C) 16p (D) 2q
æ 15p ö
13. If q Î çè 7p, ÷ , then sin–1 (sin q) + cos–1 (cos q) + tan–1 (tan q) + cot–1 (cot q) equals to:
2 ø
(A) 0 (B) p (C) 7p – q (D) 7p + q
94
Inverse Trig onometric Fu nctions
Comprehension # 2 (Q.14 to 16)
ì p + 2 tan-1 x if x < -1
1 - x 2 ì 2 tan -1 x if x ³ 0 -1 2x ï
cos -1 =í and tan = í 2 tan -1 x if -1 < x < 1
1 + x 2 î -2 tan-1 x if x < 0 1-x 2
ï -p + 2 tan -1 x if
î x >1
-1 1 - x p
2
-1 2x 2x
14. If 0 < x < 1, then number of solutions of 3 sin - 4 cos + 2 tan -1 = is
1+ x 2
1+ x 2
1-x 2
3
(A) 0 (B) 1 (C) 2 (D) 3
2
2x -1 1 - x 2x
15. If 0 < x < 1, then number of solutions of 3 sin -1 - 4 cos + 2 tan -1 = -2p is
1 + x2 1 + x2 1 - x2
(A) 0 (B) 1 (C) 2 (D) 3
2x 1 - x2 2x
16. If 0 < x < 1, then number of solutions of 3 sin -1 2
- 4 cos -1 + 2 tan -1 = -p is
1+ x 1 + x2 1 - x2
(A) 0 (B) 1 (C) 2 (D) 3
JPR\COMP.251\Allen(IIT-JEE Wing)\2020–21\Enthusiast\Mathematics\Unit-6
95
JEE-Mathematics
1. If sin2x + sin2y < 1 for all x, y Î R. If range of sin–1 (tanx . tany) is (a, b) then the value of a3 + b3 :
-1 æ 1 + ab ö æ 1 + bc ö æ 1 + ca ö
2. If cot ç + cot -1 ç + cot -1 ç = kp (a > b > c > 0) find the value of k.
è a - b ÷ø è b - c ÷ø è c - a ÷ø
3. Let cos–1x + cos–1(2x) + cos–1(3x) = p. If x satisfies the cubic ax3 + bx2 + cx – 1 = 0, then find the value of
a + b + c.
æ1- x 2
ö
If a = 2 tan -1 æç + ö÷ & b = sin -1 ç
1 x
4. ÷ . If x > 1 then the value of a + b is kp then find the value of |k|.
è1- x ø è 1 + x2 ø
-1 1 1 2
5. If tan + tan-1 = tan-1 2 The sum of all values of x satisfying the equation.
1 + 2x 1 + 4x x
6. If tan-1 (x - 1) + tan -1 (x) + tan-1 (x + 1) = tan -1 (3x) then the sum of all values of x satisfying the equation.
1 2 2n -1 ap
7. tan-1
+ tan-1 + ........ + tan -1 + ..........¥ If the sum of the series is where a and b are co prime
3 9 1+ 2 2n -1
b
then the a + b is
æ an + b ö
8. cot -1 7 + cot -1 13 + cot -1 21 + cot -1 31 + ....... to n terms. If the sum of series is cot -1 ç where a, b, c
è cn ÷ø
are co prime, then a + b + c is equal to :
y 3
9. Number of positive integral ordered pair (x, y) satifying the equation tan-1 x + cos -1 = sin-1 .
1+ y 2
10
æ pù
10. Let f(x) = cot–1(x2 + 4x + a2 –a) be a function defined R ® ç 0, ú then sum of all real values of a for which
è 2û
f(x) is onto.
JPR\COMP.251\Allen(IIT-JEE Wing)\2020–21\Enthusiast\Mathematics\Unit-6
æ1ö æ1 ö
11. There is a real number k for which there is a triangle whose angles have measure tan-1 ç ÷ , tan -1 ç + k ÷ and
è2ø è2 ø
æ1 ö
tan -1 ç + 2k ÷ . Then value of 16 k
è2 ø
96
Inverse Trig onometric Fu nctions
æ 46 p ö 2p
(B) cos -1 ç cos ÷ (q)
è 7 ø 7
æ æ -33p ö ö 3p
(C) tan -1 ç tan ç ÷÷ (r)
è è 7 øø 7
æ æ -46 p ö ö 4p
(D) cot -1 ç cot ç ÷÷ (s)
è è 7 øø 7
13. [.] and {.} represent the greatest integer and fractional part functions respectively.
Column – I Column – II
–1
(A) Number of solutions of [x] = cos x (p) 3
-1
(B) Number of solutions of sin x = sgn(x) (q) 2
2
(C) Number of solutions of {x} = e x (r) 1
sin-1 x + cos -1 x
(D) Number of solutions of = {x} (s) 0
2
(t) ¥
JPR\COMP.251\Allen(IIT-JEE Wing)\2020–21\Enthusiast\Mathematics\Unit-6
97
JEE-Mathematics
æ -1 5 2ö
1. The value of cot ç cos ec + tan-1 ÷ is equal to : [AIEEE-2008]
è 3 3 ø
6 3 4 5
(1) (2) (3) (4)
17 17 17 17
æ æ öö
2. A value of tan -1 ç sin ç cos -1 2 ÷ ÷ is : [AIEEE ONLINE-2012]
è è 3øø
p p p p
(1) (2) (3) (4)
6 2 4 3
–1 –1
3. A value of x for which sin (cot (1 + x)) = cos (tan x), is : [JEE-MAIN ONLINE-2013]
1 1
(1) (2) 0 (3) 1 (4) -
2 2
-1 -1
4. Let x Î (0,1) . The set of all x such that sin x > cos x, is the interval : [JEE-MAIN ONLINE-2013]
æ 3ö æ1 1 ö æ 1 ö
(1) ç 0, ÷ (2) ç , (3) ç ,1 (4) (0, 1)
è 2 ø è 2 2 ÷ø è 2 ÷ø
æ 43p ö
5. The principal value of tan -1 ç cot ÷ is: [JEE-MAIN ONLINE-2014]
è 4 ø
3p p p 3p
(1) - (2) (3) - (4)
4 4 4 4
æ 2x ö
6. If f(x) = 2 tan -1 x + sin -1 ç , x > 1, then f (5) is equal to : [JEE-MAIN ONLINE-2015]
è 1 + x 2 ÷ø
p æ 65 ö –1
(1) (2) p (3) tan -1 ç (4) 4 tan (5)
2 è 156 ÷ø
–1 –1
7. A value of x satisfying the equation sin [cot (1 + x)] = cos[tan x], is :
[JEE-MAIN ONLINE-2017]
1 1
(1) - (2) –1 (3) 0 (4)
2 2
9. All x satisfying the inequality (cot–1 x)2 – 7 (cot–1 x) + 10 > 0, lie in the interval :
JPR\COMP.251\Allen(IIT-JEE Wing)\2020–21\Enthusiast\Mathematics\Unit-6
[JEE-MAIN ONLINE-2019]
(1) (–¥, cot 5) È (cot 4, cot 2) (2) (cot 5, cot 4)
(3) (cot 2, ¥) (4) (–¥, cot 5) È (cot 2, ¥)
10. {
Considering only the principal values of inverse functions, the set A = x ³ 0 : tan -1 (2x) + tan -1(3x) =
p
4 }
[JEE-MAIN ONLINE-2019]
(1) is an empty set (2) Contains more than two elements
(3) Contains two elements (4) is a singleton
98
Inverse Trig onometric Fu nctions
3 1 p
11. If a = cos -1 æç ö÷ , b = tan-1 æç ö÷ , where 0 < a, b < , then a - b is equal to : [JEE-MAIN ONLINE-2019]
è 5ø è 3ø 2
æ 9 ö æ 9ö æ 9 ö æ 9 ö
(1) sin -1 ç (2) tan -1 ç ÷ (3) cos -1 ç (4) tan -1 ç
è 5 10 ÷ø è 14 ø è 5 10 ÷ø è 5 10 ÷ø
12 3
12. The value of sin -1 æç ö÷ - sin -1 æç ö÷ is equal to: [JEE-MAIN ONLINE-2019]
è 13 ø è 5ø
-1 æ 63 ö -1 æ 33 ö p æ 56 ö p æ 9ö
(1) p - sin çè ÷ø (2) p - cos çè ÷ø (3) - sin -1 ç ÷ (4) - cos -1 ç ÷
65 65 2 è 65 ø 2 è 65 ø
JPR\COMP.251\Allen(IIT-JEE Wing)\2020–21\Enthusiast\Mathematics\Unit-6
99
JEE-Mathematics
1. If x, y, z are in A.P. and tan–1x, tan–1y and tan–1z are also in A.P., then : [JEE (Main)-2013]
(1) x = y = z (2) 2x = 3y = 6z (3) 6x = 3y = 2z (4) 6x = 4y = 3z
–1 –1
2. The number of solutions of the equation, sin x = 2tan x (in principal values) is :
[JEE-MAIN ONLINE-2013]
(1) 3 (2) 1 (3) 2 (4) 4
1 1 1
3. If S = tan -1 æ ö -1 æ ö -1 æ ö , then tan S is equal to:
çè n 2 + n + 1÷ø + tan çè n 2 + 3n + 3 ÷ø + ... + tan çè 1 + (n + 19) (n + 20)÷ø
[JEE-MAIN ONLINE-2013]
n n 20 20
(1) (2) 2 (3) (4) 2
401 + 20n n + 20n + 1 401 + 20n n + 20n + 1
–1 3 –1 3 3 1
4. Statement I : The equation (sin x) + (cos x) – ap = 0 has a solution for all a ³ .
32
2
–1 –1 p æ pö 9p 2
Statement II : For any x Î [–1, 1], sin x + cos x= and 0 £ ç sin -1 x - ÷ £
2 è 4ø 16
[JEE-MAIN ONLINE-2014]
(1) Both statement I and II are true
(2) Statement I is true and statement II is false
(3) Statement I is false and statement II is true
(4) Both statements I and II are false
3x - x 3 3x + x 3 3x - x 3 3x + x 3
(1) (2) (3) (4)
1 - 3x 2 1 - 3x 2 1 + 3x 2 1 + 3x 2
é 1 + x2 + 1 - x2 ù
6. The value of tan-1 ê ú , x < 1 , x ¹ 0 is equal to : [JEE-MAIN ONLINE-2017]
êë 1 + x - 1 - x úû
2 2
2
p p p 1 p 1
(1) + cos -1 x 2 (2) - cos -1 x 2 (3) - cos -1 x 2 (4) + cos -1 x 2
4 4 4 2 4 2
-1 æ 2 ö -1 æ 3 ö pæ 3ö
7. If cos ç ÷ + cos ç ÷ = ç x > ÷ then x is equal to : [JEE-MAIN ONLINE-2019]
è 3x ø è 4x ø 2 è 4ø
JPR\COMP.251\Allen(IIT-JEE Wing)\2020–21\Enthusiast\Mathematics\Unit-6
æ 19 æ n öö
8. The value of cot ç å cot -1 ç 1 + å 2p÷ ÷ is : [JEE-MAIN ONLINE-2019]
çè n = 1 è p= 1 ø ÷ø
22 23 21 19
(1) (2) (3) (4)
23 22 19 21
100
Inverse Trig onometric Fu nctions
–1 y
–1 y 2 2
9. If cos x – cos = a, where – 1 £ x £ 1, –2 £ y £ 2, x £ , then for all x, y, 4x – 4xy cos a + y is
2 2
equal to [JEE-MAIN ONLINE-2019]
2 2 2 2 2 2 2 2
(1) 4 sin a – 2x y (2) 4 cos a + 2x y (3) 4sin a (4) 2sin a
–1 –x
10. Let f(x) = loge(sinx), (0 < x < p) and g(x) = sin (e ), (x ³ 0). If a is a positive real number such that
a = (fog)'(a) and b = (fog)(a), then : [JEE-MAIN ONLINE-2019]
2 2 2
(1) aa – ba – a = 0 (2) aa + ba – a = –2a
2 2
(3) aa + ba + a = 0 (4) aa – ba – a = 1
–1 p p
11. If ƒ'(x) = tan (secx + tanx), - < x < , and ƒ(0) = 0, then ƒ(1) is equal to :
2 2
[JEE-MAIN ONLINE-2020]
p -1 p+2 p +1 1
(1) (2) (3) (4)
4 4 4 4
JPR\COMP.251\Allen(IIT-JEE Wing)\2020–21\Enthusiast\Mathematics\Unit-6
101
JEE-Mathematics
¥
æ 1 ö
1. å tan -1 çè 2i 2 ÷ø = t , then find the value of tan(t). [JEE 2006, 1½ ]
i =1
–1 p –1 –1
2. Let (x, y) be such that sin (ax) + cos (y) + cos (bxy) = [JEE 2007, 6]
2
Match the statements in column-I with statements in column-II and indicate your answer by darkening the
appropriate bubbles in the 4 × 4 matrixgiven in the ORS.
Column-I Column-II
2 2
(A) If a = 1 and b = 0, then (x, y) (p) lies on the circle x + y = 1
2 2
(B) If a = 1 and b = 1, they (x, y) (q) lies on (x – 1) (y – 1) = 0
(C) If a = 1 and b = 2, then (x, y) (r) lies on y = x
2 2
(D) If a = 2 and b = 2, then (x, y) (s) lies on (4x – 1) (y – 1) = 0
–1 –1 2 1/2
3. If 0 < x< 1, then 1 + x 2 [{xcos(cot x) + sin(cot x)} – 1] = [JEE 2008, 3]
x
(A) (B) x (C) x 1 + x 2 (D) 1 + x2
1 + x2
æ 23 æ n
öö
4. The value of cot ç å cot -1 ç 1 + å 2k÷ ÷ is [JEE (Advanced) 2013, 2]
è n= 1 è k= 1 øø
23 25 23 24
(A) (B) (C) (D)
25 23 24 23
5. Match List-I with List-II and select the correct answer using the code given below the lists.
[JEE-Advanced 2013, 3, (–1)]
List-I List-II
1/ 2
æ æ
( ) ( ) ö÷ ö
2
cos tan-1 y + y sin tan -1 y
P. ç 1 ç +y 4÷
takes value 1.
1 5
çè è ( )
ç y 2 ç cot sin -1 y + tan sin-1 y ( ) ÷ø ÷
÷ø 2 3
æp ö
cos ç + x ÷ cos2x then possible value of secx is
JPR\COMP.251\Allen(IIT-JEE Wing)\2020–21\Enthusiast\Mathematics\Unit-6
è 4 ø
S. (
If cot sin
-1
) ( (
1 - x 2 = sin tan -1 x 6 , x ¹ 0 , then possible value of x is 4. )) 1
P Q R S
(A) 4 3 1 2
(B) 4 3 2 1
(C) 3 4 2 1
(D) 3 4 1 2
102
Inverse Trig onometric Fu nctions
6. Let f : [0, 4p] ® [0, p] be defined by f(x) = cos–1 (cos x). The number of points xÎ[0, 4p] satisfying the equation
10 – x
f(x) = is : [JEE-Advanced 2014]
10
-1 æ 6 ö -1 æ 4 ö
7. If a = 3 sin ç ÷ and b = 3 cos ç ÷ where the inverse trigonometric functions take only the principal
è 11 ø è9ø
values, then the correct option(s) is (are) [JEE-Advanced 2015, (4, -2)]
(A) cos b > 0 (B) sin b < 0 (C) cos(a + b) > 0 (D) cos a < 0
æ ¥ ¥ æ ¥ ¥ ö
æ xö ö p
i i
æ xö
8. The number of real solutions of the equation sin -1
ç
è i =1
å
x i +1
- x å çè ÷ø ÷
2 ø 2
= - cos -1
ç
ç å çè - ÷ -
2ø å (-x)i ÷÷
i =1 è i =1 i =1 ø
æ 1 1ö
lying in the interval ç - , ÷ is ______ .
è 2 2ø
é p pù
(Here, the inverse trigonometric functions sin x and cos x assume values in ê - , ú and [0, p], respectively.)
–1 –1
ë 2 2û
[JEE-Advanced 2018]
-1 æ 1 ö
For any positive integer n, define fn :(0, ¥) ® ¡ as fn (x) = å j =1 tan ç
n
9. for all x Î (0, ¥)
è 1 + (x + j)(x + j - 1) ÷ø
–1 æ p pö
(Here, the inverse trigonometric function tan x assumes values in çè - , ÷ø )
2 2
Then, which of the following statement(s) is (are) TRUE? [JEE-Advanced 2018]
5
(A) å j=1 tan 2 (fj (0)) = 55
10
(B) å j=1 (1 + fj¢(0))sec 2 (fj (0) = 10
1
(C) For any fixed positive integer n, lim tan(fn (x)) =
x ®¥ n
(D) For any fixed positive integer n, lim sec 2 (fn (x)) = 1
x ®¥
ì x ü ìï æ æ x öö ïü
10. Let E1 = íx Î ¡ : x ¹ 1 and > 0ý and E 2 = í x Î E1 : sin -1 ç log e ç ÷ ÷ is a real number ý
î x -1 þ ïî è è x - 1ø ø ïþ
–1 é p pù
(Here, the inverse trigonometric function sin x assumes values in ê - , ú )
JPR\COMP.251\Allen(IIT-JEE Wing)\2020–21\Enthusiast\Mathematics\Unit-6
ë 2 2û
æ x ö
Let f : E1 ® ¡ be the function defined by f(x) = log e ç and g:E 2 ® ¡ be the function defined by
è x - 1÷ø
æ æ x öö
g(x) = sin -1 ç log e ç [JEE-Advanced 2018]
è è x - 1÷ø ÷ø
103
JEE-Mathematics
LIST-I LIST-II
æ 1 ù é e ö
(P) The range of f is 1. çè -¥, 1 - e ú È ê e - 1 , ¥÷ø
û ë
(Q) The range of g contains 2. (0, 1)
é 1 1ù
(R) The domain of f contains 3. ê- 2 , 2 úû
ë
æ e ù
5. çè -¥, e - 1 ú
û
æ1 e ù
6. (-¥,0) È ç ,
è 2 e - 1 úû
n
æ k +1 ö æk+2 ö
å sin çè n + 2 p÷ø sin çè n + 2 p÷ø –1
k =0
11. For non-negative integers n, let f(n) = n
. Assuming cos x takes values in [0, p],
æ k +1 ö
å sin2 çè n + 2 p÷ø
k =0
–1 3
(A) sin (7 cos f(5)) = 0 (B) f(4) =
2
1 –1 2
(C) lim f(n) = (D) If a = tan (cos f(6)), then a + 2a – 1 = 0
n ®¥ 2
æ 1 10 æ 7p kp ö æ 7p ( k + 1) p ö ö é p 3p ù
12. The value of sec -1 ç å sec ç + ÷ sec ç + in the interval ê - 4 , 4 ú equals
è 4 k =0 è 12 2 ø è 12 2 ÷ø ÷ø ë û
[JEE-Advanced 2019]
JPR\COMP.251\Allen(IIT-JEE Wing)\2020–21\Enthusiast\Mathematics\Unit-6
104
Inverse Trig onometric Fu nctions
ANSWER KEY
EXERCISE-1
Que. 1 2 3 4 5 6 7 8 9 10
Ans. D D C B D B C A C A
Que. 11 12 13 14 15 16 17 18 19 20
Ans. C D A C D C C C B B
Que. 21 22 23 24 25
Ans. D B A C B
EXERCISE-2
Que. 1 2 3 4 5 6 7 8 9 10
Ans. AD ABC BCD BC BC ABCD ABD ABCD AB ABC
Que. 11 12 13 14 15 16
Ans. C A B B A A
EXERCISE-3
Que. 1 2 3 4 5 6 7 8 9 10
Ans. 0 1 26 1 3 0 5 8 2 1
Que. 11 12 13
Ans. 44 (A)–(q), (B)–(s), (C)–(q), (D)–(r) (A) – (s), (B) – (p), (C)– (s), (D)–(q)
EXERCISE-4(A)
Que. 1 2 3 4 5 6 7 8 9 10
Ans. 1 1 4 3 3 2 1 1 3 4
Que. 11 12
Ans. 1 3
EXERCISE-4(B)
Que. 1 2 3 4 5 6 7 8 9 10
Ans. 1 1 4 3 1 4 1 3 3 4
Que. 11
Ans. 3
EXERCISE-5
Que. 1 2 3 4 5 6 7 8 9
Ans. 1 A-p; B-q; C-p; D-s C B B 3 BCD 2 D
Que. 10 11 12
JPR\COMP.251\Allen(IIT-JEE Wing)\2020–21\Enthusiast\Mathematics\Unit-6
Ans. A A 0
105
106
JEE-Mathematics
IMPORTANT NOTES
JPR\COMP.251\Allen(IIT-JEE Wing)\2020–21\Enthusiast\Mathematics\Unit-6
Fu ncti on
æ 1+ x ö æ 2x ö
1. If f(x) = ln ç ÷ , then f çè 1 + x 2 ÷ø equals-
è 1- x ø
2 3
(A) [f(x)] (B) {f(x)} (C) 2f(x) (D) 3f(x)
3
2. If f : I ® I, f(x) = x + 1, then f is-
(A) one-one but not onto (B) onto but not one-one
(C) one-one onto (C) None of these
3 –1
3. If f(x) = x – 1 and domain of f = {0, 1, 2, 3}, then domain of f is-
(A) {0, 1, 2, 3} (B) {1, 0, –7, –26} (C) {–1, 0, 7, 26} (D) {0, –1, –2, –3}
3x + 2
4. If f(x) = , then :
5x - 3
–1 –1 –1 1
(A) f (x) = f (x) (B) f (x) = – f (x) (C) fof (x) = – x (D) f (x) = – f(x)
19
x y
5. The domain of definition of the function, y (x) given by the equation, 2 + 2 = 2 is :
[JEE 2000 (Scr.), 1]
(A) 0 < x £ 1 (B) 0 £ x £ 1 (C) - ¥ < x £ 0 (D) - ¥ < x < 1
ì- 1 , x < 0
6. Let g (x) = 1 + x - [x ] & f (x) = ïí 0 , x = 0 . Then for all x, f(g(x)) is equal to
ï1 , x >0
î [JEE 2001 (Screening), 1]
(A) x (B) 1 (C) f(x) (D) g(x)
where [ ] denotes the greatest integer function.
1 -1
7. If f : [1 , ¥) ® [2 , ¥) is given by, f(x) = x + , then f (x) equals : [JEE 2001 (Scr),1 ]
x
x+ x2 – 4 x x– x2 – 4
(A) (B) (C) (D) 1- x2 – 4
2 1 + x2 2
8. Let E = {1, 2, 3, 4 } & F = {1, 2}. Then the number of onto functions from E to F is
[JEE 2001 (Screening) 1]
(A) 14 (B) 16 (C) 12 (D) 8
JPR\COMP.251\Allen(IIT-JEE Wing)\2020–21\Enthusiast\Mathematics\Unit Exercise
ax
9. Let f (x) = , x ¹ - 1 . Then for what value of a is f (f (x)) = x ? [JEE 2001 (Screening) 1]
x +1
107
JEE-Mathematics
11. Let function f : R ® R be defined by f(x) = 2x + sinx for x Î R. Then f is : [JEE 2002 (Screening), 3]
(A) one to one and onto (B) one to one but not onto
(C) onto but not one to one (D) neither one to one nor onto
x 2 + x +2
12. Range of the function f(x)= is : [JEE 2003 (Screening), 3]
x 2 + x +1
é 7ù æ 7ù
(A) [1, 2] (B) [1, ¥) (C) ê2, ú (D) ç 1, ú
ë 3û è 3û
x
13. Let f(x) = defined from (0, ¥) ® [0, ¥), then by f(x) is :
1+ x
(A) one-one but not onto (B) one-one and onto
(C) Many one but not onto (D) Many one and onto
2
14. Let f(x) = sinx + cosx, g(x) = x –1. Thus g(f(x)) is invertible for x Î [JEE 2004 (Scr.)]
é p ù é p ù é p pù é pù
(A) ê – ,0ú (B) ê – , pú (C) ê – , ú (D) ê0, ú
ë 2 û ë 2 û ë 4 4û ë 2û
|sin x | + |cos x |
16. The fundamental period of is -
|sin x - cos x | + |sin x + cos x |
p 2p
(A) p (B) (C) 2p (D)
2 3
4x
17. If f(x) = , then f(x) + f(1 – x) is equal to-
4x + 2
(A) 0 (B) –1 (C) 1 (D) 4
ì4, x <–1
18. Let f(x) = í
î –4x, –1 £ x £ 0
If f(x) is an even function in R then the definition of f(x) in (0, ¥) is -
ì4x, 0 < x £ 1 ì4x, 0 < x £ 1
(A) f(x) = í (B) f(x) = í
î4, x >1 î-4, x >1
ì4, 0 < x £ 1
(C) f(x) = í (D) None of these
JPR\COMP.251\Allen(IIT-JEE Wing)\2020–21\Enthusiast\Mathematics\Unit Exercise
î4x, x >1
19. If g(x) is a polynomial satisfying g(x) g(y) = g(x) + g(y) + g(xy) - 2 for all real x and y and g(2) = 5 then g(3)
is equal to -
(A) 10 (B) 24 (C) 21 (D) None of these
x
20. If f(x) is defined on (0, 1) then the domain of definition of f(e ) + f(ln |x|) is -
(A) (–e, –1) (B) (–e, –1) È (1, e) (C) (–¥, –1) È (1, ¥) (D) (–e, e)
108
Fu ncti on
1
21. The graph of function f(x) is as shown, adjacently. Then the graph of is :
f(|x|)
y = f(x)
a b
y
y
ab x
(A) o (B) o a b x
y y
(C) –b –a o (D) –a a x
a b O
22. The graph of f(x) is given then the number of positive solution of f(x) -1 = 1 are :
y
4
x
–4 –2 O 2 4
–4
2+x
23. The range of the function f(x) = , x ¹ 2 is : [AIEEE 2002]
2-x
(A) R (B) R – {–1} (C) R – {1} (D) R – {2}
JPR\COMP.251\Allen(IIT-JEE Wing)\2020–21\Enthusiast\Mathematics\Unit Exercise
109
JEE-Mathematics
7–x
26. The range of the function f(x) = Px–3 is- [AIEEE-2004]
(A) {1, 2, 3, 4, 5} (B) {1, 2, 3, 4, 5, 6}
(C) {1, 2, 3} (D) {1, 2, 3, 4}
27. If ƒ : R ® S defined by ƒ(x) = sinx – 3 cos x + 1 is onto, then the interval of S is- [AIEEE-2004]
(A) [–1, 3] (B) [–1, 1] (C) [0, 1] (D) [0, –1]
3x 2 + 9x + 17
28. If x is real, the maximum value of is- [AIEEE - 2006]
3x 2 + 9x + 7
17 1
(A) 41 (B) 1 (C) (D)
7 4
n
29. If f : R ® R satisfies f(x + y) = f(x) + f(y), for all x, y Î R and f(1) = 7, then å f(r)
r =1
is- [AIEEE 2003]
7n(n + 1) 7n 7(n + 1)
(A) (B) (C) (D) 7n(n + 1)
2 2 2
ì n -1
ïï 2 , when n is odd
30. A function f from the set of natural numbers to integers defined by f(n) = í is- [AIEEE 2003]
ï – n , whenn is even
ïî 2
(A) neither one-one nor onto (B) one-one but not onto
(C) onto but not one-one (D) one-one and onto both
31. A real valued function ƒ(x) satisfies the function equation ƒ(x – y) = ƒ(x)ƒ(y) –ƒ(a – x)ƒ(a + y) where a is a given
constant and ƒ(0) = 1, ƒ(2a – x) is equal to [AIEEE-2005]
(A) ƒ(a) + ƒ(a – x) (B) ƒ(–x) (C) –ƒ(x) (D) ƒ(x)
æ 2x - 1ö
32. The domain of the function f(x) = - log x + 4 ç log 2 ÷ is
è 3+x ø
2
(A) (–4, –3) È (4, ¥) (B) (–¥, –3) È (4, ¥) (C) (–¥, –4) È (3, ¥) (D) None of these
34. Find the values of ‘a’ in the domain of the definition of the function f(a) = 2a 2 - a for which the roots of the
equation, x2 + (a + 1)x + (a – 1) = 0 lie between –2 & 1.
æ 1 ù é1 ö æ 2 ù é1 ö æ 1 1ù
(A) ç - ,0ú È ê ,1÷
è 2 û ë2 ø
(B) çè - ,0ú È ê ,1÷ø (C) çè - , - ú È [ 0,1) (D) None of these
3 û ë3 2 4û
-1 é 2 1ù -1 é 2 1ù
The range of the function f(x) = sin ê x + ú + cos ê x - ú , where [] is the greatest integer function, is :
JPR\COMP.251\Allen(IIT-JEE Wing)\2020–21\Enthusiast\Mathematics\Unit Exercise
36.
ë 2û ë 2û
ìp ü ì pü æ pö
(A) í , pý (B) í0, ý (C) {p} (D) ç 0, ÷
î2 þ î 2þ è 2ø
1
37. Let f(x) = - {x} , the range of f(x) is given by (where {×} represents fractional part of x)
2{- x}
110
Fu ncti on
x 2 + 1 - 3x
38. If f : R ® R, f(x) = then range of f(x) is
x2 + 1 + x
(A) (–¥, –1) (B) (–1, ¥) (C) [2, ¥) (D) (–¥, –3)
39. Let f(x) = ax2 + bx + c, where a, b, c are rational and f : Z ® Z, where Z is the set of integers. Then a + b is
(A) a negative integer (B) an integer
(C) non-integral rational number (D) none of these
sin 2 x + 4 sin x + 5
40. If f(x) = , then range of f(x) is
2sin2 x + 8 sin x + 8
æ1 ö æ5 ö é5 ù é5 ö
(A) çè , ¥ ÷ø (B) çè ,1÷ø (C) ê ,1ú (D) ê , ¥ ÷ø
2 9 ë9 û ë9
42. Let f(x) = x(2 – x), 0 £ x £ 2. If the definition of ‘f’ is extended over the set, R – [0, 2] by f(x – 2) = f(x), then
‘f’ is a :
(A) periodic function of period 1 (B) non-periodic function
1
(C) periodic function of period 2 (D) periodic function of period
2
p
43. Let 0 < a, b, g < are the solutions of the equations cos x = x, cos(sinx) = x and sin(cosx) = x respectively,,
2
then
(A) a < b < g (B) g < a < b (C) b < a < g (D) g < b < a
44. It is given that f(x) is a function defined on R, satisfying f(1) = 1 and for any x Î R, f(x + 5) ³ f(x) + 5 and
f(x + 1) £ f(x) + 1. If g(x) = f(x) + 1 – x, then g(2013) equals
(A) 2014 (B) 2013 (C) 1 (D) 0
45. Let f(x) = ([a]2 – 5[a] + 4)x3 – (6{a}2 – 5{a} + 1)x – (tan x) sgn(x) be an even function " x Î R, then the sum
of all possible values of 'a' is
(where [×] denotes G.I.F and {×} fractional part functionnal part function)
17 53 31 35
(A) (B) (C) (D)
6 6 3 3
é x 2 + 1ù
46. If g : [–2, 2] ® R where f(x) = x 3 + tan x + ê ú is a odd function, then the value of parametric P where [×]
ë P û
JPR\COMP.251\Allen(IIT-JEE Wing)\2020–21\Enthusiast\Mathematics\Unit Exercise
æ 28 ö
(A) (–¥, –3] (B) (–¥, –3 ]È[8, ¥) (C) ç -¥, - ÷ (D) None of these
è 9 ø
111
JEE-Mathematics
1
48. The domain of the function f(x) = 2
is-
[x] - [x] - 6
(A) (–¥, –2) È [4, ¥) (B) (–¥, –2] È [4, ¥) (C) (–¥, –2) È (4, ¥) (D) none of these
8
49. The range of the function y = is
9 - x2
é8 ö æ 8ö é8 ö
(A) (– ¥, ¥) – {± 3} (B) ê , ¥÷ø (C) ç 0, ÷
è 9ø
(D) (– ¥, 0) È ê , ¥÷
ë9 ë9 ø
x 2 - 6x + 10
50. The function f : R ® R, defined as f(x) = is :
3 x - 3 - x2
(A) injective but not surjective (B) surjective but not injective
(C) injective as well as surjective (D) neither injective nor surjective
2
52. The period of sin x is- [AIEEE 2002]
p 3p
(A) (B) p (C) (D) 2p
2 2
54. Which of the following functions from Z to itself are NOT bijections ?
3 2
(A) f(x) = x (B) f(x) = x + 2 (C) f(x) = 2x + 1 (D) f(x) = x + x
x2 - y2 x2 + y2 x4 - y 4
(A) (B) (C) (D) None of these
4 4 (
4 x2 + y 2 )
57. Which of the functions defined below are NOT one-one function(s) ?
2
(A) f(x) = 5(x + 4), (x Î R) (B) g(x) = 2x+(1/x)
JPR\COMP.251\Allen(IIT-JEE Wing)\2020–21\Enthusiast\Mathematics\Unit Exercise
2 –x
(C) h(x) = ln(x +x+1), (x Î R) (D) f(x) = e
58. f (x) =
LM 2x + 3 x£1
values of 'a' for which f(x) is injective is -
MN a x + 1
2
x>1
(A) –3 (B) 3 (C) 0 (D) 1
112
Fu ncti on
Comprehension–1
There are six trigonometric and six inverse trigonometric functions. Similarly there are six hyperbolic function
defined as follows :
ex - e- x ex + e- x
The quantity where x Î R is called hyperbolic sine of x and is written as sinh x, cosh x =
2 2
sinh x 1
called as hyperbolic cosx and hyperbolic tan of x is written as tanh x = . Similarly sech x = ,
cosh x cosh x
1 1
cosech x = , coth x = .
sinh x tanh x
On the basis of above information, answer the following questions
64. Solve the following problems from (a) to (d) on functional equation :
(a) The function f(x) defined on the real numbers has the property that f(f(x)).(1+f(x))=–f(x) for all x in the
domain of f. If the number 3 is in the domain and range of f, compute the value of f(3).
(b) Let f(x) be a function with two properties
(i) for any two real number x and y, f(x + y) = x + f(y) and
(ii) f(0) =2.Find the value of f(100).
1 2 x
(c) f(x) = + lnx(x – 1) (d) f(x) = log 1
2
4x 2 - 1 2 x -1
-1
æ æ7 öö
(f) f(x) = 2
(5x - 6 - x )[{l n{x}}] + (7x - 5 - 2x ) + ç l n ç - x ÷ ÷
2
è è2 øø
f
(g) If f(x) = x2 - 5x + 4 & g(x) = x + 3, then find the domain of g (x)
1 2
1 1
(h) f(x) = + log1 – {x} (x – 3x + 10) + +
[x] 2 - |x| sec(sin x)
113
JEE-Mathematics
66. The function f(x) is defined on the interval [0, 1]. Find the domain of definition of the functions.
(a) f(sinx) (b) f(2x + 3)
p2
(i) y = sin - x2
9
2
68. If g(x) = 2x + 1 and h(x) = 4x + 4x + 7, find a function ¦ such that ¦ o g = h.
69. Find two distinct linear functions which map the interval [–1, 1] onto [0, 2].
70. Let ¦ : {x, y, z} ® {a, b, c} be a one–one function. It is known that only one of the following statement is
true :
(i) ¦(x) ¹ b (ii) ¦(y) = b (iii) ¦(z) ¹ a
Find the function ¦ (as ordered pair).
x+a
73. Consider a function ¦ : x ® ; x Î R – {1} where a is a real constant. If ¦ is not a constant function,
x -1
then find the following :
–1 æ 1 ö æ æ æ 1ö ö ö
(a) the range of ¦ (b) ¦ , if it exist (c) f ç – f ç f ç f ç ÷÷÷
è f ( f x )ø
( ) ÷ è è è xøøø
1
cos x -
1- 5 x
2
(a) f ( x ) = (b) f ( x ) = log x (cos 2px) (c) f ( x ) =
7– x - 7 6 + 35x - 6x 2
JPR\COMP.251\Allen(IIT-JEE Wing)\2020–21\Enthusiast\Mathematics\Unit Exercise
2 –1
75. Let f(x) = (1 + x ) sgn x. Find its inverse and sketch the graphs of f(x) and f (x).
æ 1ö
76. If f(x) = max ç x, ÷ for x > 0 where max (a, b) denotes the greater of the two real numbers a and b. Define
è xø
æ1ö
the function g(x) = f(x) f ç ÷ and plot its graph.
èxø
114
Fu ncti on
(i) f(x) = log1/ 3 log 4 ([x]2 - 5) , where [.] denotes greatest integer function.
1
(ii) f(x) =
[| x - 1|] + [|12 - x |] - 11 , where [x] denotes the greatest integer not greater than x.
x 2 + 2x - 3
log(0.5 + x)
(iii) f(x) = (x + 0.5) 4x 2 - 4x - 3
5 -1 2 (7x + 1)!
(iv) f(x) = - 3 sin x + , where [.] denotes greatest integer function.
é x - 1ù x +1
êë 2 úû
4 2
(v) 3 y + 2x = 24 x -1
78. If a function is defined as f(x) = log h(x) g(x) , where g(x) = |sinx| + sinx, h(x) = sinx + cosx, 0 £ x £ p. Then
find the domain of f(x).
x
(i) f(x) = cos -1 log [x] , where [.] denotes the greatest integer function.
x
(ii) f(x) = log1/ 2 log 2 [x 2 + 4x + 5] , where [.] denotes the greatest integer function
-1
é æ x2 ö ù
(iii) f(x) = sin êlog 2 ç 2 ÷ ú , where [.] denotes greatest integer function.
êë è ø úû
(iv) f(x) = log[x – 1] sinx, where [ ] denotes greatest integer function.
1
(v) f(x) = tan -1 ( [x] + [- x]) + 2 - | x | + , (where [ ] denotes greatest integer function)
x2
80. Find the integral solutions to the equation [x][y] = x + y. Show that all the non-integral solutions lie on exactly
two lines. determine these lines. Here [.] denotes greatest integer function.
81. Let f(x) = Ax2 + Bx + C, where A, B, C are real numbers. Prove that if f(x) is an integer whenever x is integer,
then the numbers 2A, A + B and C are all integers. Conversely, prove that if the numbers 2A,
A + B and C are all integer then f(x) is an integer wherever x is an integer.
é x , x<1 ù
ê 2 ú –1
82. If f(x) = ê x , 1 £ x £ 4 ú , then find f (x).
ê ú
ë8 x , x>4 û
84. Let f(x) = log2log3log4log5(sinx + a2). Find the set of values of a for which domain of f(x) is R.
85. Find period of f(x) = sin(kp{x}), where k is integer and {x} = x – [x], where [x] and {x} denotes the integral
part and fractional part of x resepectively.
-1 æ x - k ö
2
æ pù
86. Let g : R ® çè 0, ú is defined by g(x) = cos ç ÷ . Then find the possible values of 'k' for which g is
3û è 1 + x2 ø
surjective.
115
JEE-Mathematics
2
87. If f0(x) = x and fn+1(x) = f0(fn(x)) for n = 0, 1, 2, ....., find a formula for f n(x).
88. Prove that if the function f(x) = sinx + cospx is periodic, then p is a rational number.
89. Given x = {1, 2, 3, 4}, find all one-one, onto mappings, f : X ® X such that,f (1) = 1 , f (2) ¹ 2 and
f (4) ¹ 4 . [REE 2000, 3]
ì x, x Î Q
90. Let f(x) = í
î1 - x, x ÏQ
Statement-I– f(f(x)) = x.
Because
Statement-II– f(x) is neither odd nor even.
(A) A (B) B (C) C (D) D
94. Let function f : R ® R is such that f(x) f(y) – f(xy) = x + y for all x, y Î R
Statement-I– ¦(x) is a Bijective function.
Because
Statement-II– ¦(x) is a linear function.
(A) A (B) B (C) C (D) D
JPR\COMP.251\Allen(IIT-JEE Wing)\2020–21\Enthusiast\Mathematics\Unit Exercise
*******
116
Fu ncti on
ANSWER KEY
YEAR LONG REVISION EXERCISE
SECTION - 1
Que. 1 2 3 4 5 6 7 8 9 10
Ans. C A C A D B A A D D
Que. 11 12 13 14 15 16 17 18 19 20
Ans. A D A C A B C A A A
Que. 21 22 23 24 25 26 27 28 29 30
Ans. C B B C A C A A A D
Que. 31 32 33 34 35 36 37 38 39 40
Ans. C A D A C C D B B C
Que. 41 42 43 44 45 46 47 48 49 50
Ans. D C B C D C A A D D
Que. 51 52
Ans. B B
SECTION - 2
53 54 55 56 57 58 59
BC ACD BD AC ABC AB AD
SECTION - 3
Que. 60 61 62
Ans. C C A
SECTION - 4
4 2 3
63. p (r + 3r + 1) 64. (a) - (b) 102
3 4
é 5p -3p ù é p pù é 3p 5p ù æ 1ö
65. (a) ê - , È ê- , ú È ê , (b) ç - 4, - ÷ È (2, ¥)
ë 4 4 úû ë 4 4û ë 4 4 úû è 2ø
é1 - 5 ö é1 + 5 ö
(c) (–1 < x < –1/2) U (x > 1) (d) ê 2 , 0 ÷÷ È ê 2 , ¥ ÷÷
êë ø êë ø
(e) 2Kp < x < (2K + 1)p but x ¹ 1 where K is non-negative integer
æ 5ö
(f) (1, 2) È ç 2, ÷ (g) (–¥, –3) È (–3, 1] È [4, ¥)
è 2ø
(h) (–2, –1) È (–1, 0) È (1, 2)
JPR\COMP.251\Allen(IIT-JEE Wing)\2020–21\Enthusiast\Mathematics\Unit Exercise
117
JEE-Mathematics
2
68. x + 6 69. f(x) = 1 + x or 1 – x
y = f(n)
é1
y = f–1(n) ê 2 if 0 < x £ 1
75. 76. g(x) = ê x
ê x2 if x > 1
y = f–1(n) ë
y = f(n)
77. (i) x Î [–3, 2) È [3, 4) (ii) R–{(0, 1) È{1, 2.....12} È (12, 13)}
æ 1 ö æ 3 ö ì1ü ì 1 1 2 3 4 5 6ü
(iii) x Î ç - ,1÷ È ç , ¥÷ - í ý (iv) Domain í - ,0, , , , , , ý
è 2 ø è 2 ø î 2þ î 7 7 7 7 7 7 7þ
æ - 3 -1 - 3 +1 ö æ 3 -1 3 +1 ö é p pö
(v) ç ,
2 ÷ Èç , ÷ 78. ê , ÷ø
è 2 ø è 2 2 ø ë6 2
ìp ü
79. (i) Domain : [2, ¥), Range : í ý (ii) Domain : x Î (-2 - 2, -3] È [ -1, -2 + 2) ; Range : {0}
î 2þ
ì p pü é¥ ù
(iii) x Î (- 8, -1] È [1, 8) and R : í - ,0, ý (iv) Domain x Î [3, p ) È ê U (2np,(2n + 1)p)ú , rangeÎ(–¥, 0]
î 2 2þ ë n =1 û
ì1 ü
(v) Domain : {–2, –1, 1, 2} ; Range : í , 2ý
î4 þ
x x<1
x 1 £ x £ 16
80. (0, 0), (2, 2), x + y = 6, x + y = 0 82. f - 1 (x)
x2
x > 16
64
83. [2, ¥) 84. a Î (-¥, - 626) È ( 626, ¥)
æ -1 ù
85. p =1 86. èç -1, ú
2û
JPR\COMP.251\Allen(IIT-JEE Wing)\2020–21\Enthusiast\Mathematics\Unit Exercise
( n +1)
2
87. f(x) = x
89. {(1, 1) , (2, 3) , (3, 4) , (4, 2)} ; {(1, 1) , (2, 4) , (3, 2) , (4, 3)} and {(1, 1) , (2, 4) , (3, 3) , (4, 2)}
Que. 90 91 92 93 94 95
Ans. B C C B A D
*******
118
Inverse Trig onometric Fu nctions
1. If we consider only the principle values of the inverse trigonometric functions then the value of
æ 1 4 ö
tan ç cos -1 - sin -1 ÷ is [JEE 1994]
è 5 2 17 ø
29 29 3 3
(A) (B) (C) (D)
3 3 29 29
æ 1 - x2 ö
2. The solution set of the equation sin -1 1 - x 2 + cos -1 x = cot -1 ç ÷ - sin -1 x is -
ç x ÷
è ø
(A) [–1, 1]–{0} (B) (0, 1] U {–1} (C) [–1, 0) U {1} (D) [–1, 1]
3. If [sin–1x] + [cos–1x] = 0, where ‘x’ is a non negative real number and [.] denotes the greatest integer function,
then complete set of values of x is -
(A) (cos1, 1) (B) (–1, cos1) (C) (sin1, 1) (D) (cos1, sin1)
–1 é æ x öù
4. The domain of sin êlog3 ç ÷ú [AIEEE - 2002]
ë è 3 øû
(A) [1, 9] (B) [-1, 9] (C) [-9, 1] (D) [-9, -1]
sin -1 (x - 3)
5. The domain of the function f (x) = is [AIEEE - 2004]
9 - x2
(A) [1, 2) (B) [2, 3) (C) [1, 2] (D) [2, 3]
æ -p p ö é - x2 -1 æ x ö ù
6. The largest interval lying in ç , ÷ for which the function ê f(x) = 4 + cos çè - 1÷ø + log (cos x) ú is defined,
è 2 2ø ë 2 û
is [AIEEE - 2007]
æ p pö é p pö é pö
(A) ç - , ÷ (B) ê - , ÷ (C) ê0, ÷ (D) [0, p]
è 2 2ø ë 4 2ø ë 2ø
æ x 2 y2 ö æ x y ö
7. sin -1 ç + ÷ + cos -1 ç + - 2÷ equals to:
è 4 9ø è2 2 3 2 ø
p p 3p
(A) (B) p (C) (D)
2 2 2
JPR\COMP.251\Allen(IIT-JEE Wing)\2020–21\Enthusiast\Mathematics\ITF\Unit Exercise
1 3
8. If a = +i and z = x + iy, then sin–1|z|2 + cos–1 (az + az – 2) equals to::
4 4
p p 3p
(A) 0 (B) (C) (D)
4 2 2
-1 2x 1 - x2 2x
9. If x > 1, then number of solutions of 3 sin - 4 cos -1 + 2 tan -1 = -p is
1+ x 2
1 + x2 1 - x2
(A) 0 (B) 1 (C) 2 (D) 3
119
JEE-Mathematics
æ pù æ x2 - k ö
10. Let g : R ® ç 0, ú is defined by g(x) = cos ç 1 + x 2 ÷ . Then the possible values of 'k' for which g is surjective
–1
è 3û è ø
function, is
ì1 ü æ 1ù ì 1ü é 1 ö
(A) í ý (B) ç -1, - ú (C) í- ý (D) ê - ,1 ÷
î2þ è 2û î 2þ ë 2 ø
11. The range of values of p for which the equation sin cos–1 (cos(tan–1x)) = p has a solution is:
æ 1 1 ù é 1 ö
(A) ç - , ú (B) [0, 1) (C) ê , 1÷ (D) (– 1, 1)
è 2 2û ë 2 ø
é 3 x2 - 7 x + 8 ù
12. The domain of definition of the function, f (x) = arc cos ê ú where [ *] denotes the greatest integer
ë 1 + x2 û
function, is :
(A) (1, 6) (B) [0, 6) (C) [0, 1] (D) (- 2, 5]
æ 2ö æ 5ö æ 10 ö
13. The value of expression : tan-1 çç -1
÷÷ + sin çç
-1
÷÷ - cos çç ÷÷ is
è 2 ø è 5 ø è 10 ø
æ1+ 2 ö -1
æ 2 + 1ö æ
-1 1 + 2ö æ1- 2 ö
(A) cot -1 çç ÷÷ (B) cot çç ÷÷ (C) -p + cot çç ÷÷ (D) p - cot -1 çç ÷÷
è1- 2 ø è 2 -1ø è1- 2 ø è1+ 2 ø
14. Value of k for which the point (a, sin–1a)(a > 0) does not lies inside the triangle formed by x + y = k with co-ordinate
axes may be -
(A) 5 (B) 6 (C) 7 (D) 2
æ 2
ö
15. Solution set of the inequality sin -1 ç sin 2x + 3 ÷ £ p - 5 is -
2
è x + 1 ø 2
1
16. Consider two geometric progressions a1,a2,a3.......an & b1, b2, b3,.....bn with a r = = 2r -1 and another sequence
br
n
t1,t2,t3.......tn such that tr = cot–1 (2ar + br) then lim å t r is -
n ®¥
r =1
p p
(A) 0 (B) (C) tan–1 2 (D)
4 2
18. The trigonometric equation sin–1 x = 2 sin–1a, has a solution for- [AIEEE-2003]
1 1 1 1
(A) a £ (B) < a < (C) all real values of a (D) a <
2 2 2 2
-1 æ x ö -1 æ 5 ö p
19. If sin ç ÷ + cos ec ç ÷ = then a value of x is- [AIEEE-2007]
5
è ø 4
è ø 2
(A) 1 (B) 3 (C) 4 (D) 5
120
Inverse Trig onometric Fu nctions
y
20. If cos–1x – cos–1 = a, then 4x2 – 4xy cos a + y2 is equal to - [AIEEE-2005]
2
(A) 2 sin 2a (B) 4 (C) 4 sin2a (D) –4 sin2a
-1 æ 2p ö
21. The principal value of sin ç sin ÷ is [JEE 1986, (2)]
è 3 ø
2p 2p 4p
(A) - (B) (C) (D) None of these
3 3 3
ì 4 2ü a
22. If numerical value of tan ícos -1 + tan -1 ý is , then :
î 5 3þ b
(A) a + b = 23 (B) a – b = 11 (C) 3b = a + 1 (D) 2a = 3b
é1 æ æ 14 p ö ö ù
23. The value of cos ê cos -1 ç cos ç - ÷ ÷ ú is/are -
ë 2 è è 5 ø øû
æ 7p ö æ p ö æ 2p ö æ 3p ö
(A) cos ç - ÷ (B) sin ç ÷ (C) cos ç ÷ (D) - cos ç ÷
è 5 ø è 10 ø è 5 ø è 5 ø
x2 - 1 2x 2p
24. cos -1 2
+ tan -1 2
= then x =
x +1 x -1 3
(A) x = 2 – 3 (B) 3 (C) 2 + 3 (D) None of these
-1 p
25. Domain of log - tan 2x - lies in interval
6
æ 1 ö æ 1 ö
(C) çè -¥, - (D) çè -¥, -
–
(A) R (B) (–¥,–1) ÷ ÷
2 3ø 3 2ø
é 1ù
26. For x Î ê –1, - ú
ë 2û
(A) a = p, b = –1 (B) a = 0, b = 1 (C) a = –p, b = –1 (D) a = –p, b = 1
JPR\COMP.251\Allen(IIT-JEE Wing)\2020–21\Enthusiast\Mathematics\ITF\Unit Exercise
é 1 1ù
27. For Î ê - , ú
ë 2 2û
(A) a = p, b = –1 (B) a = 0, b = 1 (C) a = –p (D) a = –p, b = 1
é1 ù
28. For x Î ê ,1ú
ë2 û
(A) a = p, b = –1 (B) a = 0, b = 1 (C) a = –p, b = –1 (D) a = –p, b = 1
121
JEE-Mathematics
ì 3p p
ï p+q , - 2 < q < -2
ï
-1 ï p p
tan (tan q) = í q , - <q<
ï 2 2 ,
ï p 3p
ï -p + q , 2
<q<
2
î
ì 3p p
ï -p - q , - 2 £ q < - 2
ï
-1 ï p p
sin (sin q) = í q , - £q£
ï 2 2
ï p 3p
ï p-q , 2
<q£
2
î
ì -q , -p £ q < 0
-1 ï
cos (cos q) = í q , 0£q£p
ï 2p - q , p < q £ 2p
î
1 - x2 1 - x2
JPR\COMP.251\Allen(IIT-JEE Wing)\2020–21\Enthusiast\Mathematics\ITF\Unit Exercise
1 - x2 1 - x2
(C) –tan–1 if 0 < x < 1 (D) p + tan–1 if –1 < x < 0
x x
122
Inverse Trig onometric Fu nctions
æx 1 ö
32. Express f(x) = arc cos x + arc cos ç + 3 - 3x 2 ÷ in simplest form and hence find the values of
è 2 2 ø
1
(a) f æç 2 ö÷ (b) f æç ö÷
è3ø è3ø
x y x 2 2xy y2
33. If cos-1 + cos -1 = a then prove that 2 - cos a + 2 = sin 2 a .
a b a ab b
1- x 1+ x
34. Prove that : cos -1 x = 2 sin -1 = 2cos -1
2 2
2 1 12
35. Prove that : tan-1 = tan -1
3 2 5
1 1 p 1
36. Prove that : 3 tan-1 + tan -1 = - tan -1
4 20 4 1985
ép m
ê4 > -1
æ m ö æ n - m ö n
(b) tan-1 ç ÷ + tan -1 ç ÷ = ê -3p
ènø èn+ mø ê m
< -1
êë 4 n
38. If arc sin x + arc siny + arc sinz = p then prove that : (x , y , z > 0 )
(a) x 1 - x 2 + y 1 - y 2 + z 1 - z 2 = 2xyz
(b) x 4 + y 4 + z 4 + 4x 2 y 2 z 2 = 2 ( x 2 y 2 + y 2 z 2 + z 2 x 2 )
b3 é1 b ù a3 é1 aù
39. Express cosec 2 ê tan -1 ú + sec 2 ê tan -1 ú as an integral polynomial in a & b.
2 ë2 aû 2 ë 2 bû
(Read the symbols [ * ] and { * } as greatest integers and fractional part functions respectively)
(a) 1 1
f(x) = + 2arc sin x +
x x-2
-1 éx ù 1
(b) ecos x
+ cot -1 ê - 1ú + ln{x}
ë2 û 2
2
41. If cos q = , where q Î [31p, 32p], then find the value of q.
3
123
JEE-Mathematics
1
42. If x < 0, then prove that cot –1x = p + tan–1
x
x 1 - x2
(i) cos -1 1 - x 2 (ii) tan-1 (iii) cot -1
1 - x2 x
æ1+ xö
45. If a = 2 tan–1 çè ÷
1 - xø
æ 1 - x2 ö
b = sin–1 çè 1 + x 2 ÷ø for 0 < x < 1,
æ pö
cos -1 sin ç x + ÷ æ 7p ö
then the value of f çè - ÷ø is :
è 3ø
46. If f(x) = e
4
47. Solve {cos–1x} + [tan–1x] = 0 for real values of x. Where {.} and [.] are fractional part and greatest integer
functions respectively.
48. Find the set of all real values of x satisfying the inequality sec –1x > tan–1x.
49. -1
ì cos -1 xy - 1 - x 2 1 - y 2
ï
Prove that cos x + cos y = í -1
( ) if x + y ³ 0
.
(
ï 2p - cos -1 xy - 1 - x 2 1 - y 2
î ) if x + y < 0
2x 3x - x 3
51. Express tan–1x + tan–1 –1
in terms of tan 1 - 3x 2 .
1 - x2
JPR\COMP.251\Allen(IIT-JEE Wing)\2020–21\Enthusiast\Mathematics\ITF\Unit Exercise
xy + 1
52. If y – x < 0, then prove that cot–1x – cot–1y = – p + cot–1 .
y-x
-1 x x -1 1
53. Find the solution of sin - sin -1 = sin -1 is equal to
1+ x x +1 1+ x
124
Inverse Trig onometric Fu nctions
(Read the symbols [ * ] and { * } as greatest integers and fractional part functions respectively)
1 - sin x
54. f(x) = + cos -1 (1 - {x} )
log5 (1 - 4x2 )
æ 3 - 2x ö
55. f(x) = 3 - x + cos -1 ç + log6 ( 2|x|-3 ) + sin -1 (log2 x )
è 5 ÷ø
æ 3 ö
56. f(x) = log10 (1 - log7 (x 2 - 5x + 13)) + cos -1 ç 9 px ÷
è 2 + sin 2 ø
æ 2 sin x +1 ö
cos -1 ç
57. f(x) = sin(cos x) + ln(-2 cos2 x + 3 cos x + 1) + e è 2 2 sin x ÷ø
1 - a2 1 - b2
59. 2 tan -1 x = cos -1 - cos -1 a > 0, b > 0
1 + a2 1 + b2
2
60. arc cot x – 5 arc cot x + 6 > 0
2 2
62. 4 arc tan x – 8 arc tan x + 3 < 0 & 4 arc cot x – arc cot x – 3 ³ 0
64. ( 2
) æ
è
æx
è 2
öö
sin sin -1 (log 1 x) + 2 cos ç sin -1 ç - 1÷ ÷ = 0
øø
æ -3 p 5 p ö
65. |y|= cos x and y = cot -1 (cot x) in ç ,
è 2 2 ÷ø
–1 –1 x2 + 1
66. Prove that cos tan sin cot x= [JEE 2002 (Mains), 5]
JPR\COMP.251\Allen(IIT-JEE Wing)\2020–21\Enthusiast\Mathematics\ITF\Unit Exercise
x2 + 2
1
67. Find the value of : cos(2cos -1 x + sin-1 x) at x = , where 0 £ cos -1 x £ p and -p / 2 £ sin -1 x £ p / 2 .
5
[JEE 1981, (2)]
–1 –1
éæ 1 ö ù
68. If a = sin (sin (cos (cot 3))) t hen value of êçè ÷ø ú is equal to (where [ . ] represent greatest integer function)
ë a û
–1 æxö
69. The number of integers in the domain of f(x) = sin ç ÷ + tan x is equal to –
è4ø
125
JEE-Mathematics
1 1 1 1 ù
70. Statement-I– Range of cos æç sec -1 + cos ec -1 + tan -1 x ö÷ is éê - , ú
è x x ø ë 2 2û
Because
é p 3p ù
Statement-II– Range of sin–1 x + tan–1 x + cos–1 x is ê , ú .
ë4 4 û
(A) A (B) B (C) C (D) D
2n n n n
Because
9 9
74. Statement-I– cos ec -1 æç cosec ö÷ = p - .
è 5ø 5
Because
é p 3p ù
Statement-II– cosec–1(cosecx) = p – x ; " x Î ê , - {p}
ë 2 2 úû
(A) A (B) B (C) C (D) D
126
Inverse Trig onometric Fu nctions
ì æ 1ö p ü
76. The numerical value of tan í 2 tan-1 ç ÷ - ý is equal to _______________ [JEE 1984 (2)]
î è 5ø 4 þ
æ 1ö æ 3ö
77. The greater of the two angles A = 2 tan-1 (2 2 - 1) and B = 3 sin-1 ç ÷ + sin -1 ç ÷ is __________
è 3ø è 5ø
[JEE 1989 (2)]
é 1 æ -1 ö ù
78. tan êcos -1 + tan -1 ç ÷ú = ..................
ë 2 è 3 øû
79. cos (tan–1 2) = ...................
æ 3 3ö
80. tan ç sin-1 + cot -1 ÷ = ...................
è 5 2ø
é æ - 3 ö pù
81. cos ê cos -1 çç ÷÷ + ú = ...................
ëê è 2 ø 6 ûú
82. 3 11
sin -1 + cos -1 + cot -1 3 = ..............
73 146
æ 1 ö -1 æ 1 ö -1 æ 1 ö
æ
-1 1 + 2ö
83. tan-1 ç ÷ + sin ç ÷ - cos ç ÷ - cot çç ÷÷ = .................
è 2ø è 5ø è 10 ø è1- 2 ø
ép æ - 3 öù
84. sin ê - sin-1 ç = .................
ç 2 ÷÷ ú
ëê 2 è ø ûú
æ 1 1 ö æ 10 -1 ö
85. - ç cos -1 + cos -1 -1
÷ - cos çç ÷÷ + 4 cot 1 = .................
-1
è 3 6ø è 3 2 ø
é 3 sin 2a ù -1 é tan a ù p p
86. tan -1 ê ú + tan ê 4 ú , where - < a < = ...................
ë 5 + 3cos 2a û ë û 2 2
JPR\COMP.251\Allen(IIT-JEE Wing)\2020–21\Enthusiast\Mathematics\ITF\Unit Exercise
87. The number of roots of the equation sin x = cos -1 (cos x) is ...................
*******
127
JEE-Mathematics
ANSWER KEY
YEAR LONG REVISION EXERCISE
SECTION - 1
Que. 1 2 3 4 5 6 7 8 9 10
Ans. D C D A B C D D A C
Que. 11 12 13 14 15 16 17 18 19 20
Ans. B A C D B B B A B C
Que. 21
Ans. D
SECTION - 2
22 23 24 25
ABC BCD AB ACD
SECTION - 3
Que. 26 27 28 29 30 31
Ans. C B A D C D
SECTION - 4
p æ 1ö p
32. (a) ; (b) 2cos –1 çè ÷ø – 39. (a2 + b2)(a + b)
3 3 3
40. (a) f (not defined for any real x) (b) ( –1, 1) –{0}
ìï - x 2 - 1 if x £ -1
43. cot(cosec–1 x) = í
ïî x 2 - 1 if x ³ 1
ìï - cos -1 1 - x 2 if -1 £ x < 0 x
44. (i) sin -1 x = í (ii) sin -1 x = tan-1 , for all x Î (–1, 1)
ïî cos -1 1 - x 2 if 0£ x £1 1 - x2
ì -1 1 - x 2
ïcot - p if -1 £ x < 0
ï x
(iii) sin -1 x = í
ï -1 1 - x2
ïî cot if 0 < x £ 1
x
JPR\COMP.251\Allen(IIT-JEE Wing)\2020–21\Enthusiast\Mathematics\ITF\Unit Exercise
ì -1 1
ï p - 2sin x if < x £1
ï 2
ï 1 1
49. sin -1 2x 1 - x 2 = í 2sin -1 x if - £x£
ï 2 2
ï -1 1
ï -p - 2 sin if - 1 £ x < -
î 2
128
Inverse Trig onometric Fu nctions
ì -1 3x - x
2
1
ï -p + tan 2
when - 1 < x < -
ï 1 - 3x 3
-1 -1 2x ï 3x - x 2
1
50. tan x + tan = í tan -1 when |x|> 1 or |x |<
1 - x2 ï 1 - 3x 2 3
ï -1 3x - x
2
1
ï p + tan when < x <1
î 1 - 3x 2
3
ì -1 3x - x
2
1
ï -p + tan 2
when - 1 < x < -
ï 1 - 3x 3
ï
2x = tan -1 3x - x
2
1
-1
51. tan x + tan
-1
í when |x|> 1 or |x |<
1-x 2
ï 1 - 3x 2
3
ï -1 3x - x 2
1
ï p + tan when < x <1
î 1 - 3x 2 3
æ3 ù
53. x > 0 54. x Î (–1/2, 1/2), x ¹ 0 55. çè , 2ú
2 û
21 25 p 1
56. , 57. 2n p + ; n ÎI 58. x = 0,
9 9 6 2
a-b æ 2 ù
59. x = 60. (cot2, ¥)È(–¥, cot3) 61. ç ,1ú
1 + ab è 2 ûú
1
62. æç tan ,cot1ùú 63. Infinite 64. zero
è 2 û
-2 6
65. 2 67. 68. 1
5
69. 9
SECTION - 5
70. D 71. C 72. A
73. D 74. A
SECTION - 6
-7
75. p 76. 77. A
17
1 1 17
78. 79. 80.
3 5 6
JPR\COMP.251\Allen(IIT-JEE Wing)\2020–21\Enthusiast\Mathematics\ITF\Unit Exercise
5p
81. –1 82. 83. –p
12
1
84. 85. 0 86. a
2
87. Infinite many solutions
*******
129
130
JEE-Mathematics
IMPORTANT NOTES